You are on page 1of 115

Academic Progress for every Student

Regular tests &


assignments

VEDANTU
IMPROVEMENT Report card with
PROMISE detailed analysis

VIP

Parent-teacher
meetings

Our Extraordinary Results 2021


Our Students

5.2X
HIGHER

Our Students
Our Students
3.8X
6.6X
HIGHER

All India HIGHER


All India All India

37.28% 37.28% 8.25%


9.84% 9.84% 1.25%

CBSE 10 CBSE 12 JEE


student scoring above 90% student scoring above 90% Advanced

SCAN CODE
to know more
about VIP
Published by
Vedantu Innovations Pvt. Ltd.
D. No. 1081, 3rd Floor, Vistar Arcade,
14th Main Rd, Sector 3, HSR Layout
Bangalore, Karnataka, India 560 102
www.vedantu.com

All rights reserved. No part of this book may be reproduced or utilized in any form or by any
means, electronic or mechanical, including photocopying, recording, or by any information
storage and retrieval system, without permission in writing from the publishers.
Notice: Vedantu is committed to serving students with the best resources and knowledge.
Bearing that in mind, we have obtained all the information in this book from sources regarded
as reliable, and taken utmost care in editing and printing this book. However, as authors and
publishers, we are not to be held responsible for unintentional mistakes that might have crept
in. Having stated that, errors (if any) brought to our notice shall be gratefully acknowledged
and rectified in upcoming editions.

Printed by
Softberry Technology Pvt. Ltd
2nd Floor, above PNB, Scheme No 140
Indore, Madhya Pradesh - 452016
https://www.softberry.in
How to use your
Tatva Practice Book

1
8. Quantization of charge
When a physical quantity can
have only discrete values rather
than any value, the quantity is
said to be quantised. The
smallest charge that can exist in Scan the QR Code in each
nature is the charge of an chapter’s theory section to
electron.
view micro concept videos
related to the chapter, on
SCAN CODE the Vedantu app.
Electrostatics

Exercise - 2:

2
Solve all types of
1. The rate constant for a first
order reaction is 4.606 ×
10–3s–1. The time required to
reduce 2.0g of the reactant to
0.2g is:
exercise questions (JEE 2020)
based on the latest JEE
pattern. (a) 500s (b) 1000s
(c) 100s (d) 200s

Answer Key

3
CHAPTER-1: ELECTROSTATICS
For extra exam
Exercise-1: Basic Objective preparation content,
visit the Vedantu app.
Scan the QR code in the You can find previous
Answers Section to view years’ JEE papers with
detailed solutions for all solutions and detailed
exercise questions. analysis.
Founder’s Message
Dear Student,
I am delighted to present to you a Ready Reckoner and an amazing book to guide you for your
exams-‘TATVA’. Tatva—which means the ‘Core’ is fully aligned with the culture, the mission, and the
vision of Vedantu and therefore it gives me immense pleasure and joy to share this book with you. We
at Vedantu have always believed in revolutionizing the teaching and learning process and always
speedily progressed in the direction of bringing superior quality education and content to your table.
Tatva is a step forward in this direction. This book is your guide, your practice guru, and your
companion in moving towards your dreams. The book is a result of the consistent effort, diligence, and
research by our experienced team of subject experts and teachers.
This book has been customized with curated content to suit the needs of JEE aspirants like you and
guide you on the right path to cracking JEE and optimizing your efficiency. Tatva is a comprehensive
amalgamation of important concepts, theories, derivations, definitions, solved examples, concept
videos, practice questions, and important questions. We have ensured that high-quality content and
the right ingredients are in place in this booklet to help you climb up the success ladder.
A few guiding points to optimally use Tatva with a planned approach:
Tatva equips you with Theory, Concept Videos, and Solved examples to help you revise concepts,
mark your notes, walk you through the entire summary, and eventually makes you capable of
clearing all your conceptual doubts all by yourself.
We suggest revision of theory followed by practice of solved examples.
Practice relevant questions daily after finishing Vedantu lectures and session assignments. We
believe that a daily dose of Tatva will keep all your exam blues at bay.
Use the Tatva booklet to mark notes so that it always comes in handy for last-minute revision
sessions before your exams. Notes should include key points of theory, solved examples, and some
questions which you couldn't solve in the first attempt.
Exercise 1 and Exercise 2 of JEE Tatva deal with basic questions and those which can be asked or
already asked in JEE Main. Similarly, Exercise 3 and Exercise 4 deal with JEE Advanced level
questions. We recommend you solve basic JEE Main questions before moving to JEE Advanced
level questions.
Before wrapping up, the practice mantra: “Don't practice until you get it right. Practice until you
can't get it wrong.”

We strongly believe in you and your capabilities. So believe in yourself


because success is only one step away. Wishing that your talent shines
bright. All the very best!

Anand Prakash
Founder and Academic Head, Vedantu

Anand Prakash Sir has been a pioneer in producing Top Ranks in JEE/NEET
and Olympiads. He has personally taught and mentored AIR 1, 6, 7 (JEE
Advanced), AIR-1, 7, 9(AIIMS), and thousands of more students who have
successfully cleared these competitive exams in the last few years.
Credits
“Happiness lies in the joy of achievement
and the thrill of creative effort.”
—Franklin D. Roosevelt

Tatva is the brainchild of a group of creative Vedans who have strived tirelessly to weave success stories for you.
We extend our heartfelt gratitude to the superb team of Vedans who give wings to the vision of Vedantu,
starting with our leaders who have been guiding and encouraging us at every step of the way:
Vamsi Krishna Sir, Anand Prakash Sir and Pulkit Jain Sir

We thank our leaders for their insight and mentorship. They steered the project in the right direction and were
instrumental in making Tatva a reality:
Sahil Bhatia, Sudhanshu Jain, Shubam Gupta, Ajay Mittal, Arshad Shahid, Jaideep Sontakke

The managers who embodied every aspect of what Tatva aimed to accomplish and brought their ideas and
diligence to the table to execute this vision immaculately:
Harish Rao, Neha Surana, Charubak Chakrabarti, Prashant Palande

Physics Team
We truly appreciate all the Master Teachers of Vedantu whose relentless efforts helped us translate this vision
into reality. Our heartfelt gratitude to our creative content developers and the typesetting team, who have put
in their hard work, insight, and eagerness to nurture and execute Tatva into ‘your ready handbook’ and bring a
positive learning experience to you.
Teachers Subject Matter Experts
Shreyas Hebbare Gagandeep Singh Atul Singh (Team Lead)
Anmol Gupta Md Fazlur Rehman

Typesetting Team Graphic Designers


Nitesh Kumar Santhosh Mani Vishwanath G

We cannot thank the creative team enough. Their creative minds and contagious energy have added a visual
flair, truly making Tatva the treasure trove of knowledge that it is.
Kajal Nilanjan Chowdhury Rabin Jacob Mohit Kamboj
Kiran Gopal Balaji Sakamuri Thamam Mubarish Haritha Ranchith
Sarib Mohammad

We thank and appreciate the enthusiastic support provided by Arunima Kar, Savin Khandelwal, and Dipshi
Shetty.

The journey of bringing Tatva to life, from an idea to the book you are holding, would not have been possible
without the extensive support of our diligent Operations Team, our amazing Academic Team, our dedicated
team of Teachers, and our talented Tech Team.
6

TABLE OF CONTENTS

THERMAL PHYSICS

Theory ................................................................................................................................................ 08

Solved examples ............................................................................................................................... 25

Exercise - 1 : Basic Objective Questions............................................................................................ 31

Exercise - 2 : Previous Year JEE MAIN Questions ........................................................................... 39

Exercise - 3 : Advanced Objective Questions .................................................................................. 41

Exercise - 4 : Previous Year JEE Advanced Questions ..................................................................... 48

Answer Key ........................................................................................................................................ 107


7

KINETIC THEORY OF GASES & THERMODYNAMICS

Theory ................................................................................................................................................ 53

Solved examples ............................................................................................................................... 71

Exercise - 1 : Basic Objective Questions............................................................................................ 76

Exercise - 2 : Previous Year JEE MAIN Questions ........................................................................... 84

Exercise - 3 : Advanced Objective Questions .................................................................................. 90

Exercise - 4 : Previous Year JEE Advanced Questions ..................................................................... 99

Answer Key ........................................................................................................................................ 109


THERMAL PHYSICS 8

THERMAL PHYSICS

SCAN CODE
THERMAL PHYSICS
Chapter 14
THERMAL PHYSICS 9

THERMAL PHYSICS

1. TEMPERATURE AND HEAT The zeroth law of thermodynamics states that if two
thermodynamic systems are each in thermal equilibrium with
1.1 Introduction to Temperature and Heat a third one, then they are in thermal equilibrium with each
Temperature : Temperature is a relative measure of hotness other.
or coldness of a body.
There are also various ways to state the zeroth law of
SI Unit : Kelvin (K) thermodynamics. However, in simple terms, it can be said,
Commonly Used Unit : °C or °F 'System that are in thermal equilibrium exist at the same
Conversion : t(k) = t°C + 273.15 temperature'.
Heat : Heat is a form of energy flow (i) between two bodies Zeroth law of thermodynamics takes into account that
or (ii) between a body and its surroundings by virtue of temperature is something worth measuring because it predicts
temperature difference between them. whether the heat will transfer between objects or not. This is
SI Unit : Joule (J) true regardless of how the objects interact. Even if two objects
are not in physical contact, heat still can flow between them,
Commonly Used Unit : Calorie (Cal)
by means of radiation mod of heat transfer. Whereas, zeroth
Conversion : 1cal = 4.186 J law of thermodynamics states that, if system are in thermal
Note: equilibrium, no heat flow will takes place
Heat always flows from a higher temperature system to a Thermal Equilibrium
lower temperature system. Temperature is a property that distinguishes thermodynamics
1.2 Zeroth law of thermodynamics from other sciences. This property can distinguish between
Zeroth law of thermodynamics is one of the four laws of hot and cold. When two or more bodies at different
thermodynamics. The credit for formulating the law goes to temperatures are brought into contact then after some time
Ralph H. Fowler. Interestingly, the zeroth law of they attain a common temperature and they are said to exist
thermodynamics was actually developed much later than the in thermal equilibrium.
original three laws. However, there was some confusion Systems are said to be in thermal equilibrium if there is no
regarding the nomenclature, whether it should be named the heat transfer, even if they are in a position to transfer heat,
fourth law of some other name. The complication arose based on other factors. For example, if we put food in the
because the new law gave a much clearer definition of the refrigerator overnight then that food is in thermal equilibrium
temperature and basically replaced what the other three laws with the air of that refrigerator. Heat no longer flows from
had to state. Fowler finally came up with the name to end this food to the air or from the air to the food, this state is known
conflict. as thermal equilibrium.
The zeroth law of Thermodynamics frames an idea of 1.3 Temperature Scales
temperature as an indicator of thermal equilibrium.
Measurement of Temperature
When a body 'A' is in thermal equilibrium with another body
Principle : Observation of Thermometric property with the
'b', and also separately in thermal equilibrium with a body 'C',
change in temperature and comparing it with certain reference
then body 'B' and 'C' will also be in thermal equilibrium with
situations.
each other. This statement defines the zeroth law of
thermodynamics. The law is based on temperature  Reference situation is generally ice point or steam point.
measurement.
Celcius and Fahrenheit Temperature Scales

In Celsius Scale In Fahrenheit Scale

Ice point  0°C Ice point  32°F


Steam point  100°C Steam point  212°F

Fig. 14.1

SCAN CODE
THERMAL PHYSICS
THERMAL PHYSICS 10

It implies that 100 division in celcius scales is equivalent to


180 scale divisions in fahrenheit scale. Thermometers
Instrument used to measure temperature of any system is called
t f  32 t
Hence  c as thermometer.
180 100
Examples : Liquid in Glass thermometer, Platinum Resistance
Thermometer, Constant Volume Gas Thermometers.
Liquid in Glass thermometer and Platinum Resistance
thermometer give uniform readings for ice point & steam point
but go non uniform for different liquids and different materials.
 Constant volume gas thermometer gives same readings
irrespective of which gas. It is based on the fact that at low
pressures and constant volume, P × T for a gas is constant.
Pressure
Gas A

Gas B

–273.15°C 0°C Temperature


(°C)
Fig. 14.2 Fig. 14.4
Absolute Temperature Scale  All gases converge to absolute zero at zero pressure.
It is kelvin scale
Ice point  273.15 K 2. THERMAL EXPANSION
Steam point  373.15 K
Most substances expand when they are heated. Thermal
Comparing it with the celcius scale, number of scale division expansion is a consequence of the change in average
in both the scales is same. separation between the constituent atoms of an object. Atoms
of an object can be imagined to be connected to one another
t c  0C t k  273.15
 by stiff springs as shown in Fig. At ordinary temperatures,
100 100 the atoms in a solid oscillate about their equilibrium positions
–11
 Kelvin scale is called as absolute scale, because it is with an amplitude of approximately 10 m. The average
– 10
practically impossible to go beyond 0 K into the negative spacing between the atoms is about 10 m. As the
side. temperature of solid increases, the atoms oscillate with greater
amplitudes, as a result the average separation between them
Steam
Point
373.15 K 100°C 212.0°F increases, consequently the object expands.

Ice
273.15 K 0°C 32°F
Point

Absolute
0K –273.15°C –459.67°F
zero
Kelvin Scale Celcius Scale Fahrenheit Scale

1K 1°C 1.8°F Fig. 14.5

Comparison of Temperature Scales


More precisely, thermal expansion arises from the
asymmetrical nature of the potential energy curve.
Fig. 14.3

SCAN CODE
THERMAL PHYSICS
THERMAL PHYSICS 11

At the atomic level, thermal expansion may be understood by as shown in figure, the average position of an atom will not
considering how the potential energy of the atoms varies be at the minimum point.
with distance. The equilibrium position of an atom will be at When the temperature is raised the amplitude of the vibrations
the minimum of the potential energy well if the well is increases and the average position is located at a greater
symmetric. At a given temperature, each atom vibrates about interatomic separation. This increased separation is manifested
its equilibrium position and its average position remains at as expansion of the material.
the minimum point. If the shape of the well is not symmetrical,
2.1 Linear, Areal and Volumetric Expansion

x
 KT constant (K)
x
Linear Expansion Coefficient of Linear expansion (a) :

L
L  a T Increase in length per unit length per
L
degree rise in temp.

Area Expension Coefficient of Area Expansion (b) :

A
 b T
A Increase in area per unit area per degree
rise in temp.

Volume Expansion Coefficient of volume expansion (g) :

V
 g T
V Increase in volume per unit volume per
degree rise in temp.
V

Units of a,b,g = /°C or /K


 In general with change in volume the density will also change.
 a for metals generally higher than a for non-metals
 gis nearly constant at high temperatures but at low temp it depends on temp.

Fig. 14.6
Coefficient of volume expansion of Cu as a function of temperature.

SCAN CODE
THERMAL PHYSICS
THERMAL PHYSICS 12

For ideal gases g is inversely proportional to temperature at From the figure it is observed that,
constant pressure CB = CA+AB
nRT Real expansion = Apparent expansion + expansion of the
V
P container
V T So, Vr = Va + Vc
 
V T Unlike solids, liquids have no fixed length or surface area but
1 always take up the shape of the containing vessel. When a
 g liquid is heated in a container, heat flows through the container
T
to the liquid; which means that the container expands first,
2.2 Relation between Real Expansion and Apparent
Expansion due to which the level of the liquid falls. When the liquid gets
heated, it expands more, beyond its unique level. We cannot
If the liquid is heated directly without using any container
monitor the intermediate state. We can only observe the initial
then the expansion that you observe is termed as a real
and final levels. This observed expansion of the liquid is
expansion of the liquid. The expansion of the liquid apparently
known as the apparent expansion of the liquid.
observed without considering the expansion of the container
is called the apparent expansion of the liquid. If we consider The real expansion of liquid = Apparent expansion of liquid
the expansion of the container also and measure the total + Volume expansion of the container.
expansion in the volume of the liquid, then the expansion is Therefore, in the case of liquids, we are concerned only with
termed as the absolute expansion of the liquid. volume changes when they are heated. The real (or absolute)
Explanation: A glass bulb with a long graduated stem is filled expansivity of a liquid is the fraction of its volume by which
with liquid up to the mark A. Now keeping an eye on the it expands per kelvin rise in temperature.
liquid column the bulb is heated and it is observed that the 2.3 Anomalous Expansion of Water
upper level of the liquid comes down from position A to
As an exception, water contracts on heating from 0°C to 4°C
position B. After that, it moves up from the graduated line B
and hence its density increases from 0°C to 4°C. Thus is called
crossing the mark A and reaches to mark C.
as anamolous expansion
The reason is that when the heat is applied the volume of the
1 gm/cc
bulb increases at first. Due to this cause liquid comes down
Density
to B from A. Later on as soon as the liquid gets heated its
volume starts to increase and reaches from B to C. It happens
so as the expansion of liquid is more than the expansion of
solid.
Apparently it appears to us that the liquid was at mark A and
4°C 4°C
finally reaches to mark C. Therefore, CA is the apparent (a) (b)
expansion. CB is the real expansion and AB indicates the Fig. 14.8 Fig. 14.9
expansion of the container.
Note:
 In general

3
g  3a  b
2
Proof : Imagine a cube of length, l that expands equally in
all directions, when its temperature increases by small T;
We have
l = alT
Also
3 3 3 2 2 2 3
V = (l l) – l = l + 3l l + 3ll + l – l
2
= 3l l ...(1)
Fig. 14.7

SCAN CODE
THERMAL PHYSICS
THERMAL PHYSICS 13

2 3
In Equation (1) we ignore 3ll & l as l is very small as
compared to l.
So

3V V
V  l = 3VaT [Using  l2 ] ...(2)
l l
V
 3aT
 V
Fig. 14.11
 g = 3a
For example, when the temperatures of a brass rod and a steel
Similarly we can prove for area expansion coefficient
rod of equal length are raised by the same amount from some
 In case, thermal expansion is prevented inside the rod common initial value, the brass rod expands more than the
by fixing its ends rigidly, then the rod acquires a compressive steel rod because brass has a greater average coefficient of
strain due to external fones at the ends corresponding stress expansion than steel. Such type of bimetallic strip is found in
set up in the rod is called thermal stress. practical devices such as thermostats to break or make
we know electrical contact.

V
 aT  compressive strain
V

YL
Also     Thermal stress
L

T  YaT
Where Y = young madulus of elasticity ...(3)
 Practical applications in railway tracks, metal tyres of
cart wheels, bridges and so many other applications.
Fig. 14.12
 If a solid object has a hole in it, what happens to the size
of the hole, when the temperature of the object increases. Variation of Density with Temperature
A common misconception is that if the object expands, ariation of density with temperature
the hole will shrink because material expands into the Most substances expand when they are heated, i.e. volume of
hole. But the truth is that if the object expands, the hole a given mass of a substance increases on heating, so the density
will expand too, because every linear dimension of an
object changes in the same way when the temperature  1
should decrease as  as    . Let us see how the density
changes.  V

 varies with increase in temperature.

m

V

1
or  (for a given mass)
V
Fig. 14.10  V V V 1
    
2.4 Applications of Thermal Expansion p V  V  t V  gT 1  gT
Expansion of a Bimetallic Strip
Each substance has its own characteristic average coefficient 
  
of expansion. 1  gT

SCAN CODE
THERMAL PHYSICS
THERMAL PHYSICS 14

This expression can also be written as Weight of the solid = Upthrust on solid from liquid

   1  gT   VS g  Vi l g ...(1)


–1

Here, S  density of solid


If g very small. 1  gT 
–1
 1 – gT
l  density of liquid
    1 – gT 
Vi  immersed volume of solid and
Effect of temperature on apparent weight when immersed
in a liquid V  Total volume of solid
When a solid body is complete immersed a liquid its apparent From Eqn. (1)
weight gets decreased due to an upthrust acting on it by the
Vi S
liquid. The apparent weight is given by  f ...(2)
V l
Wapp = W – F
where, f = immersed fraction of solid.
Here, F = upthrust =  VS L g
With increase in temperature S and l both will decrease.
Where, VS = volume of solid and L  density of liquid
Therefore, this fraction may increase, decrease or remain
Now, as the temperature is increased VS increases while L constant. At some higher temperature,
decreases. So, F may increase or decrease (or may remain S
constant also) depending upon the condiction that which factor f  ...(3)
l
dominates on the other. We can write.
From Eqs. (2) and (3), we have
F  VS L
f   S  l 
F VS L  VS  VS   1     
or      f  S  l 
F VS L VS  1  g L
T 

 V  g V T  1   1 
 S S S     1  g l  
 VS   1  g L T   1  g S
 

 1  g S T 
or F  F   f   1  g l  
 
 1  g L T  or f  1  gS  
Now, if g S  g L , F  F
Now, if g l  g S , f   f or immersed fraction will increase.
or   Wapp and vice-versa.
Waoo
If g l  g S , f   f or immersed fraction will remain unchanged
and if g S  g L , F  F
or   Wapp
Wapp and if, g l  g S , then f   f or immersed fraction will
decrease.
Effect of Temperature on Immersed Fraction of a Solid
Effect of temperature on the time period of a pendulum
in Floating condition
The time period of a simple pendulum is given by
When a solid, whose density is less than the density of liquid
is floating in it, then l
T  2
g

or T l
As the temperature is increased, length of the pendulum and
hence, time period gets increased or a pendulum clock
becomes slow and it loses the time.
Fig. 14.13

SCAN CODE
THERMAL PHYSICS
THERMAL PHYSICS 15

T l l  l F  YAaT
 
T l l Expansion of Liquids
Here, we l = l a in place of l aT so as to avoid the For heating a liquid it has to be ut in some container. When
confusion with change in time period. Thus, the liquid is heated, the container will also expand. We define
coefficient of apparent expansion of a liquid as the apparent
T l  l a increase in volume per unit origional volume per °C rise in
 1  a 
12

T l temperature. It is represented by g a . Thus,

 1  g r  coefficient of volume expansion of liquid


or T   T 1  a  (if a  1 )
 2  and g g  coefficient of volume expansion of the container

1
or T  T  – T  Ta
2
3. CALORIMETRY
Time lost in time t (by a pendulum clock whose actual time
period is T and the changed time period at some higher When two systems at different temperatures are connected
temperature is T  ) is together then heat flows from higher temperature to lower
temperature till the time their temperatures do not become
 T  same.
t   t
 T  Principle of calorimetry states that, neglecting heat loss to
 At some higher temperature a scale will expand and scale surroundings, heat lost by a body at higher temperature is
reading will be lesser than true value. equal heat gained by a body at lower temperature.

However, at lower temperature scale reading will be more heat gained = heat lost
or true value will be less. Whenever heat is given to any body, either its temperature
 When a rod whose ends are rigidly fixed such as to prevent changes or its state changes.
from expansion or contraction undergoes a change in 3.1 Change in Temperature
temperature, thermal stresses are developed in the rod.
This is because, if the temperature is increased, the rod When the temp changes on heating,
has a tendency to expand with since it is fixed at two Then
ends it is not allowed to expand. So, the rod exerts a force
Heat supplied  change in temp (T)
on supports to expand.
 amount of substance (m/n)
 nature of substance (s/C)
 H = msT
m = Mass of body
s = specific heat capacity per kg
T = Change in temp
Fig. 14.14 or H = nCT
n = Number of moles
l
Thermal strain   aT C = Specific/Molar heat Capacity per mole
l
T = Change in temp
So thermal stress   g  (thermal strain) YaT Specific Heat Capacity : Amount of heat required to raise
the temperature of unit mass of the substance through one
or force on supports F = A (stress) YAaT degree.
Here, Y = Young’s modulus of elasticity of the rod.

SCAN CODE
THERMAL PHYSICS
THERMAL PHYSICS 16

Units  In case any material is not at its B.P or M.P, then on heating
the temperature will change till the time a particular state
SI  J/KgK SH O L  = 1 cal/g°C
2
change temperature reaches.
Common  cal/g°C SH O ice  = 0.5 cal/g°C For Example : If water is initially at –50°C at 1 Atm pressure
2

in its solid state.


Molar Heat Capacity : Amount of heat required to raise the
On heating.
temperature of unit mole of the substance through one degree
Step - 1 : Temp changes to 0°C first
Units
Step - 2 : Ice melts to H2O(l) keeping the temp constant
SI  J/mol K
Step - 3 : Temp. increase to 100°C
Common  Cal/g°C
Step - 4 : H2O(l) boils to steam keeping the temp constant
Heat Capacity : Amount of heat required to raise the
temperature of a system through one degree Step - 5 : Further temp increases
 H = ST
where S = Heat Capacity
Units
SI  J/K
Common  Cal/°C
 For H2O specific heat capacity does change but fairly very
less.
 Materials with higher specific heat capacity require a lot
of heat for same one degree rise in temperature Fig. 14.15
3.2 Change in State  The slope is inversely proportional to heat capacity.
 Length of horizontal line depends upon mL for the process.
When the phase changes on heating
Then 3.3 Pressure dependence on melting point and boiling
point
Heat supplied  amount of substance which changes the state (m)
  nature of substance (L)  For some substance melting point decreases with increase
in pressure and for other melting point increases
 H = mL
 Melting poing increases with increase in temperature. We
Where L = Latent Heat of substance
can observe the above results through phaser diagrams.
Latent Heat : Amount of heat required per mass to change
the state of any substance. P P
B B
(atm) (atm)
Units
C C
Liq Liq
SI  J/Kg Solid
Solid
Common  Cal/g
O Vapour O Vapour
 The change in state always occurs at a constant
A A
temperature. T(°C) T(°C)
For H2O For CO2
For example

Solid  Liq Lf Fig. 14.16 Fig. 14.17


Line AO  Sublimation curve
Liq  Gas Lv
Line OB  Fusion curve
Lf = Latent Heat of fusion Line OC  Vapourization curve
Lv = Latent heat of vaporization Point O  Triple Point

SCAN CODE
THERMAL PHYSICS
THERMAL PHYSICS 17

Point C  Critical temperature


Triple Point : The combination pressure and temperature at
which all three states of matter (i.e. solids, liquids gases co-
exist.
For H2O it is at 273.16 K and 0.006 Atm.
Critical Point : The combination of pressure & temp beyond
which a vapour cannot be liquified is called as critical point.
Corresponding temperature, pressure are called as critical
temperature & critical pressure.
From the phasor diagram, we can see that melting point
decreases with increases in pressure for H2O.
Based on this is the concept of regelation.
Regelation : The phenomena of refreezing of water melted
below the normal melting point due to increase in pressure.
 It is due to this pressure effect on boiling point that cooking
is tough on mountains and easier in pressure cooker.
3.4 Mechanical equivalent of heat
In the history of science, the mechanical equivalent of heat
states that motion and heat are mutually interchangeable and
that in every case, a given amount of work would generate
the same amount of heat, provided the work done is totally
converted to heat energy. Fig. 14.18
Note:

HEAT TRANSFER
Water equivalent of a container
Normally, a liquid is heated in a container. So, some heat is
wasted in heating the container also. Suppose water
equivalent of a container is 10 g, then it implies that heat 1. HEAT TRANSFER
required to increase the temperature of this container is equal
to heat required to increase the temperature of 10 g of water. 1.1 Introduction to heat transfer
3.5 Calorimeter Heat transfer is the process of the movemnet of energy dut to
Calorimeter, device for measuring the heat developed during a temperature difference. The calculations we are interested
a mechanical, electrical, or chemical reaction, and for in include determining the final temperatures of materials and
calculating the heat capacity of materials. how long it takes for these materials to reach these
A calorimeter consists of an insulated container, water, a temperatures.
thermometer, a stirring rod, and an object that will either
1.2 Modes of Heat Transfer
absorb or emit heat. To do a Calorimetry experiment, an object
with a certain mass and temperature is placed in the water There are three modes of heat transfer.
and the change in the temperature measured .
 Conduction
A calorimeter is a device that is in use for measuring the
warmth of chemical reactions or physical changes also as heat  Convection
capacity. The most common types of calorimeters are
 Radiation
differential scanning calorimeters, titration calorimeters,
isothermal micro calorimeters, and accelerated rate
calorimeters.

SCAN CODE
THERMAL PHYSICS
THERMAL PHYSICS 18

2. CONDUCTION AND CONVECTION 2.2 Electrical Analogue of Thermal Conduction


Thermal Resistance (R)
2.1 Conduction

Thermal conduction is the process in which thermal energy is dQ T T


We know dt  H  l KA  R
transferred from the hotter part of a body to the colder one or
from hot body to a cold body in contact with it without any Here, T  temperature difference (TD) and
transference of material particles.
l
TC > T D R  thermal resistance of the rod.
TC TD KA
L
Consider a section ab of a rod as shown in figure. Suppose
A
Q1 heat enters into the section at ‘a’ and Q2 leaves at ‘b’, then
Direction of
heat flow Q2 < Q1. Part of the energy Q1 – Q2 is utilized in raising the
temperature of section ab and the remaining is lost to
Fig. 14.19 atmosphere through ab. If heat is continuously supplied fro
At steady state, the left end of the rod, a stage comes when temperature of
the section becomes constant. In that case Q1 = Q2 if rod is
The rate of heat energy flowing through the rod becomes insulated from the surroundings (or loss through ab is zero).
constant. This is called the steady state condition. Thus, is steady state
temperature of different section of the rod becomes constant
T  TD 
This is rate Q  KA
C
...(i) (but not same). Hence, in the figure:
L

for uniform cross-section rods


where Q = Rate of heat energy flow (J/s or W)
2
A = Area of cross-section (m )
TC,TD = Temperature of hot end and cold end respectively
(°C or K)
L = Length of the rod (m)
K = coefficient of thermal conductivity
Coefficient of Thermal Conductivity :
It is defined as amount of heat conducted during steady state
in unit time through unit area of any cross-section of the Fig. 14.20
substance under unit temperature gradient, the heat flow being T1 = constant, T2 = constant etc. and T1 > T2 > T 3 > T4
normal to the area.
Now, a natural question arises, why the temperature of
Units whole rod not becomes equal when heat is being
SI  J/msK or W/mK. continuously supplied? The answer is: there must be a
 Larger the thermal conductivity, the greater will be rate of temperature difference in the rod for the heat flow, same as
heat energy flow for a given temperature difference. we require a potential difference across a resistance for the
current flow through it.
 Kmetals > Knon metals
In steady state, the temperature varies linearly with
 Thermal conductivity of insulators is very low. Therefore,
distance along the rod if it is insulated.
air does not let the heat energy to be conducted very easily.

SCAN CODE
THERMAL PHYSICS
THERMAL PHYSICS 19

2.3 Series and Parallel Combination of Rods


Series Combination:
We can compute the effective coefficient of heat conductivity
when different materials are linked with varying coefficients
of thermal conductivity. When the rods are linked in series,
the flow rate through both sections is the same. However,
because the different connections get varying amounts of heat
energy, the temperature differential between them will be
varied.
The total of the temperature differences between the junctions
Fig. 14.21 equals the temperature difference between the first and last
ends. When two materials are linked in series and their
 Comparing equation number (iii), i.e. heat current physical dimensions are the same.
dQ T  I 
H   where R   2K1 K 2
dt R  KA  KS 
K1  K 2
with the equation, of current flow through a resistance,
dQ V  I  Parallel Combination:
I   where R  
dt R  A  When two materials are linked in a parallel configuration,
We find the following similarities in heat flow through a the entire available heat energy per second is divided between
rod and current flow through a resistance. them. In any case, the temperature will be the same at the
end. When the rods are linked in parallel, the effect on thermal
S.No. Heat flow Current flow
resistance is identical to the effect on electric resistance.
through a through a
conducting rod resistance We may construct a simple equation for the influence on
1. Conducting rod Electrical coefficient of heat conductivity when two distinct rods have
resistance the same length and cross-section area, as shown below.
2. Heat flows Charge flows
3. TD is required PD is required K1  K 2
4. Heat current Electric current Kp 
2
dQ dQ
H  rate of I  rate of Note:
dt dt
heat flow charge flow Low thermal conductivity materials transmit heat at a slower
5. T TD V PD pace than high thermal conductivity materials. Metals, for
H  I 
R R R R example, have a high thermal conductivity and are extremely
6. l l effective at transferring heat, whereas insulating materials
R R
KA A like Styrofoam are the polar opposite. High thermal
7. K=Thermal   electrical conductivity materials are commonly utilised in heat sink
conductivity conductivity applications, while low thermal conductivity materials are
used as thermal insulation. Thermal resistance is the
counterpart of thermal conductity.
From the above table it is evident that flow of heat through
rods in series and parallel is analogous to the flow of current 2.4 Growth of Ice in Lakes
through resistance in series and parallel. This analogy is of Warm water generally gets more dense as it gets colder, and
great importance in solving complicated problems of heat therefore sinks. This fact may lead you to believe that ice
conduction. should form on the bottom of a lake first. But a funny thing
happens to water as it gets even colder. Colder than 4° Celsius
(39° Fahrenheit), water begins expanding and becomes less

SCAN CODE
THERMAL PHYSICS
THERMAL PHYSICS 20

dense as it gets colder. As a result, close to freezing, colder George D. Ashton states, "As a lake cools from above 4° C,
water floats to the top and the warmer water sinks to the the surface water loses heat, becomes more dense and sinks.
bottom. The density of water as a function of temperature This process continues until all the water in the lake is at 4°
can be seen in the plot on the right. Eventually, the coldest C, when the density of water is at its maximum. With further
water, which has floated to the top of the lake in wintry cooling (and without mechanical mixing) a stable, lighter layer
conditions, freezes to form a layer of ice. Right when the of water forms at the surface. As this layer cools to its freezing
water freezes to ice, the ice becomes significantly less dense point, ice begins to form on the surface of the lake."
than the water and continues to float on the lake's surface.
In deep lakes, water pressure may also play a role. The
gravitational weight of all the water higher up in the lake
presses down on the water deep in the lake. The pressure
allows the water near the bottom of the lake to get cold without
expanding and rising. Because of the pressure, the water at
the bottom of deep lakes can become cold without freezing
to ice.

2.5 Convection
The process in which heat is transferred from one point to
another by the actual movement of the heated material particles
from a place at higher temperature to another place of lower
temperature is called as thermal convection.
 If the medium is forced to move with the help of a fan or
a pump, it is called as forced convection.
Fig. 14.22 If the material moves because of the differences in density
of the medium, the process is called natural or free
Ice is less dense than water because of the way it forms a
convection.
hexagonal crystalline structure. Each water molecule consists
of two hydrogen atoms bonded to the bottom of an oxygen  Examples of forced convection
atom. When ice forms, the hydrogen atoms of one water Circulatory system, cooling system of an automobile heat
molecule form weak hydrogen bonds with the top of the connector
oxygen atoms of two other water molecules.
 Examples of natural convection
Lining up the water molecules in this pattern takes up more
space than having them jumbled randomly together (as is the Trade winds, Sea Breeze/Land Breeze, Monsoons,
case in liquid water). And because the same mass of molecules Burning of Tea.
takes up more space when frozen, ice is less dense than liquid
water. For this same reason, water below 4° Celsius becomes
increasingly less dense as it gets colder. Close to freezing 3. RADIATION
temperatures, the molecules in the liquid water begin to line
up into the space-filling hexagonal structure. It is a process of transmission of heat in which heat travels
directly from one place to another without the agency of any
intervening medium.
 This radiation of heat energy occurs in the form of EM
waves.
 These radiators are emitted by virtue of its temperature,
like the radiation by a red hot iron or light from a filament
lamp.
 Every body radiates energy as well as absorbs energy from
surroundings.
Fig. 14.23

SCAN CODE
THERMAL PHYSICS
THERMAL PHYSICS 21

 The proportion of energy absorbed depends upon the colour 3.2 Ideal Black Body
of the body.
A body that absorbs all the radiation incident upon it and has
3.1 Basic Fundamental Terms in Radiation an emissivity equal to 1 is called a perfectly black body. A
black body is also an ideal radiator. It implies that if a black
Radiant Energy
body and an identical another body are kept at the same
All bodies radiate energy in the form of electromagnetic waves temperature, then the black body will radiate maximum power
by virtue of their temperature. This energy is called the radiant 4
as is obvious from equation P = eA also. Because
energy. e = 1 for a perfectly black body while for any other body
Absorptive Power ‘a’ e <1.
“It is defined as the ratio of the radiant energy absorbed by it Materials like black velvet or lamp black come close to being
in a given time to the total radiant energy incident on it in the ideal black bodiies, but the best practical realization of an
same interval of time.” ideal black body is a small hole leading into a cavity, as this
absorbs 98% of the radiation incident on them.
energy absorbed
a
energy incident
As a perfectly black body absorbs all radiations incident on
it, the absorptive power of a perfectly black body is maximum
and unity.

Spectral Absorptive Power ‘ a λ ’


The absorptive power ‘a’ refers to radiations of all wavelengths
(or the total energy) while the spectral absorptive power is
the ratio of radiant energy absorbed by a surface to the radiant
energy incident on it for a particular wavelength  . It may
have different values for different wavelengths for a given
surface. Let us take an example, suppose a = 0.6, a  = 0.4
Fig. 14.24
for1000 Å and a  = 0.7 for 2000 Å for a given surface. Then,
3.3 Kirchoff's law of Thermal Radiation
it means that this surface will absorb only 60% of the total
radiant energy incident on it. Similarly, it absorbs 40% of the Kirchhoff’s law
energy incident on it corresponding to1000 Åand 70% ‘’According to this law the ratio of emissive power to
absorptive power is same for all surfaces at the same
corresponding to 2000 Å.The spectral absorptive power a  is
temperature.’’
related to absorptive power a through the relation

a   a  d
0

Emissive Power ‘e’


(Do not confuse it with the emissivity e which is different
from it, although both have the same symbol e).
“For a given surface it is defined as the radiant energy emitted Fig. 14.25
per second per unit area of the surface.”
e1 e 2  e 
It has the units of W/m
2
Hence   
2 4
a1 a 2  a  perfactly black body
or J/s–m . For a black body e T =  .
But a  black body
1
Spectral Emissive Power ‘ e λ ’
and e E  say 
“It is emissive power for a particular wavelength  .” Thus, black body

e

d   e  d Then,    constant  E
0  a for any surface

SCAN CODE
THERMAL PHYSICS
THERMAL PHYSICS 22

Similarly, for a particular wavelength , 3.5 Newtons Law of Cooling


Newton’s Law of cooling states that, the rate of loss of heat
 e 
   E  dQ
 a  for any body of the body is directly proportional to the difference of
dt
Here, E = emissive power of black body at temperature T
temp between body and surrounding.
 T 4
From the above expression, we can see that dQ
Now  k  T2  T1 
e  a  dt
...(4)
3.4 Stefan's Law
The amount of radiation emitted per second per unit area by where k is a positive constant depending upon area and nature
a black body is directly proportional to the fourth power of of the surface of the body. Suppose a body of mass m, specific
its absolute temperature. heat capacity s is at temperature T2 & T1 be the temp of
Amount of radiation emitted E  T4 surroundings, if dT the fall of temperature in time dt.
where T = temperature of ideal black body (in K) Amount of heat lost is
E =  T4 dQ = msdT
This law is true for only ideal black body
 Rate of loss of heat is given by
SI Unit : E = watt/m2
 = Stefen's constant = 5.67 × 10–8 watt /m2 K4 dQ dT
 ms
Dimensions of  : M1 L0 T–3 –4 dt dt
Total radiation energy emitted out by surface of area A in ...(5)
time t : From Equation 4 and 5
Ideal black body QIBB =  A T 4 t and for any other body QGB dT
= erA T 4 t  ms  k  T2  T1 
dt
Rate of emission of radiation
When Temperature of surrounding T0 (Let T0 < T) dT k
  dt   Kdt
Rate of emission of radiation from ideal black body surface T2  T1 ms
E1 =  T4
k
Rate of emission of radiation from surrounding where K 
ms
E2 = T04
Net rate of loss of radiation from ideal black body surface is On integrating
E = E1 – E2 = T4–  T04 =  ( T4 – T04 ) log (T2 – T1) = –Kt + C
Net loss of radiation energy from entire surface area in time t –Kt C
or T2 = T1 + C1e where C1 = e ...(6)
is QIBB = A ( T4 – T04 ) t
For any other body QGB = er A ( T4 – T04 ) t equation (6) enables you to calculate the time of cooling of a
If in time dt the net heat energy loss for ideal black body is body through a particular range of temperature.
dQ and because of this its temperature falls by d

dQ
Rate of loss of heat RH = = σ A(T 4 – T04 )
dt
It is also equal to emitted power or radiation emitted per
second
Rate of fall in temperature (Rate of cooling)

dθ σA 4  dQ dθ 
 dt = m s J dt 
4
RF = dt = ms J (T – T0 )
 
Fig. 14.26

SCAN CODE
THERMAL PHYSICS
THERMAL PHYSICS 23

3.6 Wein's Displacement Law

At ordinary temperatures (below about 600°C) the thermal


radiation emitted by a body is not visible, most of it is
concentrated in wavelengths much longer than those of visible
light.

Fig. 14.27
 For small temp diff, the rate of cooling, due to conduction,
convection & radiation combined is proportional to
difference in temperature.
 Approximation : If a body cools from Ta to Tb in t times
in medium where surrounding temp is T0, then

Ta  Tb  T  Tb 
 K a  T0 
t  2 

 Newton’s Law of cooling can be verified experimentally.


Fig. 14.29
Figure shows how the energy of a black body radiation varies
T2 T1 with temperature and wavelength. As the temperature of the
black body increases, two distinct behaviors are observed.
The first effect is that the peak of the distribution shifts to
loge (T 2-T1)

shorter wavelengths. This shift is found to obey the following


relationship called Wien’s displacement law.
C
mT  b
V
Here, b is a constant called Wien’s constant. The value of this
t constant for perfectly black body in SI unit is 2.898×10–3 m-
(a) (b) K. Thus,

1
m 
Fig. 14.28 T
Set Up : A double walled vessel (v) containng water in Here,  m is the wavelength corresponding to the
between two walls.
maximum spectral emissive power e . The second effect
A copper calorimeter (C) containing hot water placed inside is that the total amount of energy the black body emits per
the double walled vessel. Two thermometers through the lids
unit area per unit time   T  increases with fourth
4
are used to note the temperature T2 of H2O in calorimeter and
T1 of water in between the double walls respectively. power of absolute temperature T. This is also known as the
emissive power. We know
Experiment : The temperature of hot water in the calorimeter

after equal intervals of time is measured. e   e d  Area under e –  graph  T 4
0

Result : A graph is plotted between log (T2 – T1) and time (t).
A 2   2  A1  16A1
2
or Area  T 4 
The nature of the graph is observed to be a straight line as it
should be from Newton’s law of cooling.

SCAN CODE
THERMAL PHYSICS
THERMAL PHYSICS 24

Thus, if the temperature of the black body is made two


fold,  m remains half while the area becomes 16 times.
3.7 Solar Constant
Solar constant, the total radiation energy received from the
Sun per unit of time per unit of area on a theoretical surface
perpendicular to the Sun’s rays and at Earth’s mean distance
from the Sun. It is most accurately measured from satellites
where atmospheric effects are absent.

Fig. 14.30

Fig. 14.31

SCAN CODE
THERMAL PHYSICS
THERMAL PHYSICS 25

SOLVED EXAMPLES
Example - 1 (d) The atmosphere serves purpose of a blanket over the
Burns from steam are usually more serious than those earth and it does not allow earth’s heat to be radiated
during night.
from boiling water. Why ?
(e) This is due to the fact that steam contains more heat in
Sol. For water, the latent heat of fusion and vaporisation are, the form of latent heat (540 calories/ gram) than water.
Lf = 3.33 × 105 J kg–1 and LV = 22.6 × 105 J respectively. It
means 3.33 × 105 J of heat are needed to melt 1 kg of ice at Example - 3
0°C and 22.6 × 105 J of heat are needed to convert 1 kg of Two slabs of thickness l1 and l2 and conductivities K1
water to steam at 100°C. So, steam at 100°C carries 540 cal/gm and K2 are in the thermal constant with each other as
more heat than water at 100°C. This is why burns from steam shown in the figure alongside. The temperature of their
are usually more serious than those from boiling water. outer surfaces are T 1 and T 2 respectively (T 1 > T 2).
Example - 2 Determine the temperature at their interface and the rate
of heat transfer through the slabs in the steady state
Explain why :
condition. Also find the equivalent conductivity of the
(a) a body with large reflectivity is a poor emitter.
system.
(b) a brass tumbler feels colder than a wooden tray in
l1 l2
winter.
(c) On optical pyrometry (for measuring high
temperatures) caliberated for an ideal black body
radiation, gives too low a value for the temperature
of a red hot iron piece in the open, but gives a correct
value for the temperature when the same piece is in
the furnace.
Sol. If T be the temperature at the interface, then the rate at
(d) the earth without its atmosphere would be which heat is transferred through slab ‘2’ is given by :
inhospitably cold.
d1 K1 A  T1  T 
(e) a heating system based on circulation of steam are  ...(i)
more efficient in warming a building than those based dt l1
on circulation of hot water.
d 2 K 2 A  T  T2 
and  ...(ii)
Sol. (a) The body with large reflectivity would be poor absorber dt l2
of radiations. Poor absorbers are poor radiators of In steady state,
radiation. Hence, body with large reflectivity is a poor
emitter. K1 A  T1  T  K 2 A  T  T2 

(b) Brass is a good conductor of heat. When we touch l1 l2
brass tumbler with our fingers, our body heat is quickly
K1T1l2  K 2 l1T2
conducted to the brass tumbler and hence temperature  T ...(iii)
K1l2  K 2 l1
of the finger tips is reduced. Thus, the brass-tumbler
feels colder. Putting the value of (iii) in equations (i) or (ii), we get
On the other hand, wood is a bad conductor, hence d A  T1  T2 
our body heat is not conducted to the wooden tray.  [ l1 = l2]
dt  l1 l 
(c) The temperature of red hot iron in the oven is given by   2 
 K1 K 2 
E 1 =  T 4. When iron is taken out in the open
temperature (T 0), its radiation energy is given by,
ES =  (T4 – T04). Thus, the pyrometer measures low
l +l2  2K 1 K 2
 K  K K
or K 1

values for the red hot iron in the open.  l1 l  1 2


  2 
 K1 K 2 
THERMAL PHYSICS 26
Example - 4 Example - 6
Two rods A and B are of equal length. Each ends at A blacksmith fixes iron ring on the rim of the wooden
temperatures T1 and T2. What is the condition that will wheel of a bullock cart. The diameter of the rim and the
ensure equal rates of flow of heat through the rods A and iron ring are 5.243 m and 5.231 m respectively at 27°C. To
B? what temperature should be ring be heated so as to fit the
rim of the wheel ?
Sol. Suppose that the two rods 1 and 2 have same temperature
difference T1 – T2 across their ends and the length of each Sol. Given, T1 = 27°C, L T = 5.231 m
1

rod is ‘d’.
L T = 5.243 m
2
For their same rate of heat condition, we have
So, L T = L T [1 + 1 (T2 – T1)]
K1 A1  T1  T2  K 2 A 2  T1  T2 
2 1

=  5.243 = 5.231 [1 + 1.20 × 10–5 °C–1 (T2– 27°C)]


d d
or T2 = 218°C.
 K1A1 = K2A2
Example - 7
A1 K 2 Two ideal gas thermometers A and B use oxygen and
 A 2 = K1 hydrogen respectively. The following observations are
made :
i.e., for the same rate of heat conduction, the area of cross-
Temperature Pressure Pressure
section of the two rods should be inversely proportional
thermometer A thermometer B
to their coefficients of thermal conductivity.
Triple point 1.250 × 105 Pa 0.200 × 105 Pa
Example - 5
of water
The difference between length of a certain brass rod and Normal melting 1.797 × 105 Pa 0.287 × 105 Pa
that of a steel rod is claimed to be constant at all point of sulphur
temperatures. Is this possible ?
(a) What is the absolute temperature of normal melting
Sol. Yes, it is possible. Let 1, and 2 be the coefficients of point of sulphur as read by the thermometer A and B ?
linear expansion and l1 and l2 be the lengths of brass and (b) What do you think is the reason for the slightly
steel rods respectively. Suppose that the temperature of different answers from A and B? (Thermometers are
the rod is increased by T. Then, change in length of brass not faulty). What further procedure is needed in the
rod, experiment to reduce the discrepancy between the
l1 = l1 1 T two readings ?
and change in the length of the steel rod,
Sol. (a) For thermometer A,
l2 = l2 2 T.
ptr = 1.250 × 105 Pa
If the difference between the lengths of the two rods is to
Ttr = 273.16 K
remain same, then
p = 1.797 × 105 Pa
or l1 = l2
and T=?
or l1 1 T = l2 2 T.
p tr p T
l1  Now, using   T  p tr
 2 Ttr T p tr
or l2 1

Thus, the lengths of the two rods should be in the inverse 1.797  105  273.16
=
ratio of their coefficients of linear expansion. 1.250  105

= 392.69 K
THERMAL PHYSICS 27

Example - 9
For the thermometer B,
A brass boiler has a base area of 0.15 m2 and thickness 1.0 cm.
p tr = 0.200 × 105 Pa, Ttr = 273.16 K
It boils water at the rate of 6.0 kg/min when placed on a
P = 0.287 × 105 Pa, T = ? gas stove. Estimate the temperature of the part of the
flame in contact with the boiler. Thermal conductivity of
Ttr 0.287  105 brass = 109 Js–1 m–1 °C–1. (Heat of vaporization of water
Ttr = p.   273.16
p tr 0.200  105 = 2256 × 103 J kg–1)

= 391.98 K Sol. Here, base area A = 0.15 m2

(b) The discrepancy between the two readings is due to Thickness, x = 1 cm = 1 × 10–2 m
the fact that the gases are not perfectly ideal gases. To K = 109 Js–1 m–1 °C–1
reduce the discrepancy, the readings should be taken L = 2256 × 103 J kg–1
at low pressures so that the gases could show perfect T = (T – 100)
behaviour. Here, T = temperature of the part of the flame in contact
with boiler.
Example - 8 Rate of boiling of water = 6 kg min–1. Rate at which heat is
being supplied to water is :
What is the temperature of the steel-copper junction in
the steady state of the system. Length of the steel rod  mL 6  2256  103
 
= 15.0 cm; length of the copper rod = 10.0 cm, temperature t t 60
of the furnace = 300°C, temperature of the other end
= 2.2556 × 105 Js–1
= 0°C. The area of cross-section of the steel rod is twice
that of the copper rod. [Theorem conductivity of steel   L 
Also, = KA  
= 50.2 Js –1 m–1 K–1; and of copper = 385 Js–1 m–1 K–1] t  x 
Steel
109  0.15   T  100 
=
Furnace Ice 1 102
(300°) 0°C
= 1635 (T – 100)
Insulating
Copper
material 
Equating the above two values of
t
Sol. Heat loss is reduced by placing insulating material around
the rods. In the steady state, the rate of heat flowing across 1635 (T – 100) = 2.256 × 105
a cross-section of the rod is the same at every point along
2.256  105
the length of the combined steel-copper rod. Let T be the  T – 100 =  138
1635
temperature of the steel-copper junction in the steady state.
 T = 138 + 100 = 238°C
K1 A1  300  T  K2 A2 T  0
Then, 
L1 L2
Example - 10
Where 1 and 2 refer to the steel and copper rods A clock which keeps correct time at 25°C has a pendulum
respectively. For A1 = 2A2, L1 = 15 cm, made of brass. How many seconds will it gain per day
L2 = 10 cm, when the temperature falls to the freezing point?
K1 = 50.2, K2 = 385, we have (brass = 18.7 × 10–6/°C)

Sol. Let l0 be the length of the pendulum at 0°C and l at 25°C


50.2  2  300  T  385T
 and let T0 and T be the corresponding time periods.
15 10
Solving it, we get, T = 44.4°C T l l 1  t 
Thus, T =  0  1  t
0 l0 l0
THERMAL PHYSICS 28

= 1  18.7  106  25 80  T   65  T 


or
0
 0

 65  T   55  T 
0 0

= 1  467.5  106 
1/ 2

or (80 – T0) (55 – T0) = (65 – T0)2


1 or 4400 – 55T0 – 80T0 + T02 = 4225 – 130T0 + T02
=  1 (467.5 × 10–6) = 1.00023375
2 or 5T0 = 175 or T0 = 35°C
But as the pendulum keeps correct time at 25°C, T = 1s and Example - 12
so at this temperature the pendulum makes (24 × 60 × 60 =
A ‘thermocol’ ice box is a cheap and efficient material for
86400) complete oscillations.
storing small quantities of cooked food in summer in
1 particular. A cubical ice box of side 30cm has a thickness
At 0°C, the time period is given by T0  s of 5.0 cm. If 4.0 kg of ice are put in the box, estimate the
1.00023375
amount of ice remaining after 6 h. The outside temperature
Number of oscillations made per day
is 45°C, and coefficient of thermal conductivity = 0.01 J/s
86400s mC°. Heat of fusion of water = 335 J/kg.

1/1.00023375  s Sol. We are given that
Number of seconds gained by the clock each of side of cubical box, L = 30 cm = 0.3 m
= 86420.2 s – 86400 s = 20.2 s thickness of each side, x = 5 cm = 0.05 cm
Example - 11 temperature difference between the outside and the inside
of the box,
A body takes 5 minutes to cool from 80°C to 65°C and 10
minutes to cool to 55°C. Find the temperature of the T1 – T2 = 45 – 0 = 45°C
surroundings. time for which heat flow is considered
Sol. In the first case, T1 = 80°C, T2 = 65°C, t = 5 min t = 6 h = (6 × 60 × 60)s = 21600 s
Let T0 be the temperature of the surroundings thermal conductivity of thermocole, k = 0.01 J/s mC°
heat of fusion of water, L = 335 J/g
T  T 
From ln
1
 Kt, we get
0
surface area of the box
T  T 
2 0
= 6 (0.30 m × 0.30 m) = 0.54 m2
If m is the mass of ice that melts in 6 h,
80  T0 
ln 65  T  5K ...(i)
 0
Q = mL ...(i)

In the second case, kA  T1  T2  t


Also, Q  ...(ii)
x
T1 = 65°C, T2 = 55°C, t = 10 min – 5 min = 5 min
From equations (i) and (ii), we get
 T1  T0 
From ln T  T  Kt, we get kA  T1  T2  t
 2 0 mL 
x

 65  T0  kA  T1  T2  t
ln 55  T  5K ...(ii) or m
 0
Lx

From equation (i) and (ii), 0.01 J / smC   0.54 m 2   45 C   21600s 


or m
 335 J / g   0.05m 
 80  T 
0  65  T  0
ln = ln
 65  T 
0  55  T  0
= 313 g = 0.313 kg
Mass of ice left after 6 h = 4 kg – 0.313 kg
= 3.687 kg = 3.7 kg
THERMAL PHYSICS 29

Example - 13 Ans. (c)


4 4
From what height must a block of ice be dropped in order E 2 eA 2 T24  A 2   T2  2  800 
Sol.       
that it may melt completely. It is assumed that the whole E1 eA1T1  A1   T1 
4
1  400 
of energy is retained by ice. E2 = 32 E1
(Latent heat of ice = 3.33 × 10 J/kg)5
= 4800 W
Sol. Let mass of ice block = m (kg) E = 4.8 KW
Height through which it is dropped = h (m) Example - 16
PE of the block, W = mgh (J) The area of a hole of heat furnace is 10–4 m2. It radiates
Since it is assumed that whole of this energy is retained by l.58 × 105 cal of heat per hour. If the emissivity of the furnace
ice and is converted into heat Q, is 0.80, then its temperature is :

Q = mgh (J) ...(i) (a) 1500 K (b) 2000 K


(c) 2500 K (d) 3000 K
If this heat is to melt the ice, then
Ans. (c)
Q = (m kg) × (3.33 × 105 J/kg) = 3.33 × 105 m(J) ...(ii)
Sol. E  eAT 4
From equation (i) and (ii),
mgh = 3.33 × 105 m 1.58 105  4.2
 0.8  5.67  108  104  T 4
60  60
3.33  105
or h=
9.8
or h = 3.4 × 104 m T = 2524  2500 K

Example - 14 Example - 17
If the total surface area of human body is 1.2 m2 and the
The temperature of body is increased from 27°C to 127°C.
temperature is 30°C, then the net rate of radiation from the
The radiation emitted by it increases by a factor of :
body if surrounding temperature is 20°C would be [Take
(a) 256/81 (b) 15/9
emissivity of human body = 1]
(c) 4/5 (d) 12/27
(a) 574 W (b) 72 W
Ans. (a)
(c) 800 W (d) 60 W
Sol. E  eAT 4
Ans. (b)

E  eA  T 4  Ts4 
4
E 2  T2  Sol.
 
E1  T1 
E  1  5.67  108  1.2   3034  2934 
4
E 2  400 
  E = 72 W
E1  300 
Example - 18
256 A solid sphere cools at the rate of 2.8°C/min, when its
E2  E1
81 temperature is 127°C. The rate at which another solid sphere
of same material of twice the radius will lose its temperature
Example - 15
at 327°C is given by [Take the surrounding temperature at
A sphere has a surface area of 1.0 m2 and a temperature of 27°C]
400 K and the power radiated from it is 150 W. Assuming
(a) 6.8 °C/min (b) 5.6 °C/min
the sphere is a black body radiator, the power in kilowatt
radiated when the area expands to 2.0m2 and the temperature (c) 9.72 °C/min (d) 8.4 °C/min
changes to 800 K is : Ans. (c)
(a) 6.2 (b) 9.6
(c) 4.8 (d) 16
THERMAL PHYSICS 30

Sol. Heat lost per second = msdT

4 3 
Heat lost by first sphere   r   s  2.8 ...(i)
3 

From Stefan’s law, heat lost  A  T  Ts  ...(ii)


4 4

From equation (i) and (ii)

4 3 
 r   s  2.8   4r  400    300  
2 4 4

3 

2.8
rs    400    300   ...(iii)
4 4

3  
For second phase

2 dT
   600    300   ...(iv)
4 4
rs
3 dt  
Divided equation (iv) by equation (iii)

 600    300 
4 4
dT 2
   9.72 C/ min
dt 2.8  400 4   300 4
THERMAL PHYSICS 31

EXERCISE - 1 : BASIC OBJECTIVE QUESTIONS


Heat, Temperature and Calorimetry 6. The radius of a ring is R and its coefficient of linear expansion
Temperature and Heat is . If the temperature of ring increases by , then its
1. At absolute zero circumference will increase by :
(a) all substances exist in solid form (a)  R (b) 2 R

(b) molecular motion ceases  


(c)  R (d)  R
(c) water freezes 2 4
(d) None of the above 7. A steel wire of cross–sectional area 0.5 mm2 is held between
2. At critical temperature, the surface tension of a liquid is : two fixed supports. If the wire is just taut at 20°C, determine
(a) zero the tension when the temperature falls to 0°C. Coefficient
of linear expansion of steel is 1.2 × 10–5/°C and its Young’s
(b) infinity
modulus is 2.0 × 1011 N/m2.
(c) the same as that at any other temperature
(a) 24 N (b) 36 N
(d) cannot be determined
(c) 12 N (d) 6 N
3. The graph between two temperature scales A and B is
shown in figure. 8. A solid ball of metal has a spherical cavity inside it. If the
ball is heated the volume of cavity will :
(a) increase (b) decrease
(c) remain unchanged (d) data insufficient
Thermal Expansion
9. An iron ball of mass 0.2 kg is heated to 10°C and put into a
block of ice at 0°C. 2.5 g of ice melts. If the latent heat of
fusion of ice is 80 cal/g, then the specific heat of iron in
cal/g°C is :
(a) 1 (b) 0.1
(c) 0.8 (d) 0.08
Between upper fixed point and lower fixed point, there are
150 equal divisions on scale A and 100 on scale B. The 10. Equal masses of three liquids A, B and C have temperatures
relationship for conversion between the two scales is given 10°C, 25°C and 40°C respectively. If A and B are mixed,
by : the mixture has a temperature of 15°C. If B and C are mixed,
the mixture has a temperature of 30°C. If A and C are mixed,
t A  180 t B t A  30 t the mixture will have a temperature of:
(a)  (b)  B
100 150 150 100 (a) 16°C (b) 20°C
(c) 25°C (d) 29°C
t B  180 t A t B  40 t A
(c)  (d)  11. The temperatures of equal masses of three different liquids
150 100 100 180
A, B and C are 12°C, 19°C and 28°C respectively. The
4. The reading of Centigrade thermometer coincides with that
temperature when A and B are mixed is 16°C, and when B
of Fahrenheit thermometer in a liquid. The temperature of
and C are mixed, it is 23°C. What will be the temperature
the liquid is :
when A and C are mixed ?
(a) –40°C (b) 0°C
(a) 15.6 °C (b) 23.2 °C
(c) 100°C (d) 300°C
(c) 20.3 °C (d) 25.8 °C
5. Which of the following parameters does not characterize
the thermodynamic state of matter? 12. 50 g of ice at 0°C is mixed with 50 g of water at 60°C, final
(a) temperature (b) pressure temperature of mixture will be :

(c) work (d) volume (a) 0°C (b) 40°C


(c) 10°C (d) 15°C
THERMAL PHYSICS 32

13. Two metal rods A and B are having their initial length in 19. Steel wire of length L at 40°C is suspended from the ceiling
the ratio 2 : 3 and the co-efficients of linear expansion in and then a mass m is hung from its free end. The wire is
the ratio 3 : 4. When they are heated through the same cooled down from 40°C to 30°C to regain its original length
temperature difference, the ratio of their linear expansion L. The coefficient of linear thermal expansion of steel is
is 10–5/°C. Young’s modulus of steel is 1011N/m2 and radius
(a) 3 : 4 (b) 1 : 2 of the wire is 1 mm. Assume that length (L) >> diameter
9d) of the wire. Then the value of ‘m’ (in kg to nearest,
(c) 2 : 3 (d) 4 : 3
integer).
14. A bi-metallic strip is made of two strips A and B having
co-efficients of linear expansion A and B. If A <B, 20. Two rods of different materials having coefficients of
then on heating the strip will thermal expansion 1 ,  2 and Young’s moduli Y1, Y2
(a) bend with A on outer side respectively are fixed separately between two rigid walls.
(b) bend with B on outer side The rods are heated such that they undergo the same
increase in temperature. There is no bending of the rods.
(c) not bend at all
If 1 :  2 = 2 : 3; then thermal stress developed in both
(d) None of these
15. Two rods of different materials having coefficients of Y1
rods would be equal, if Y is
thermal expansion 1 and 2 and Young’s moduli Y1 and 2

Y2 respectively are fixed between two rigid massive walls.


The rods are heated such that these undergo the same 21. As a result temperature rise of 32°C, a bar with a crack at
increase in temperature. There is no bending of the rods. its center buckles upwards. If the fixed distance L0 is 4 m,
If 1 : 2 = 2 : 3, the thermal stresses developed in the two and the coefficient of linear expansion of bar is 25 × 10–6
rods are equal, provided Y1 : Y2 is °C–1. Find the rise x (in cm, to the nearest integer) of the
center.
(a) 2 : 3 (b) 1 : 1
(c) 3 : 2 (d) 4 : 9
16. Which of the following qualities are best suited for a
cooking utensil
(a) high specific heat and low thermal conductivity
(b) high specific heat and high thermal conductivity
(c) low specific heat and low thermal conductivity
(d) low specific heat and high thermal conductivity
17. What should be the lengths of steel and copper rods at
0°C so that the length of the steel rod is 5 cm longer than
the copper rod at any temperature? 22. Two vessels connected at the bottom by a thin pipe with a
 (Steel) = 1.1 × 10–5 °C–1 and sliding plug contain liquid at 20°C and 80°C respectively.
The coefficient of cubic expansion of liquid is 10–3 K–1.
 (Copper) = 1.7 × 10–5 °C–1
(a) 14.17 cm, 9.17 cm (b) 9.17 cm, 14.17 cm  h 20 
The ratio of heights columns in the vessel  h  is nearest
(c) 28.34 cm, 18.34 cm (d) 14.17 cm, 18.34 cm  80 

18. The radius of a metal sphere at room temperature T is R, to which integer?


and the coefficient of linear expansion of the metal is . 23. The area of cross - section of a railway track is 0.01 m2.
The sphere is heated a little by a temperature T so that its The temperature variation is 10°C. Coefficient of linear
new temperature is (T + T). The increase in the volume expansion of material of track is 10–5/°C. The energy stored
of the sphere is approximately : per meter in the track is _____ J/m.
(a) 2R T (b) R2 T (Young’s modulus of material of track is 1011 Nm–2)
(c) 4R T/3
3
(d) 4R T
3
THERMAL PHYSICS 33

Calorimetry 31. Three copper blocks of masses M 1, M 2 and M 3 kg,


24. Relation between molar and principal specific heat of gases respectively are brought into thermal contact till they reach
(a) Cp = Mcp (b) Cp = M + cp equilibrium. Before contact, they were at temperatures T1,
T2, T3 (T1 > T2 > T3). Assuming there is no heat loss to the
(c) cp = MCp (d) Cp = M – cp
surroundings, the equilibrium temperature T is :
25. Liquids at temperature 60°C and 20°C, respectively, have
mass ratio 3 : 4 and their specific heats in the ratio 4 : 5. If (s is specific heat of copper)
the two liquid mixed, the resultant temperature
T1  T2  T3
(a) 70°C (b) 0°C (a) T 
3
(c) 35°C (d) 40°C
26. Heat given to a body which raises its temperature by 1°C M1T1  M 2 T2  M 3 T3
is (b) T 
M1  M 2  M 3
(a) water equivalent (b) thermal capacity
(c) specific heat (d) temperature gradient M1T1  M 2 T2  M 3 T3
(c) T 
27. If m is the mass,  is temp. and ‘s’ is specific heat, then 3  M1  M 2  M 3 
thermal capacity K is given by
(a) K = ms  (b) K = m  M1T1s  M 2 T2 s  M 3 T3s
(d) T 
ms M1  M 2  M 3
(c) K  (d) K = ms

32. In two experiments with a continuous flow - calorimeter
28. Dimensions of latent heat are to determine the specific heat capacity of a liquid, an input
(a) [M1 L2 T–2] (b) [M0 L2 T–2] power of 16 W produced a rise of 10 K in the liquid.
(c) [M L T ]
1 1 –1
(d) [M1 L 1 T–2] When the power was doubled, the same temperature rise
was achieved by making the rate of flow of liquid three
29. Equal masses of two liquids A and B contained in vessels
times faster. Find the power lost (in W) to the surroundings
of negligible heat capacity are supplied heat at the same
in each case, assuming that heat lost to surroundings
rate. The (temperature vs time) graphs for the two liquids
depends on temp of liquid and atmospheric temp. only.
are shown in figure. If S represents specific heat and L
represents latent heat of liquid, then 33. A copper ball of mass 100 gm is at a temperature T. It is
dropped in a copper calorimeter of mass 100 gm filled
with 170 gm of water at room temperature. Subsequently,
the temperature of the system is found to be 75° C. (given
room temperature = 30°C, specific heat of copper = 0.1
cal/gm°C. Temperature T (in °C) is:
34. Two liquids A and B are at 32° C and 24°C respectively.
When mixed in equal masses the temperature of the mixture

 SA 
if found to be 28°C. The ratio of specific heats  S  is
 B
(a) SA > SB ; LA < LB (b) SA > SB ; LA > LB
(c) SA < SB ; LA < LB (d) SA < SB ; LA > LB
cal
30. If mass-energy equivalence is taken into account, when water 35. A 50 gm lead bullet (specific heat 0.020 gC ) is initially
is cooled to form ice, the mass of water should
at 30°. It is fired vertically upwards with a speed of 84 m/
(a) increase
s. On returning to the starting level, it strikes a slab of ice
(b) remain unchanged
kept at 0°C. (A × 100) mg of ice is melted due to this. Find
(c) decrease the value of ‘A’.
(d) first increase then decrease
(Take:- Lice = 80 cal/gm and 1 cal = 4.2 J)
THERMAL PHYSICS 34

36. A liquid at 30°C is poured very slowly into an open 43. Two metal rods A and B of equal lengths and equal cross
Calorimeter that is at temperature of 110°C. The boiling sectional areas are joined end-to-end. The co-efficients of
temperature of the liquid is 80°C. It is found that the first thermal conductivity of A and B are in the ratio 2 : 3. When
5 gm of the liquid completely evaporated. After pouring the free end of A is maintained at 100°C and the free end
another 80 gm of the liquid the equilibrium temperature is of B is maintained at 0°C, the temperature of the junction
found to be 50 °C. The ratio of the Latent heat of the liquid is
to its specific heat will be ____ °C. (Neglect the heat (a) 30°C (b) 40°C
exchange with surrounding)
(c) 50°C (d) 60°C
37. How much heat is required to convert 8.0 g of ice at –
44. In steady state
15°C to steam at 100°C? (Given, Cice = 0.53 cal/g–°C, Lf
= 80 cal/g and Lv = 539 cal/g, and cwater = 1 cal/g–°C) (a) temperature does not change with time

Give the answer in kcal correct to one decimal place. (b) all parts of the body are at same temperature

38. The temperature of equal masses of three different liquids (c) there is no flow of heat
A, B and C are 12°C, 19°C and 28°C respectively. the (d) all of the above
temperature when A and B are mixed is 16° C and when B 45. Two metallic plates of equal thicknesses and thermal
and C are mixed it is 23°C. What should be the temperature conductivities K1 and K2 are put together face to face and
when A and C are mixed (in °C, correct to two decimal a common plate is constructed, figure. The equivalent
places)? thermal conductivity will be:
39. An ice cube of mass 0.1 kg at 0°C is placed in an isolated
k1 k2
container which is at 227°C. The specific heat s of the
container varies with temperature T according to the l l
empirical relation s = A + BT, where A = 100 cal/kg-K and
K1 K 2 2K1 K 2
B = 2 × 10–2 cal/kg-K2. If the final temperature of the (a) K  K (b) K  K
container is 27°C, determine the mass of the container (in 1 2 1 2

kg, correct to one decimal place). (Latent heat of fusion


K  K 22 
2 3/ 2

for water = 8 × 104 cal/kg, specific heat of water = 103 cal/ K1  K 2 1


(c) (d)
kg-K). 2 2K1 K 2
40. m grams of steam at 100°C is mixed with 200g of ice at its 46. Two metallic plates of equal lengths and thermal
melting point in a thermally insulated container. If it conductivities k1 and k2 are put together such that their
produces liquid water at 40°C [heat of vaporization of ends coincide. If their cross-sectional areas are the same,
water is 540 cal/g and heat of fusion of ice is 80 cal/g], the then the equivalent thermal conductivity of the combination
value of m (in grams) is _______. will be :
41. 1 kg ice at –10°C is mixed with 0.2 kg of steam at 200°C.
If final temperature (°C) of mixture at equilibrium is A k1

A k2
 255 
Teq    , then fill the value of x.
 x 
k1 k 2 2k1k 2
Latent heat of fusion of ice = 80 cal/gram, latent heat of (a) k  k (b) k  k
1 2 1 2
vaporization of water = 540 cal/gram, specific heat of water
= 1 cal/gm K
k1  k 2
specific heat capacity of ice  specific heat of steam = (c) (d) k1 k 2
2
0.5 cal/gram-K.
47. The ends of a copper rod of length 1m and area of cross
Heat Transfer 2
section 1 cm are maintained at 0°C and 100°C. At the
Heat Transfer, Conduction and Convection centre, power is supplied at a constant rate of 25 J/s. The
42. Dimension of co-efficient of thermal conductivity are temperature gradient on higher temperature side of the rod
(a) [L0M1T–3K–1] (b) [L1M1T–3K–1] in steady state (in °C/m) will be
(c) [L1M1T–3K] (d) [L1M–1T–2K–1] (K = 400 J/m-K-s)
THERMAL PHYSICS 35

48. Three rods made of same material and having same cross 51. The temperature of the two outer surface (end surfaces
- section have been joind as shown in the figure. Each rod shown) of a composite slab, consisting of two materials
is of same length. The left end is kept at 0°C and both the having coefficients of thermal conductivity K and 2K,
right ends are kept at 90°C. The temperature of the junction thickness ‘x’ and ‘4x’ are T2 and T1(T2 > T1). The rate of
of the three rods (in degree Celsius) will be heat transfer through the slab in steady state is

 A  T2 – T1  K  1
  , where ‘f’ is equal to
 x f
(all quantities measured in S.I unit)

49. Two rectangular blocks, having identical dimensions, can


be arranged either in configuration I or in configuration II
as shown in the figure. One of the blocks has thermal
conductivity K and other 2K. The temperature difference 52. Two conducting cylinders of equal length but different radii
between the ends along the x-axis is the same in both the are connected in series between two heat baths kept at
configurations. It takes 9 s to transport certain amount of temperature T1 = 300 K and T2 = 100 K, as shown in the
heat from the hot end in the configuration I. The time (in figure. The radius of the bigger cylinder twice that of the
seconds) to transport the same amount of heat in the smaller one and the thermal conductivities of the materials
configuration II is: of the smaller and the larger cylinders are K1 and K2
respectively. If the temperature at the junction of the two
cylinders in the steady state is 200 K, then K1/K2 =
_______.

50. Figure shows a metal rod of uniform cross section area A,


with variable thermal conductivity given by

  
k  x   k 0 sec  x  . If the end A is maintained at
 6L 
temperature T0 , the rod carries a thermal current I0 (from
Radiation
I0 L 
B to A) in steady state and k AT  3 ; find the 53. A sphere, a cube and a thin circular plate, all of same
0 1 material and same mass, are initially heated to same high
temperature of the end B of the rod. Let’s say this temperature. Choose the correct statement.
temperature is kT0, find integer value k. (a) The plate will cool fastest and cube the slowest.
(b) The sphere will cool fastest and cube the slowest.
(c) The plate will cool fastest and sphere the slowest.
(d) The cube will cool fastest and plate the slowest.
54. Velocity of heat radiation v as related to the velocity of
light c is
(a) v > c (b) v = c
(c) v < c (d) no definite relation
THERMAL PHYSICS 36

55. Heat is transferred most rapidly by the process of 59. A body cools from 50°C to 46°C in 5 minutes and to 40°C
(a) Conduction (b) Convection in the next 10 minutes. The surrounding temperature is :
(c) Radiation (d) Combustion (a) 30°C (b) 28°C
56. Two circular discs A and B with equal radii are blackened. (c) 36°C (d) 32°C
They are heated to same temperature and then cooled under
60. Newton’s law of cooling is used in laboratory for
identical conditions. What inference do you draw from
determining:
their cooling curves as shown in figure?
(a) Specific heat of gases (b) Specific heat of liquids
B (c) Latent heat of gases (d) Latent heat of liquids
61. If the rate of change of temperature is 0.2°C/ min and
R A excess temperature of a body over surrounding is 20°C,
the constant of proportionality is
(a) 0.1 (b) 0.01
( – 0) (c) 1 (d) 0.001

(a) A and B have same specific heats 62. Newton’s law of cooling is applicable for
(b) specific heat of A is less (a) Any excess of temperature over the surrounding
(c) specific heat of B is less (b) Small excess of temperature over the surrounding
(d) nothing can be said (c) Large excess of temperature over the surrounding
57. The temperature of coffee in a cup with time is most likely (d) Very large excess of temperature over the surrounding
given by the curve in figure.
63. Newton’s law of cooling leads us to the following
expression,
Temperature

Temperature

(a) ( – 0) = Kt + C (b) log ( – 0) = Kt + C


(c) log  = Kt + C (d)  = K0 + C
(a) (b)
64. Two spheres of the same material have radii 1 m and 4 m
Time Time and temperatures 4000 K and 2000 K respectively. The
ratio of the energy radiated per second by the first sphere
to that by the second is
Temperature

Temperature

(a) 1 : 1 (b) 16 : 1
(c) (d) (c) 4 : 1 (d) 1 : 9
65. Which of the following is more close to a black body ?
Time Time
(b) Black board paint (b) Green leaves
58. A block of steel heated to 100°C is left in a room to cool.
Which of the curves shown in the figure represents the (c) Black holes (d) Red roses
decrease of temperature with time? 66. Infrared radiations are detected by
(a) spectrometer (b) pyrometer
D
C (c) nanometer (d) photometer
Temperature

B
A

Time
(a) A (b) B
(c) C (d) D
THERMAL PHYSICS 37

67. A liquid in a beaker has temperature (t) at time t and 0 is 71. A piece of metal is heated to temperature  and then
temperature of surroundings, then according to Newton’s allowed to cool in a room which is at temperature 0. The
law of cooling, the correct graph between loge ( – 0) and graph between the temperature T of the metal and time t
will be closed to :
t is :

T
T
(a) (b)

O t O t

T T
(c) (d)
O t O t

72. If the temperature of the sun were to increase from T to 2T


and its radius from R to 2R, then the ratio of the radiant
energy received on earth to what it was previously, will be
(a) 4 (b) 16
(c) 32 (d) 64
73. In the figure, the distribution of energy density of the
radiation emitted by a black body at a given temperature
is shown. The possible temperature of the black body is
68. The maximum wavelength of radiations emitted at 900 K 3
n × 10 K. Write n as the nearest integer.
is 4 m. What will be the maximum wavelength of –3
(Take:- b = 2.898 × 10 mK)
radiations emitted at 1200K:
(a) 3 m (b) 0.3 m
(c) l m (d) none of these
69. A bucket full of hot water is kept in a room and it cools
from 75°C to 70°C in T1 minutes, from 70°C to 65°C in T2
minutes and from 65°C to 60°C in T3 minutes then
(a) T1 = T2 = T3 (b) T1 < T2 > T3
(c) T1 > T2 > T3 (d) T1 < T2 < T3
70. Assuming the sun to be a spherical body of radius R at a
temperature of T K, evaluate the total radiant power,
incident on earth, at a distance r from the sun, where r0 is
radius of earth 74. Two starts (say S1 and S2) emit their maximum intensity
–6 –7
radiations at wavelengths 10 m and 0.4 × 10 m
4r02 R 2 T 4 r02 R 2 T 4
(a) (b)  T2 
r2 r2 respectively. The ratio of their surface temperature  T 
 1
r02 R 2 T 4 R 2 T 4 is
(c) (d)
4r 2 r2
THERMAL PHYSICS 38

75. A body cools in 7 minutes from 60°C. What will be its 78. A glass of boiling water at 100°C cools down to 90°C in
temperature (in °C) after the next 7 minutes? The 10 minutes when placed in surrounding temperature of
temperature of surroundings in 10°C. 30°C. It will cool down to 80°C in an additional time
76. A solid cube of side a, density d and specific heat ‘s’ is at nx – n  x –1
t  10
temperature 400 K. It is placed in an ambient temperature n  x  1 – nx minutes. Assume Newton’s law of
of 200 K.
–3 3 3 cooling to hold. Find x
Take: a = 0.9 m, d = 4.8 ×10 kg/m , s = 2.0 × 10 J/kg K.
79. A body cools in 7 minutes from 60°C to 40°C. What will
Stefan’s constant   6  10 –8 W K 4 m 2 . Consider the be its temperature in °C after next 7 minutes? The tem-
cube to be a black body. If the time for the temperature of perature of surrounding is 10°C.
the cube to drop by 5 K is 1000x second, find x in nearest 80. The emissive power of a black body at T = 300 K is 460
integer. W/m2. Consider a body B of area A = 100 cm2 , coefficient
77. Two planets X and Y revolving around the sun is circular of reflectivity r = 0.3 and coefficient of transmission t =
orbits, have temperature of their surfaces as T1 and T2. If 0.5. Its temperature is 300 K. Then the power radiated by
their distance from the sun are in the ratio of 1 : 4, then B is ____ W.
find T1 : T2. Assume the planets to be in the steady state,
and the sun and the planets to be black-bodies. Neglect
the energy exchange between the two planets.
THERMAL PHYSICS 39

EXERCISE - 2 : PREVIOUS YEARS JEE MAIN QUESTIONS


Heat, Temperature and Calorimetry 4. A bimetallic strip consists of metals A and B. It is mounted
1. Consider a spherical shell of radius R at temperature T. rigidly as shown. The metal A has higher coefficient of
The black body radiation inside it can be considered as an expansion compared to that of metal B. When the bimetallic
ideal gas of photons with internal energy per unit volume strip is placed in a cold bath, it will : (2021)

U 1U
u  T 4 and pressure P =   . If the shell now
V 3 V
undergoes an adiabatic expansion the relation between T
and R is : (2015)

1 1 (a) Not bend but shrink


(a) T  (b) T 
R R3 (b) Bend towards the left
(c) Bend towards the right
(c) T  eR (d) T  e3R
(d) Neither bend nor shrink
2. A container with 1 kg of water in it is kept in sunlight, which Heat transfer
causes the water to get warmer than the surroundings. The
5. An experiment take 10 minutes to raise the temperature of
average energy per unit time per unit area received due to
water in a container from 0°C to 100°C and another 55
the sunlight is 700Wm–2 and it is absorbed by the water
minutes to convert it totally into steam by a heater
over an effective area of 0.05m2. Assuming that the heat supplying heat at a uniform rate. Neglecting the specific
loss from the water to the surroundings is governed by heat of the container and taking specific heat of water to
Newton’s law of cooling, the difference (in ºC) in the be 1cal/ g° C, the heat of vapourization according to this
temperature of water and the surroundings after a long time experiment will come out to be : (2015)
will be _____________. (Ignore effect of the container, (a) 560 cal/g (b) 550 cal/g
and take constant for Newton’s law of cooling=0.001s–1,
(c) 540 cal/g (d) 530 cal/g
Heat capacity of water = 4200 J kg –1 K –1
6. A pendulum clock loses 12 s a day if the temperature is
40°C and gains 4 s a day if the temperature is 20°C. The
(2020)
temperature at which the clock will show correct time, and
3. Each slide of a box made of metal sheet in cubic shape is ‘a’ the co-efficient of linear expansion () of the metal of the
at room temperature 'T’, the coefficient of linear expansion pendulum shaft are respectively : (2016)
of the metal sheet is '  ' . The metal sheet is heated uniformly,, (a) 60°C;  = 1.85 × 10–4/°C
by a small temperature T , so that its new temperature is
(b) 30°C;  = 1.85 × 10–3/°C
T  T . Calculate the increase in the volume of the metal
box. (2021) (c) 55°C;  = 1.85 × 10–2/°C
(d) 25°C;  = 1.85 × 10–5/°C
(a) 3a 3 T (b) 4a 3 T
7. 200 g water is heated from 40°C to 60°C. Ignoring the slight
expansion of water, the change in its internal energy is
4 3 close to (Given specific heat of Water = 4184 J/kg/K) :
(c) a T (d) 4a 3T
3
(2016)
(a) 8.4 kJ (b) 4.2 kJ
(c) 16.7 kJ (d) 167.4 kJ
THERMAL PHYSICS 40

8. A simple pendulum made of a bob of mass m and a metallic 12. A body takes 10 minutes to cool from 600 C to 500 C. The
wire of negligible mass has time period 2 s at T = 0°C. If temperature of surroundings is constant at 250 C. Then,
the temperature of the wire is increased and the the temperature of the body after next 10 minutes will be
corresponding change in its time period is plotted against approximately : (2018)
its temperature, the resulting graph is a line of slope S. If
(a) 470 C (b) 410 C
the coefficient of linear expansion of metal is α then the
value of S is (2016) (c) 450 C (d) 430 C
13. When the temperature of a metal wire is increased from 0ºC
 to 10ºC, its length increases by 0.02%. The percentage
(a)  (b)
2 change in its mass density will be closest to :
(2020)
1
(c) 2 (d) (a) 0.06 (b) 0.008

(c) 2.3 (d) 0.8
9. The temperature of an open room of volume 30 m 3
14. The temperature  at the junction of two insulating sheets,
increases from 17°C to 27°C due to the sunshine. The having thermal resistances R1 and R 2 as well as top and
atmospheric pressure in the room remains 1× 10 Pa. If ni
5
bottom temperatures 1 and 2 (as shown in figure) is given
and nf are the number of molecules in the room before and by: (2021)
after heating, then nf – ni will be : (2017)

(a) –2.5 × 1025 (b) – 1.61 × 1023

(c) 1.38 × 1023 (d) 2.5 × 1025


10. A copper ball of mass 100 gm is at a temperature T. It is
dropped in a copper calorimeter of mass 100 gm, filled
with 170 gm of water at room temperature. Subsequently,
the temperature of the system is found to be 75°C. T is
given by : (Given : room temperature = 30°C, specific heat 2 R 2  1 R1 1R 2  2 R1
of copper = 0.1 cal/gm°C) (2017) (a) R 2  R1 (b) R 2  R1
(a) 825°C (b) 800°C
1R1  2 R 2 1R 2   2 R 1
(c) 885°C (d) 1250°C (c) (d)
R1  R 2 R 2  R1
11. In an experiment a sphere of aluminium of mass 0.20 kg is
heated upto 150° C. Immediately, it is put into water of 15. Two identical metal wires of thermal conductivities K1 and
volume 150 cc at 27°C kept in a calorimeter of water
K 2 respectively are connected in series. The effective
equivalent to 0.025 kg. Final temperature of the system is
40°C. The specific heat of aluminium is : thermal conductivity of the combination is: (2021)

(take 4.2 Joule = 1 calorie) (2017) 2K1K 2 K1  K 2


(a) K  K (b) 2K K
(a) 378 J/kg–°C (b) 315 J/kg–°C 1 2 1 2

(c) 476 J/kg–°C (d) 434 J/kg–°C


K1  K 2 K1 K 2
(c) K K (d) K  K
1 2 1 2
THERMAL PHYSICS 41

EXERCISE - 3: ADVANCED OBJECTIVE QUESTIONS


Heat, temperature and calorimetry 5. If two rods of length L and 2L, having coefficient of linear
expansion  and 2 respectively are connected end–on–
Single Choice Questions
end, the average coefficient of linear expansion of the
1. Find the ratio of the lengths of an iron rod and an aluminium composite rod, equals :
rod for which the difference in the lengths is independent
of temperature. Coefficients of linear expansion of iron and 3 5
aluminium are 12 × 10–6/°C and 23 × 10–6/°C respectively. (a)  (b) 
2 2
(a) 23 : 12 (b) 12 : 23
5
(c) data insufficient (d) Not possible (c)  (d) none of these
3
2. Two rods of different materials having coefficients of thermal
expansion 1, 2 and Young’s modulli Y1, Y2 respectively 6. The coefficient of volume expansion of glycerin is
are fixed between two rigid massive walls. The rods are 49 × 10–5 K–1. What is the fractional change in its density
heated such that they undergo the same increase in for a 30°C rise in temperature ?
temperature. There is no bending of the rods. If1 : 2 = 2 (a) increases by 0.015 (b) increases by 0.005
: 3, the thermal stresses developed in the two rods are equal
(c) decreases by 0.015 (d) decreases by 0.005
provided Y1 : Y2 is equal to :
7. A steel rod is clamped at its two ends and rests on a fixed
horizontal base. The rod is unstrained at 20°C. Find the
longitudinal strain developed in the rod if the temperature
rises to 50°C. Coefficient of linear expansion of steel = 1.2
× 10–5/°C.
(a) 6 × 10–4 (b) 3.6 × 10–5
(c) 2.4 × 10–4 (d) 3.6 × 10–4

(a) 2 : 3 (b) 1 : 1 8. A wooden wheel of radius R is made of two semicircular


parts (see figure). The two parts are held together by a ring
(c) 3 : 2 (d) 4 : 9 made of a metal strip of cross-sectional area S and length L.
3. A blacksmith fixes iron ring on the rim of the wooden wheel L is slightly less than 2R. To fit the ring on the wheel, it is
of a bullock cart. The diameter of the rim and the iron ring heated so that its temperature rises by T and it just steps
are 5.243 m and 5.231 m respectively at 27°C. To what over the wheel. As it cools down to surrounding temperature,
temperature should the ring be heated so as to fit the rim of it presses the semicircular parts together. If the coefficient
the wheel ? ( for iron = 1.2 × 10–5/°C) of linear expansion of the metal is  and its Young’s modulus
is Y, the force that one part of the wheel applies on the other
(a) 191 °C (b) 254 °C
part is
(c) 218 °C (d) 164 °C
4. A hole is drilled in a copper sheet. The diameter of the hole
is 4.24 cm at 27.0°C. What is the change in the diameter of
the hole when the sheet is heated to 227°C ? Coefficient of
linear expansion of copper = 1.70 × 10–5 K–1.
(a) Diameter decreases by 7.2 × 10–3 cm
(b) Diameter increases by 7.2 × 10–3 cm
(c) Diameter decreases by 1.44 × 10–2 cm
(a) 2SYT (b) SYT
(d) Diameter increases by 1.44 × 10–2 cm
(c) SYT (d) 2SYT
THERMAL PHYSICS 42

9. A sphere of aluminium of 0.06 kg placed for sufficient time 14. A copper block of mass 2.5 kg is heated in a furnace to a
in a vessel containing boiling water, so that the sphere is at temperature of 500 °C and then placed on a large ice block.
100°C. It is then immediately transfered a vessel containing What is the maximum amount of ice that can melt ? (Specific
0.25 kg of water at 20°C. The temperature of water rises and heat of copper = 0.39 J g–1 K–1; heat of fusion of water = 335 J
attains a steady state at 24°C. Calculate the specific heat g–1).
capacity of aluminium. (neglect heat loss to vessel and (a) 2.6 kg (b) 10 kg
surroundings)
(c) 3.8 kg (d) 1.5 kg
(a) 870 J kg–1 k–1 (b) 1024 J kg–1 k–1
15. A block of ice of mass M = 10 kg is moved back and forth
(c) 921 J kg–1 k–1 (d) 708 J kg–1 k–1 over the flat horizontal surface of a large block of ice. Both
10. A metal block is made from a mixture of 2.4 kg of aluminium blocks are at 0°C and the force that produces the back–and
1.6 kg of brass and 0.8 kg of copper. The amount of heat –forth motion acts only horizontally. The coefficient of
required to raise the temperature of this block from 20°C to friction between the two surfaces is 0.060. If m = 15.2 g of
80°C is (specific heats of aluminium, brass and copper are water is produced, the total distance travelled by the upper
0.216,0.0917 and 0.0931 cal/kg°C respectively): block relative to the lower is : (Lice = 3.34 × 105 J/kg)
(a) 96.2 cal (b) 44.4 cal (a) 432 m (b) 863 m
(c) 86.2 cal (d) 62.8 cal (c) 368 m (d) 216 m
11. Suppose the specific heat capacity of a substance is varying Multiple Choice Questions
with temperature according to, s = A + BT2 where A and B 16. Two identical beakers are filled with water to the same level
are positive constants and T is temperature in °C. If at 4°C. If one say A is heated while the other B is cooled,
temperature of m kg of this substance has to raise from T°C then:
to 2T°C, then the amount of energy supplied to the substance
is (a) water level in A will rise
(b) water level in B will rise
 7BT 3   5BT 3  (c) water level in A will fall
(a) m  AT  J (b) m  AT  J
 3   3 
(d) water level in B will fall
(c) m [AT + 2BT3] J (b) None of these 17. An aluminium sphere of 20 cm diameter is heated from 0°C
12. A mass of a material exists in its solid form at its melting to 100°C. Its volume changes by (given that coefficient of
temperature 10°C. The following processes then occur to linear expansion for aluminium Al = 23 × 10–6/°C)
the material. (a) 28.9 cc (b) 2.89 cc
Process I : An amount of thermal energy Q is added to the (c) 9.28 cc (d) 49.8 cc
material and 3/4 of the material melts.
Assertion & Reason
Process II : An identical additional amount of thermal energy
Q is added to the material and the material is now a liquid at (A) Statement–I is True, Statement–II is True; Statement–II is a
50°C. correct explanation for Statement–I.

What is the ratio of the latent heat of fusion to the specific (B) Statement–I is True, Statement–II is True; Statement–2 is
heat of the liquid for this material ? NOT a correct explanation for Statement–I.

(a) 80°C (b) 60°C (C) Statement–I is True, Statement–II is False.

(c) 40°C (d) None of these (D) Statement–I is False, Statement–II is True.

13. When 0.15 kg of ice of 0°C mixed with 0.30 kg of water at 18. Statement -1 : A drop of cool water accidentally falls on an
50°C in a container, the resulting temperature is 6.7°C. incandescent lamp which is continuously glowing from long
Calculate the heat of fusion of ice. time, then the bulb breaks.

(swater = 4186 J kg–1 K–1) Statement -2 : As temperature of an object decreases then


it tries to contract and thermal contraction occurs.
(a) 3.34 × 105 J kg–1 k–1 (b) 3.63 × 105 J kg–1 k–1
(a) A (b) B
(c) 3.34 × 106 J kg–1 k–1 (d) 4.19 × 105 J kg–1 k–1
(c) C (d) D
THERMAL PHYSICS 43

19. Statement - 1 : The expanded length l of a rod of original 23. A wall has two layers A and B, each made of different
length l0 is not correctly given by (assuming  to be constant material. Both the layers have the same thickness. The
with T) l = l0 (1 + T), thermal conductivity for A is twice that of B and, under steady
if T is large. condition, the temperature difference across the wall is 36°C.
Statement - 2: It is given by l = l0 eT, which cannot be The temperature difference across the layer A is:
treated as being approximately equal to l = l0 (1 + T) for (a) 6°C (b) 12°C
large values of T.
(c) 24°C (d) 18°C
(a) A (b) B
24. Three rods of same dimensions have thermal conductivities
(c) C (d) D
3k, 2k and k. They are arranged as shown, with their ends at
20. Statement - 1 : The coefficient of volume expansion has 100°C, 50°C and 0°C. The temperature of their junction is :
dimension K–1.
Statement - 2 : The coefficient of volume expansion is
defined as the change in volume per unit volume per unit
change in temperature.
(a) A (b) B
(c) C (d) D
Heat transfer
Assertion & Reason
(A) Statement–I is True, Statement–II is True; Statement–II is a 200
(a) 75°C (b) C
correct explanation for Statement–I. 3
(B) Statement–I is True, Statement–II is True; Statement–2 is
NOT a correct explanation for Statement–I. 100
(c) 40°C (d) C
3
(C) Statement–I is True, Statement–II is False.
(D) Statement–I is False, Statement–II is True. 25. Equal temperature differences exist between the ends of two
metallic rods 1 and 2 of equal lengths. Their thermal
21. Statement - 1 : In natural convection, the fluid motion is
conductivities are Kl and K2 and area of cross-section are
caused by density difference produced by temperature
Al and A2, respectively. The condition of equal rates of heat
gradient.
transfer is :
Statement - 2 : In forced convection, the fluid is forced to
flow along the solid surface by means of fans or pumps. (a) K1A2 =K2A1 (b) K1A1 = K2A2
(a) A (b) B (b) K1A12 = K2A22 (d) K12A2 = K22A1
(c) C (d) D 26. Two rods of copper and brass (KC > KB ) of same length
22. One end of a thermally insulated rod is kept at a temperature and area of cross-section are joined as shown. End A is kept
T1 and the other at T2. The rod is composed of two sections at 100°C and end B at 0°C. The temperature at the junction:
of lengths l1 and l2 and thermal conductivities K1 and K2
respectively. The temperature at the interface of the two A Copper Brass B
sections is
(a) will be more than 50°C
l1 l2
T1 T2 (b) will be less than 50°C
(c) will be 50°C
(d) may be more or less than 50°C depending upon the size
of rods
K1 K2
(a) (K2l2T1 + K1l1T2)/(K1l1 + K2l2)
(b) (K2l1T1 + K1l2T2)/(K2l1 + K1l2)
(c) (K1l2T1 + K2l1T2)/(K1l2 + K2l1)
(d) (K1l1T1 + K2l2T2)/(K1l1 + K2l2)
THERMAL PHYSICS 44

27. A long metallic bar is carrying heat from one of its ends to 30. The length of the two rods made up of the same metal and
the other end under steady–state. The variation of having the same area of cross-section are 0.6 m 0.8 m and
temperature  along the length x of the bar from its hot end respectively. The temperature between the ends of first rod
is best described by which of the following figure ? (2009) is 90°C and 60°C and that for the other rod is 150°C and
110°C. For which rod the rate of conduction will be greater
(a) first (b) second
(c) same for both (d) none of these
31. A ring consisting of two parts ADB and ACB of same
conductivity K carries an amount of heat H. The ADB part
is now replaced with another metal keeping the temperatures
T1 and T2 constant. The heat carried increases to 2 H. What
should be the conductivity of the new ADB part ? Given

ACB
 3:
ADB

28. The temperature of the two outer surface of a composite


slab, consisting of two materials having coefficients of
thermal conductivity K and 2K thickness x and 4x,
respectively are T2 and T1 (T2 > T1). The rate of heat transfer

 A  T2  T1  K 
through the slab, in a steady state is  x
 f, with
 
f equals to

x 4x

7
(a) K (b) 2 K
3
T2 K 2K T1

5
(c) K (d) 3 K
2

32. Water is being boiled in a flat bottomed kettle placed on a


(a) 1 (b) 1/2
stove. The area of the bottom is 300 cm2 and the thickness
(c) 2/3 (d) 1/3
is 2 mm. If the amount of steam produced is 1g min–1, then
29. Two identical square rods of metal are welded end to end as the difference of the temperature between the inner and the
shown in figure (i), 20 cal of heat flows through it in 4 min. outer surface of the bottom is : (thermal conductivity of the
If the rods are welded as shown in figure (ii), the same material of the kettle = 0.5 cal–cm–1 °C–1 S–1 and latent
amount of heat will flow through the rods in : heat of the steam is equal to 540 cal g–1)
(a) 12°C (b) 1.2°C
(c) 0.2°C (d) 0.012°C

(a) 1 min (b) 2 min


(c) 4 min (d) 16 min
THERMAL PHYSICS 45

Use the following passage, solve Q. 33 to Q. 36 36. A cylindrical rod of length 50 cm and cross–sectional area 1
PASSAGE –1 cm2 is fitted between a large ice chamber at 0°C and an
evacuated chamber maintained at 27°C as shown in figure.
The rate of flow of heat depends on the nature of material,
Only small portions of the rod are inside the chambers and
cross-sectional area and temperature gradient. If a material
the rest is thermally insulated from the surrounding. The
of conductivity k has set up a temperature gradient in x
cross–section going into the evacuated chamber is blackened
direction, then rate of heat flow at cross-section having area
so that it completely absorbs any radiations falling on it.
dQ dT The temperature of the blackened end is 17°C when steady
A is kA
dt dx state is reached. Stefan constant  = 6 × 10–8 W/m2–K4. Find
the thermal conductivity of the material of the rod.
Between any two points of conductor, if temperature difference
is T and rate of heat flow is H, then the resistance or
17°C
T
opposition offered by the material to flow is defined as = .
H 0°C 27°C
Now consider two very thin concentric metallic shells A and 37. A hot body, obeying Newton’s law of cooling is cooling
B of radii R1 and R2 (R2 > R1) and temperature T1 and T2 down from its peak value 800C to an ambient temperature
(T1 > T2) respectively. The hollow space between them is of 300C. It takes 5 minutes in cooling down from 800C to
filled with sand of thermal conductivity k. 400C. How much time will it take to cool down from 620C
33. Thermal resistance offered by the sand is to 320C? (Given ln 2 = 0.693, ln 5 = 1.609)
(a) 3.75 minutes (b) 8.6 minutes
1  1 1  1  1 1 
(a)    (b) 4k  R  R  (c) 6.5 minutes (d) 9.6 minutes
2k  R1 R 2   1 2 
38. If wavelength of maximum intensity of radiation emitted
by sun and moon are 0.5 × 10–6 m and 10–4 m respectively
 1 1   1 1  the ratio of their temperature is :
(c) 4k    (d) 2k   
 R1 R 2   R1 R 2  (a) 2000 (b) 1000
34. The rate of heat flow through the sand is (c) 100 (d) 200
39. In the figure, the distribution of energy density of the
R1R 2 R1R 2
(a) 4k  T1  T2  (b) 2k  T1  T2  radiation emitted by a black body at a given temperature is
R1  R 2 R1  R 2 shown. The possible temperature of the black body is :

 T1  T2   1 1   T1  T2   1 1 
(c)    (d)   
4k  R1 R2  2k R
 1 R 2 

35. The temperature T of a point at distance r from the centre in


steady state condition where R1 < r < R2 is

 R 2  r  R1  
(a)  T1  T2   r R  R 
  2 1 

 R  r  R1  
(b) T1   T1  T2   2
 r  R  R  
 2 1  (a) 1500 K (b) 2000 K
(c) 2500 K (d) 3000 K
 R 1  r  R1   40. If a black body radiates 10 cal/s at 227°C, it will radiate at
(c)  T1  T2   r R  R 
  2 1 
727°C :
(a) 10 cal/s (b) 80 cal/s
 R r  R2   (c) 160 cal/s (d) none of these
(d) T1   T1  T2   1
 r  R  R  
 2 1 
THERMAL PHYSICS 46

41. A body cools from 50°C to 40°C in 5 min. The surrounding


d
temperature is 20°C. In what further time (in min) will it 45. In Newton’s law of cooling,  k    0  , the constant
dt
cool to 30°C?
‘k’ is proportional to :
15 (a) A, surface area of the body
(a) 5 (b)
2 (b) S, specific heat of the body

25 (c) 1/m, m being mass of the body


(c) (d) 10
3 (d) e, emmisivity of the body

42. If a body cools down from 80°C to 60°C in 10 min when the
temperature of the surrounding is 30°C. Then, the Assertion & Reason
temperature of the body after next 10 min will be : (A) Statement–I is True, Statement–II is True; Statement–II is a
(a) 50°C (b) 48°C correct explanation for Statement–I.
(c) 30°C (d) none of these (B) Statement–I is True, Statement–II is True; Statement–2 is
NOT a correct explanation for Statement–I.
43. A liquid cools from 50°C to 45°C in 5 min and from 45°Cto
41.5°C in the next 5 min. The temperature of the surrounding (C) Statement–I is True, Statement–II is False.
is : (D) Statement–I is False, Statement–II is True.
(a) 27°C (b) 40.3°C 46. Statement - 1 : Two stars S1 and S2 radiate maximum energy
(c) 23.3°C (d) 33.3°C at 360 nm and 480 nm, respectively. Ratio of their absolute
temperature is 4 : 3.
Statement - 2 : According to Wien’s law T = b (constant).
Multiple Choice Questions
(a) A (b) B
44. Two identical objects A and B are at temperatures TA and
TB respectively. Both objects are placed in a room with (c) C (d) D
perfectly absorbing walls maintained at a temperature 47. A black body radiates power P and maximum energy is
T (TA > T > TB). The objects A and B attain the temperature radiated by it around a wavelength 0 . The temperature of
T eventually. Select the correct statements from the following the black body is now changed such that it radiates maximum
(a) A only emits radiations, while B only absorbs it until 3 0
energy around the wavelength . The power radiated by
both attain the temperature T. 4
(b) A loses more heat by radiation than it absorbs, while B it now is :
absorbs more radiation than it emits, until they attain the
256 27
temperature T (a) P (b) P
81 64
(c) Both A and B only absorb radiation, but do not emit it, until
they attain the temperature T. 64 81
(c) P (d) P
(d) Each object continues to emit and absorb radiation even 27 256
after attaining the temperature T. 48. The maximum energy in thermal radiations from a blackbody
occurs at wavelength 4000 Å. The effective temperature of
the source is :
(a) 7000 K (b) 80000 K
(c) 104 K (d) 106 K
THERMAL PHYSICS 47

Match the Column 50. A black coloured solid sphere of radius R and mass M is
49. A ball has surface temperature T initially at time t = 0, that inside a cavity with vacuum inside. The walls of the cavity
is less than surrounding constant temperature T0. On the are maintained at temperature T0. The initial temperature
vertical axis of the graph shown has either thermal energy of the sphere is 3T0. If the specific heat of the material of
radiated/absorbed per unit time or total energy radiated/ the sphere varies as T3 per unit mass with the temperature
absorbed till time t by the ball. Correctly match the curves T of the sphere, where  is a constant, then the time taken
marked in the graph : for the sphere to cool down to temperature 2T0 will be ( is
Stefan Boltzmann constant)

M 3 M  16 
(a) 2
n   (b) n  
2
4R   2  16R   3 

M 3 M  16 
(c) n  
2 (d) 2
n  
16R   2  4R   3 
Column I Column II

(A) Thermal energy emitted (p) 1


per unit time
(B) Thermal energy absorbed (q) 2
per unit time
(C) Total energy emitted till time t (r) 3
(D) Total energy absorbed till time t (s) 4
THERMAL PHYSICS 48

EXERCISE - 4 : PREVIOUS YEARS JEE ADVANCED QUESTIONS


Heat, Temperature and Calorimetry 5. A current carrying wire heats a metal rod. The wire provides
1. A cubical block of co-efficient of linear expansion s , is a constant power (P) to the rod. The metal rod is enclosed
in an insulated container. It is observed that the temperature
submerged partially inside a liquid of co-efficient of volume
(T) in the metal rod changes with time (t)
expansion   . On increasing the temperature of the system
as T(t)  T0 (1   t1/4 ) where  is a constant with
T , the height of the cube inside the liquid remains
appropriate dimension while T0 is a constant with dimension
unchanged. Find the relation between s , and l . of temperature.The heat capacity of metal is :
(2004) (2019)
2. A piece of ice (heat capacity = 2100 J kg–1 °C–1 and latent
heat = 3.36 × 105 J kg–1) of mass m gram is at –5°C at 4P(T(t) - T0 )4 4P(T(t) - T0 )3
atmospheric pressure. It is given 420 J of heat so that the ice (a) (b)
β 4 T05 β 4 T04
starts melting. Finally when the ice–water mixture is in
equilibrium, it is found that 1 g of ice has melted. Assuming
there is no other heat exchange in the process, the value of 4P(T(t) - T0 ) 4P(T(t) - T0 )2
m is (2010) (c) 4 (d)
β T02 β 4 T03
3. Steel wire of length L at 40°C is suspended from the ceiling
and then a mass m is hung from its free end. The wire is 6. Two identical beakers A and B contain equal volumes of
cooled down from 40°C to 30°C to regain its original length two different liquids at 60°C each and left to cool down.
L. The coefficient of linear thermal expansion of the steel Liquid in A has density of 8 × 102 kg/m3 and specific heat of
is 10–5/ºC, Young’s modulus of steel is 1011N/m2 and radius 2000 Jkg–1K–1 while liquid in B has density of 103 kg m–3 and
of the wire is 1 mm. Assume that L>> diameter of the wire. specific heat of 4000 Jkg–1K–1. Which of the following best
Then the value of m in kg is nearly. (2011) describes their temperature versus time graph
4. The figure below shows the variation of specific heat schematically? (assume the emissivity of both the beakers
capacity(C) of a solid as a function of temperature (T). The to be the same) (2019)
temperature is increased continuously from 0 to 500 K at a
constant rate. Ignoring any volume change, the following
statement(s) is (are) correct to reasonable approximation.
(2013)

100 200 300 400 500


T(K)
(a) the rate at which heat is absorbed in the range 0–100
K varies linearly with temperature T
(b) heat absorbed in increasing the temperature from
0–100 K is less than the heat required for increasing
the temperature from 400–500 K
(c) there is no change in the rate of heat absorbtion in the
range 400–500 K
(d) the rate of heat absorption increases in the range 200–
300 K
THERMAL PHYSICS 49

7. A liquid at 30° C is poured very slowly into a open 11. An ideal black-body at room temperature is thrown into a
Calorimeter that is at temperature of 110°C. The boiling furnace. It is observed that (2002)
temperature of the liquid is 80°C. It is found that the first 5
(a) initially it is the darkest body and at later times the
gm of the liquid completely evaporates. After pouring brightest
another 80 gm of the liquid the equilibrium temperature is
found to be 50°C. The ratio of the Latent heat of the liquid (b) it is the darkest body at all times
to its specific heat will be _____ ºC. (Neglect the heat (c) it cannot be distinguished at all times
exchange with surrounding]
(d) initially it is the darkest body and at later times it cannot
(2019) be distinguished.
8. When the temperature of a metal wire is increased from 0ºC
12. The graph, shown in the adjacent diagram, represents the
to 10ºC, its length increases by 0.02%. The percentage
variation of temperature (T) of two bodies, x and y having
change in its mass density will be closest to :
same surface area, with time (t) due to the emission of
(2020) radiation. Find the correct relation between the emissivity
(a) 0.06 (b) 0.008 and absorptivity powers of the two bodies (2003)
(c) 2.3 (d) 0.8

Heat Transfer
9. The plots of intensity versus wavelength for three black
bodies at temperatures T1, T2 and T3 respectively are as
shown. Their temperature are such that (2000)

(a) Ex > Ey and ax < ay (b) Ex < Ey and ax > ay


(c) Ex > Ey and ax > ay (d) Ex < Ey and ax < ay
13. Two identical rods are connected between two conatiners.
One of them is at 100°C containing water and another is at
0°C containing ice. If rods are connected in parallel then the
rate of melting of ice is ql g/s. If they are connected in series
q2
then the rate is q2. The ratio q is (2004)
1

(a) 2 (b) 4
(a) T1 > T2 > T3 (b) T1> T3 > T2
(C) T2 > T3 > T1 (d) T3 > T2 > T1 1 1
(c) (d)
10. When a block of iron floats in mercury at 0ºC, fraction k1 of 2 4
its volume is submerged, while at the temperature 60ºC, a 14. Three discs, A, B and C having radii 2 m, 4 m and 6 m
fraction k2 is seen to be submerged. If the coefficient of respectively are coated with carbon black on their outer
volume expansion of iron is Fe and that of mercury is Hg, surfaces. The wavelengths corresponding to maximum
then the ratio k1/k2 can be expressed as (2001) intensity are 300 nm, 400 nm and 500 nm, respectively. The
power radiated by them are QA, QB and QC respectively
1  60  Fe 1  60  Fe (2004)
(a) (b)
1  60  Hg 1  60  Hg
(a) QA is maximum (b) QB is maximum

1  60  Fe 1  60  Hg (c) QC is maximum (d) QA = QB = QC


(c) (d)
1  60  Hg 1  60  Fe
THERMAL PHYSICS 50

15. Variation of radiant energy emitted by sun, filament of 20. A metal rod AB of length 10x has its one end A in ice at 0°C
tungsten lamp and welding arc as a function of its and the other end B in water at 100°C. If a point P on the rod
wavelength is shown in figure. (2005) is maintained at 400°C, then it is found that equal amounts
of water and ice evaporate and melt per unit time. The latent
heat of evaporation of water is 540 calg–1 and latent heat of
melting of ice is 80 calg–1. If the point P is at a diatance of x
from the ice end A, find the value of . (Neglect any heat
loss to the surrounding). (2009)
21. Two spherical bodies A (radius 6 cm) and B (radius 18 cm)
are at temperatures T1 and T2, respectively. The maximum
intensity in the emission spectrum of A is at 500 nm and in
that of B is at 1500 nm. Considering them to be black bodies,
what will be the ratio of the rate of total energy radiated by
Which of the following option is the correct match?
A to that of B ? (2010)
(a) Sun-T3, tungsten filament-T1, welding arc-T2
22. A composite block is made of slabs A, B, C, D and E of
(b) Sun-T2, tungsten filament-T2, welding arc-T3 different thermal conductivities (given in terms of a
(c) Sun-T3, tungsten filament-T2, welding arc-T1 constant K) and sizes (given in terms of length L) as shown
(d) Sun-T1, tungsten filament-T2, welding arc-T3 in the figure. All slabs are of same width. Heat Q flows
16. In which of the following processes, convection does not only from left to right through the blocks. Then in steady
take place primarily? (2005) state (2011)
(a) sea and land breeze
(b) boiling of water
(c) heating air around a furnace
(d) warming of glass of bulb due to filament.
17. A spherical body of area A and emissivity e = 0.6 is kept
inside a perfectly black body. Energy radiated per second
by the body at temperature T is (2005)
(a) 0.4  AT T 4 (b) 0.8  ATT 4

(c) 0.6  AT T4 T4
(d) l.0  AT
18. Water of volume 2 litre in a container is heated with a coil of
1 kW at 27°C. The lid of the container is open and energy (a) heat flown through A and E slabs are same
dissipates at rate of 160 J/s. In how much time temperature
(b) heat flown through slab E is maximum
wil rise from 27°C to 77°C.
[Given specific heat of water is 4.2 kJ/kg] (2005) (c) temperature difference across slab E is smallest
(a) 7 min (b) 6 min 2 s (d) heat flown through C = heat flow through B + heat
(c) 8 min 20s (d) 14 min flown through D.
19. Initially a black body at absolute temperature T is kept inside 23. Three very large plates of same area are kept parallel and
a closed chamber at absolute temperature T0. Now the close to each other. They are considered as ideal black
chamber is slightly opened to allow sun rays to enter. It is surfaces and have very high thermal conductivity. The first
observed that temperatures T and T0 remains constant. and third plates are maintained at temperatures 2T and 3T
Which of the following statements is/are true? (2006) respectively. The temperature of the middle (i.e., second)
(a) The rate of emission of energy from the black body plate under steady state condition is : (2012)
remains the same
1 1
(b) The rate of emission of energy from the black body  65  4  97  4
increases (a)   T (b)   T
 2   4 
(c) The rate of absorption of energy by the black body
increases. 1
(d) The energy radiated by the black body equals the energy  97  4 1
(c)   T (d)  97  4 T
absorbed by it.  2 
THERMAL PHYSICS 51

24. Two rectangular blocks, having indentical dimensions, can 28. A water cooler of storage capacity 120 litres can cool water
be arranged either in configuration I or in configuration II at a constant rate of P watts. In a closed circulation system
as shown in the figure. One of the blocks has thermal (as shown schematically in the figure), the water from the
conductivity K and the other 2K. The temperature difference cooler is used to cool an external device that generates
between the ends along the x-axis is the same in both the constantly 3 kW of heat (thermal load). The temperature of
configurations. It takes 9s to transport a certain amount of water fed into the device cannot exceed 30°C and the entire
heat from the hot end to the cold end in the configuration I. stored 120 litres of water is initially cooled to 10°C. The
The time to transport the same amount of heat in the entire system is thermally insulated. The minimum value of
configuration II is : (2013) P (in watts) for which the device can be operated for 3
Configuration II hours is (2016)

Configuration I

2K

K 2K K
X
(a) 2.0 s (b) 3.0 s
(c) 4.5 s (d) 6.0 s
25. Heater of an electric kettle is made of a wire of length L and (Specific heat of water is 4.2 kJ kg–1 K–1 and the density of
diameter d. It takes 4 minutes to raise the temperature of 0.5 water is 1000 kg m–3)
kg water by 40 K. This heater is replaced by a new heater
(a) 1600 (b) 2067
having two wires of the same material, each of length L and
diameter 2d. The way these wires are connected is given in (c) 2533 (d) 3933
the options. How much time in minutes will it take to raise 29. A metal is heated in a furnace where a sensor is kept above
the temperature of the same amount of water by 40 K? the metal surface to read the power radiated (P) by the
(2014) metal. The sensor has a scale that displays log2 (P/P0), where
P0 is a constant. When the metal surface is at a temperature
(a) 4 if wires are in parallel (b) 2 if wires are in series
of 487°C, the sensor shows a value 1. Assume that the
(c) 1 if wires are in series (d) 0.5 if wires are in parallel emissivity of the metallic surface remains constant. What
26. Two spherical stars A and B emit black body radiation. The is the value displayed by the sensor when the temperature
radius of A is 400 times that of B and A emits 104 times the of the metal surface is raised to 2767°C?
(2016)
 A 
30. Two conducting cylinders of equal length but different radii
power emitted from B. The ratio   of their
 B  are connected in series between two heat baths kept at
temperatures T1 = 300 K and T2 = 100 K, as shown in the
wavelengths A and B at which the peaks occur in their figure. The radius of the bigger cylinder is twice that of the
respective radiation curves is (2015) smaller one and the thermal conductivities of the materials
27. The ends Q and R of two thin wires, PQ and RS, are soldered of the smaller and the larger cylinders are K1 and K2
(joined) together. Initially each of the wires has a length of respectively. If the temperature at the junction of the two
1 m at 10°C. Now the end P is maintained at 10°C, while the cylinders is the steady state is 200 K, then K 1 /K 2
end S is heated and maintained at 400°C. The system is = _________.
thermally insulated form its surroundings. If the thermal (2018)
conductivity of wire PQ is twice that of the wire RS and the
coefficient of linear thermal expansion of PQ is 1.2 × 10–5 K–
1
, the change in length of the wire PQ is
(2016)
(a) 0.70 mm (b) 0.90 mm
(c) 1.56 mm (d) 2.34 mm
THERMAL PHYSICS 52

Find Answer Key and Detailed Solutions at the end of this book

THERMAL PHYSICS

Please share your valuable feedback by


scanning the QR code.
KINETIC THEORY OF GASES AND THERMODYNAMICS 53

15
KINETIC THEORY OF GASES &
THERMODYNAMICS

SCAN CODE
KINETIC THEORY OF GASES & THERMODYNAMICS
Chapter
KINETIC THEORY15
OF GASES & THERMODYNAMICS 54

KINETIC THEORY OF GASES & THERMODYNAMICS

1. INTRODUCTION TO KINETIC THEORY OF GASES 1.2 Pressure Calculation


Pressure of an Ideal Gas and Its Expression:
In this topic, we discuss the behaviour of gases and how Pressure exerted by the gas is due to continuous
are the various state variable like P, V, T, moles, U etc bombardment of gas molecules against the walls of the
are interrelated with each other? container.
Expression:
1.1 Postulates of Kinetic Theory of Gases
Consider a gas enclosed in a cube of side l. Take the axes
• A gas consists of a very large number of molecules
to be parallel to the sides of the cube, as shown in figure.
(of the order of Avogadro’s number, 1023), which are
perfect elastic spheres. For a given gas they are (
A molecule with velocity v x , v y , v z ) hits the planar
identical in all respects, but for different gases, they wall parallel to yz-plane of area A (= l2). Since the
are different. collision is elastic, the molecule rebounds with the same
• The molecules of a gas are in a state of incessant velocity; its y and z components of velocity do not
random motion. They move in all directions with change in the collision but the x-component reverses
different speeds., (of the order of 500 m/s) and obey sign. That is, the velocity after collision is
Newton’s laws of motion.
( −v x , v y , v z ) . The change in momentum of the
• The size of the gas molecules is very small as
compared to the distance between them. If typical molecule is: −mv x − ( mv x ) =
−2mv x . By the principle
size of molecule is 2Å, average distance between the of conservation of momentum, the magnitude of
molecules is 320Å. Hence volume occupied by the momentum imparted to the wall in the collision = 2mvx.
molecules is negligible in comparison to the volume
of the gas.
• The molecules do not exert any force of attraction or
repulsion on each other, except during collision.
• The collisions of the molecules with themselves and
with the walls of the vessel are perfectly elastic. As
such, that momentum and the kinetic energy of the
molecules are conserved during collisions, though
their individual velocities change.
• There is no concentration of the molecules at any Fig 15.1
point inside the container i.e. molecular density is
uniform throughout the gas. To calculate the force (and pressure) on the wall, we need
• A molecule moves along a straight line between two to calculate momentum imparted to the wall per unit
successive collisions and the average straight time, if it is within the distance v x ∆t from the wall, that
distance covered between two successive collisions is, all molecules within the volume. Av x ∆t only can hit
is called the mean free path of the molecules.
1
• The collisions are almost instantaneous, i.e., the time the wall in time ∆T is A v x ∆t n, where n is the
2
of collision of two molecules is negligible as
number of molecules per unit volume. The total
compared to time interval between two successive
momentum transferred to the wall by these molecules in
collisions.
time ∆t is: Q ( 2mv x )  nAv x ∆t  . The force on the
1
=
2 
wall is the rate of momentum transfer Q/∆t and pressure
is force per unit area:

SCAN CODE
KINETIC THEORY OF GASES & THERMODYNAMICS
KINETIC THEORY OF GASES AND THERMODYNAMICS 55

Q 1.3 Ideal Gas Laws


=P = nmv 2x
( A ∆t ) • Ideal Gas
That gas which strictly obeys the gas laws, (such as
Actually, all molecules in a gas do not have the same
Boyle’s Law, Charle's law, Gay Lussac’s Law etc.)
velocity; there is a distribution in velocities. The above
equation therefore, stands for pressure due to the group Characteristics
of molecules with speed vx in the x-direction and n stands 1. The size of the molecule of an ideal gas is zero.
for the number density of that group of molecules. The 2. There is no force of attraction or repulsion amongst
total pressure is obtained by summing over the
the molecules of an ideal gas.
contribution due to all groups:
2 • Real Gas
P = nmv x All gases are referred to as real Gases. All real gases
2 show nearly the ideal gas behavior at low pressures
where v x is the average of v 2x . Now the gas is isotropic, and temperatures high enough, where they cannot be
i.e. there is no preferred direction of velocity of the liquified.
molecules in the vessel. Therefore by symmetry, Gay Lussac’s Law:
2 2 2 2 
v=
x v=
y vz We know that PV = NK
3

( )
2  1  2 2 2  1  2 
=
vx   vx + vy + =
vz  v where K is the average kinetic energy of translation per
3 3 
gas molecule. At constant temperature. K is constant
2 and for a given mass of the gas. N is constant.
where v is the speed and v denotes the mean of the
squared speed. Thus Thus, PV = constant for given mass of gas at constant
temperature, which is also called Boyle’s Law.
1 2
P =   nmv Charle’s Law:
3
2 
2 1 M 1 2 We know that PV = NK
1 3
P= mnv = = ρv
3 3V 3
For a given mass of gas, N is constant.
M = Total mass of gas molecules  3 
=
Since K k B T, K ∝ T and as such PV ∝ T.
V = Total volume of gas molecules 2

Relation Between Pressure and KE of Gas Molecules If P is constant, V ∝ T, which is the Charles’ Law.
Constant Volume Law:
1 2
From equation P= ρv
3 2 
We know that PV = NK
3
2 1 
⇒ P =  Mv −2  For a given mass of gas, N is constant. Since
3V  2 
 3 
=K k B T, K ∝ T
2 K.E 2
⇒P=
3 V Thus, PV ∝ T
2 If V is constant, P ∝ T, which the constant volume law.
⇒ P =E
3
Avogadro’s Law:
Pressure exerted by an ideal gas is numerically equal to Consider two gases 1 and 2. We can write
two third of mean kinetic energy
2  2 
=P1V1 = N1 K1 , P2 V2 N2 K 2
3 3

SCAN CODE
KINETIC THEORY OF GASES & THERMODYNAMICS
KINETIC THEORY OF GASES & THERMODYNAMICS 56

If their pressures, volumes and temperatures are the Real Gas Law
same, then By explicitly including the effects of molecular size and
 
=P1 P=2 , V1 V=
2 , K1 K2. intermolecular forces, the Dutch physicist Johannes van
der Waals modified the ideal gas law to explain the
Clearly, N1 = N2 Thus: behavior of real gases. The Vander Waal real gas
Equal volumes of all ideal gases existing under the same equation is given below.
conditions of temperature and pressure contain equal Real gas law equation,
number of molecules which is Avogadro’s Law or
 n 2a 
 ( V − nb ) =
hypothesis. +
 P nRT
This law is named after the Italian physicist and chemist,  V2 
Amedeo Avogadro (1776 – 1856). Where a and b represent the empirical constant which is
PV unique for each gas.
Aliter:= =
As PV Nk B T, N
k BT n2
represents the concentration of gas.
If P, V and T are constants, N is also constant. V2
1.4 Ideal Gas Equation P represents pressure

 3 R represents a universal gas constant and T is the


2 
As PV = NK and K = k B T temperature.
3 2
Difference Between Ideal and Real Gases
2 3 
PV = N  k B T  or PV = Nk B T The table below shows the properties and the behaviour
3 2 
of ideal and real gases.
which is the ideal gas equation
Ideal Gas Real Gas
1.5 Real Gas Equation and Related Concepts No definite volume Definite volume
Real Gas Definition: A real gas is defined as a gas that Elastic collision of Non-elastic collisions
at all standard pressure and temperature conditions does particles between particles
not obey gas laws. It deviates from its ideal behavior as
No intermolecular Intermolecular attraction
the gas becomes huge and voluminous. True gases have
attraction force force
velocity, mass, and volume. They liquefy when cooled to
their boiling point. The space filled by gas is not small Does not really exist in the It really exists in the
when compared to the total volume of gas. environment and is a environment
hypothetical gas
Ideal and Real Gas Equation An ideal gas is defined as a
gas that obeys gas laws at all pressure and temperature High pressure The pressure is less when
conditions. Ideal gases have velocity as well as mass. compared to ideal gas
They have no volume. The volume taken up by the gas is Independent Interacts with others
small as compared to the overall volume of the gas. It
Obeys PV = NRT Obeys
does not condense, and triple-point does not exist.
 n 2a 
The ideal gas law is the equation of the state of a  P + 2  ( V − nb ) =
nRT
hypothetical ideal gas, also called the general gas  V 
equation. Under many conditions, it is a reasonable
approximation of the behavior of several gases, but it has
many limitations. In 1834, Benoit Paul Emile Clapeyron
first described it as a variation of the empirical law of
Boyle, the law of Charles, the law of Avogadro, and the
law of Gay-Lussac. In an empirical form, the ideal gas
law is also written:
PV = nRT

SCAN CODE
KINETIC THEORY OF GASES & THERMODYNAMICS
KINETIC THEORY OF GASES AND THERMODYNAMICS 57

2. SPEEDS OF GAS MOLECULES Vrms =


3RT
M
Maxwell’s speed Distribution Law, average, RMS and Where M = Molecular Mass of Gas
most Probable Speeds.  1
N∫
Similarly, Vav= V= VdN v
Molecule Nature of Matter: Same as Atomic Theory
given by Dalton, according to him, atoms are the smallest 8RT
=
constituents of elements. All atoms of one element are πM
identical, but atoms of different element are different. dN v
But VMP is velocity at which =0
In solids: Atoms are tightly packed, interatomic spacing dv
about 1A°. Interatomic force of attraction are strong.
3RT
In liquids: Atoms are not as rigidly fixed as in solids. ⇒ VMP =
M
Interatomic spacing is about the same 2Å. Interatomic
Physically VMP is velocity possessed by Maximum
force a attraction are relative weaker.
number of molecules.
In Gases: Atoms are very free. Inter atomic spacing is
NOTE:
about tens of Angstroms. Interatomic forces are much
weaker in gases than both in solids and liquids. Vrms > Vav > VMP

In this chapter, we mainly focus on gases


2.1 Maxwell’s Law of Distribution of Molecular 3. ENERGY OF GAS MOLECULES
Velocities
Assumptions of Maxwell Distribution 3.1 Degrees of Freedom
• Molecules of all velocities between 0 to ∞ are The number of degrees of freedom of a dynamical
present. system is defined as the total number of co-ordinates or
• Velocity of one molecule, continuously changes, independent quantities required to describe completely
though fraction of molecules in one range of the position and configuration of the system.
velocities is constant. Example:
Result • A particle moving in straight line, say along X-axis
3/ 2 mv 2
 M  − need only x coordinate to define itself. It has only
N v = 4πN  
2
Ve 2k B T
one degree of freedom.
 2πk B T 
• A particle in a plane, needs 2 co-ordinates, hence
dN v
where N v = has 2 degrees of freedom.
dV In general if
where dNv = Total number of molecules with speeds A = number of particles in the system
between V and V + dV R = number of independent relations among the
N = Total number of molecules. particles
N = Number of degrees of freedom of the system
N = 3A – R
Monoatomic Gases
The molecules of a monoatomic gas (like neon, argon,
helium etc) consists only of one atom.
∴A=1
R=0
Fig 15.2 ∴N=3
Based on this we define three types of speed for Here 3 degrees of freedom are for translational motion
molecules of gas

( )
 2 1/ 2  1 
1/ 2

=
Vrms =  N ∫ V dNv 
2
V
 

SCAN CODE
KINETIC THEORY OF GASES & THERMODYNAMICS
KINETIC THEORY OF GASES & THERMODYNAMICS 58

Diatomic Gases Polyatomic Gas


A=2 A polyatomic gas has 3 translational, 3 rotational degrees
Assuming the distance between the two molecules is of freedom. Apart from them if there V vibrational
fixed then R = 1 modes then there will be additional 2V vibrational
⇒ N = 3× 2 −1 = 5 degrees of freedom.
Here 5 degrees of freedom implies combination of ∴ Total degree of freedom
3 translational energies and 2 rotational energies. n = 3 + 3 + 2V = 6 + 2V
3.2 Internal Energy and Kinetic Energy
Internal Energy: As studied in thermodynamics, Internal
Energy of any substance is the combination of Potential
Energies and Kinetic Energies of all molecules inside a
given gas.
• In real gas
Internal Energy = P.E of molecules + K.E of
Molecules
• In real gas: Internal Energy = K.E of Molecules
Here PE of molecules is zero as assumed in Kinetic
theory postulates; There is no interaction between
the molecules hence its interactional energy is zero.
Average KE per Molecule of the Gas:
1 M 2
We know, P = v
3V
1 2
⇒ PV = Mv
3
1 2
Fig 15.3
If vibrational motion is also considered then [only at very Hence, nRT = Mv
3
high temperatures]
1 2
N=7 ⇒ nRT = Nmv
where 3 for translational 3
2 for rotational n 3RT 1  2
⇒ = mv
2 for vibrational N 2 2
Triatomic Gas Also N = nN A
3 R 1 2 3
Fig 15.4 ⇒ T= ( K.E )avg
mv ⇒ K B T =
2 NA 2 2
Linear
Average KE of translation per molecule of the gas
A=3
R=2 3
K BT
⇒N=3×3–2=7 2
Non-Linear Kinetic Interpretation of Temperature
From above equations, we can easily see that KE of one
molecule is only dependent upon its temperature.
⇒ KE of molecule will cease if, the temperature of the
gas molecules become absolute zero.
∴ Absolute zero of a temperature may be defined as that
temperature at which the root mean square velocity of the
Fig 15.5
gas molecule reduces to zero.
A=3
All the Ideal gas laws can be derived from Kinetic
R=3⇒N=3×3–3=6 Theory of gases.
• Here again vibrational energy is ignored.

SCAN CODE
KINETIC THEORY OF GASES & THERMODYNAMICS
KINETIC THEORY OF GASES AND THERMODYNAMICS 59

3.3 Law of Equipartition of Energy ∆Q  ∆U


=
CV =  [⸪ W = 0 for constant V]
Statement: According to this law, for any dynamical ∆T  V ∆T
system in thermal equilibrium, the total energy is 3R
⇒ Cv =
distributed equally amongst all the degrees of freedom, 2
and the energy associated with each molecule per degree Cp 5
5R
1 ∴ Cp = and =
γ = .
of freedom is k B T, where kB is Boltzman constant and 2 Cv 3
2
Diatomic Gases
T is temperature of the system.
When no vibration
k T
Application: U = f B where f = Total degrees of Degree of freedom = 5
2
5
freedom. This law is very helpful in determining the total Average energy for one mole = RT
2
internal energy of any system be it monatomic, diatomic
∆U 5
or any polyatomic. Once the internal energy is know we ∴ Cv = = R
can very easily predict Cv and Cp for such systems. ∆T 2
Remark: In case vibrational motion is also there in any 7R
Cp =
system, say for diatomic molecule, then there should be 2
energy due to vibrational as well given by Cp 7
2
=
γ =
1  dy  1 2 Cv 5
=Ev m   + ky
2  dt  2 When vibration is present.
dy ky 2 There is only one mode of vibration between 2
where = vibrational velocity and = Energy molecules.
dt 2
∴ Degree of freedom = 7
due to configuration.
7
According to Law of Equipartition ∴ U =RT
1 2
Energy per degree of freedom = k B T 7 9
2 ⇒ Cv = R and Cp = R
1 1 2 2
⇒ Total energy = k B T + k B T = k B T 9
2 2 and γ =
is energy for complete one vibrational mode 7
Polyatomic Gases
3.4 Specific Heat of Gases
Degree of freedom
Specific Heat Capacity: = 3 for translational
As we know the law of equipartition, we can predict the + 3 for rotational
heat capacity of various gases. + 2V for vibrational
= 6 + 2V
Monoatomic Gas
If v = Number of vibrational modes
Degree of freedom = 3. RT
∴ U = ( 6 + 2V ) K
∴ Average Energy of a molecule at temperature T 2
1  ⇒ C v =( 3 + V ) R
⇒E=
3  k BT 
2  C=
p (4 + V) R
Energy for one mole ⇒ E × N A 4+V
and γ =
3+ V
3
⇒ U =( k B N A ) T
2 Specific Heat Capacity of Water

3 Water is treated like solid.


⇒ U =RT
2 Water has three atoms, 2 of hydrogen and one of oxygen
In thermodynamics, we studied ∴ Total degree of freedom for every atom

SCAN CODE
KINETIC THEORY OF GASES & THERMODYNAMICS
KINETIC THEORY OF GASES & THERMODYNAMICS 60

=3×2=6 3.5 Mean Free Path


∴ Total degree of freedom for every molecule of water The path traversed by a molecule between two
= 3 × 6 = 18 successive collisions with other molecule is called the
mean free path
 1  
18 × R∆T  Total distance travelled by a molecule
∆Q ∆U  2  l=
∴ C= = = No. of collisions it makes with other molecules
∆T ∆T ∆T
Expression:
C = 9R
Mean Free Path
Specific Heat Capacity of Solids
• In solids, there is very less difference between heat
capacity at constant pressure and at that constant
volume. Therefore we do not differentiate between
Cp and Cv for solids.
∆Q ∆U
∴ C= =
∆T ∆T
{As solids hardly expand or expansion is negligible}
Now in solid the atoms are arranged in an array Fig 15.7
structure and they are not free to move
Suppose the molecules of a gas are spheres of diameter
independently like in gases. 
Therefore the atoms do not possess any translational d. Focus on a single molecule with the average speed V.
or rotational degree of freedom. It will suffer collision with any molecule that comes
On the other hand, the molecules do possess within a distance d between the centres. In time ∆t, it

vibrational motion along 3 mutually perpendicular sweeps a volume πd 2 V ∆t wherein any other molecule
directions.
will collide with it (as shown in figure). If n is the number
Hence for 1 mole of a solid, there are NA number of
of molecules per unit volume, the molecule suffers
atoms. The energy associated with every molecule 
nπd 2 V ∆t collisions in time ∆t. thus the rate of
 1  
= 3 2 × k BT  =3K B T
 2  collisions is nπd 2 V or the time between two successive
∴ U = 3 RT for one mole 1
collisions is on the average. τ =  2
∴ C=
∆Q ∆U
= = 3R ( nπ V d )
∆T ∆T
• The above equation is called as Dulong and Petit’s The average distance between two successive collisions,
Law. called the mean free path l, is:
• At low temperatures the vibrational mode may not  1
=l V= τ
be that active hence, heat capacity is low at low ( nπd 2 )
temperatures for solids.
In this derivation, we imagined the other molecules to be
at rest. But actually all molecules are moving and the
collision rate is determined by the average relative

velocity of the molecules. Thus we need to replace V by

2V in equation. A more exact treatment.
 1
=
Fig 15.6
( 2nπd 2 )

SCAN CODE
KINETIC THEORY OF GASES & THERMODYNAMICS
KINETIC THEORY OF GASES AND THERMODYNAMICS 61

Result Equation of State: The equation which connects the


 1
pressure (P), the volume (V) and absolute temperature
=
( )
(T) of a gas is called the equation of state.
2nπd 2
PV = constant (Boyle’s law)
for N molecules PV = NKBT V
= cons tan t (Charle’s law)
T
N P
⇒n= = ⇒ PV = nRT
V K BT
Thermodynamic Process: A thermodynamic process is
 K BT
= said to take place when some changes occur in the state
2πd 2 P of a thermodynamic system, i.e., the thermodynamic
parameters of the system change with time. Types of
NOTE: these thermodynamic process are Isothermal, Adiabatic,
Mean free path depends inversely on the number density Isobaric and Isochoric.
and size of the molecule. Quasi Static Process: A thermodynamic process which
is infinitely slow is called as quasi-static process.
4. INTRODUCTION TO THERMODYNAMICS • In quasi static process, system undergoes change so
slowly, that at every instant, system is in
It is the study of interrelations between heat and other equilibrium, both thermal and mechanical, with the
forms of energy surroundings.
Thermodynamic System: A collection of large number • Quasi-static process is an idealised process. We
of molecules of matter (solid, liquid or gas) which are so generally assume all the processes to be quasistatic
arranged that these possess certain values of pressure, unless stated.
volume and temperature forms a thermodynamic system. Indicator or P-V, Diagram: A graph between pressure
• The parameters pressure, volume, temperature, and
internal energy etc which determine the state or volume of a gas under thermodynamic operation is called
condition of system are called thermodynamic state P-V. diagram.
variables.
• In thermodynamics we deal with the thermodynamic
systems as a whole and study the interaction of heat
and energy during the change of one thermodynamic
state to another.
Thermal Equilibrium
The term ‘equilibrium’ in thermodynamics implies the Fig 15.8
state when all the macroscopic variables characterising a → Isobaric
the system (P, V, T, mass etc) do not change with time. b → Isothermal
• Two systems when in contact with each other come c → Adiabatic
to thermal equilibrium when their temperatures d → Isochoric
become same. Area under P – V diagram gives us work done by a gas.
• Based on this is zeroth law of thermodynamics. 4.2 Heat, Work Done and Internal Energy of Gas
According to zeroth law, when the thermodynamic
systems A and B are separately in thermal Internal Energy is the energy possessed by any system
equilibrium with a third thermodynamic system C, due to its molecular K.E. and molecular P.E. Here K.E
then the systems A and B are in thermal equilibrium and PE are with respect to centre of mass frame. This
with each other also. internal energy depends entirely on state and hence it is
a state variable. For a real gases internal energy is only
4.1 Basic Terms of Thermodynamics by virtue of its molecular motion.
State Variables: P, V, T, no. of moles and internal nfRT
U= for ideal gases where
energy They can be extensive or intestive. 2
n = number of moles

SCAN CODE
KINETIC THEORY OF GASES & THERMODYNAMICS
KINETIC THEORY OF GASES & THERMODYNAMICS 62

f = Degree of freedom 5.1 Relation of Heat and Internal Energy


R = Universal Gas Constant Let ∆Q = Heat supplied to the system by the
T = Temperature in Kelvin surroundings
Internal Energy can be change either by giving heat
energy or by performing some work. ∆W = Work done by the system on the surroundings
Heat Energy is the energy transformed to or from the ∆U = Change in internal energy of the system.
system because of the difference in temperatures by First law of thermodynamics states that energy can
conduction, convection or radiation. neither be created nor be destroyed. It can be only
The energy that is transferred from one system to another transformed from one form to another.
by force moving its point of application in its own
direction is called work. Mathematically: ∆Q = ∆U + ∆W
Sign Conventions:
• When heat is supplied to the system, then ∆Q is
positive and when heat is withdrawn from the
system, ∆Q is negative.
• When a gas expands, work done by the gas is
positive and when a gas contracts then work is
negative.
• ∆U is positive, when temperature rises and ∆U is
Fig 15.9
negative, when temperature falls. Remember here
Work done by the system = ∫ F dx we always take work done by the system.
= ∫ Ps Adx 5.2 Mayer’s Formula
= ∫ Ps dV  dQ 
CP =  
Where Ps is the Pressure of system on the piston. This  dT  P
work done by system is positive if the system expands Or, dQ = C P dT
and it is negative if the system contracts.
From equation
• Work and Heat are path functions whereas internal
= CP dT
dQ = dU + PdV
energy is a state function.
Again, from equation (2) substituting
• Heat and work are two different terms through they
might look same. dU = CV dT
C=
P dT CV dT + PdV... ( 4 )
5. FIRST LAW OF THERMODYNAMICS For one mole of gas ( µ =1) , from ideal gas equation,
The first law of thermodynamics is a thermodynamics- PV = RdT
adapted version of the law of conservation of energy. In PdV = RdT
principle, the conservation law asserts that an isolated From equations
system's total energy remains constant; energy can be ( CP − CV ) dT =
RdT
transferred from one form to another, but it cannot be
Or, CP − C V =
R
created or destroyed.
Where C P is the Specific heat at constant pressure and
The first law states that the change in internal energy of
the system ( ∆U system) is equal to the difference CV is the specific heat at constant volume.
between the heat provided to the system (Q) and the work
(W) done by the system on its surroundings in a closed
system (i.e., there is no transfer of matter into or out of
the system).
∆U system =
Q−W

SCAN CODE
KINETIC THEORY OF GASES & THERMODYNAMICS
KINETIC THEORY OF GASES AND THERMODYNAMICS 63

6. CYCLIC AND NON-CYCLIC PROCESSES Ti = Tf


⇒ Ui =
Uf
6.1 Introduction to Cyclic and Non-Cyclic Processes Or, ∆U net =
0
Cyclic Process: A cyclic process is one in which the If there are three process in a cyclic abc, then
system returns to its initial stage after undergoing a series ∆U ab + ∆U bc + ∆U ca = 0
of changes. From first law of thermodynamics,
Indicator Diagram Q= W + ∆U,if ∆U net= 0, then
Q net = Wnet
Or, Qab + Q bc + Qca = Wab + Wbc + Wca
Further, Wnet = area under P-V diagram. For example,
Wnet = + area of triangle ‘abc’ in the shown diagram.
Cycle is clockwise. So, work done will be positive.
6.3 Reversible and Irreversible Processes

Fig 15.10 Reversible Process: A reversible process is the process


∆U = 0 where it never occurs; on the contrary the irreversible
W = Area enclosed by the loop. process is the one which can be said to be the natural
Q = W as per First Law of thermodynamics process and cannot be reversed.
Here W is positive if the cycle is clockwise and it is Thermodynamics is the example of the reversible
negative if the cyclic is anti clockwise. process. Here the system and the surroundings return to
Non-Cyclic Process: In Non-cyclic process the series of the same stage at the end of the process.
changes involved do not return the system back to its NOTE:
initial state. A Reversible process takes two processes into account
i.e., change in internal energy for cyclic process is zero while in the first process participants convert into
and also ∆U ∝ ∆T ⇒ ∆T =0 another form, tin the case of this second process the
i.e., temperature of system remains constant. i.e, heat reverse reaction takes space where the resultants get back
supplied is equal to the work done by the system. to the initial state.
6.2 Work done and Heat exchange in Cyclic Types of reversible processes: There are two types of
Processes
reversible processes. The internally reversible process
In cyclic process, initial and final points are same. and the external reversible process. Internal reversible
process involves no irreversibility within the system
boundaries. This states that the system undergoes the
stage of equilibrium but when it returns it again passes
through the same stage.
• In the externally reversible process there are no
irreversibility’s
Irreversible Process: an irreversible process is a
naturally occurring phenomenon, which does not go back
to its original state.
Factors behind Irreversibility of process:
An irreversible process can be said to be the
thermodynamics process that departs equilibrium. When
Fig 15.11
we talk in terms of pressure, we can say that it occurs
Therefore, ( pi Vi , Ti ) = ( p f , Vf , Tf )
when the pressure of the system changes and the volume
Internal energy is a state function which only depends on does not have time to reach equilibrium.
temperature (in case of an ideal gas).

SCAN CODE
KINETIC THEORY OF GASES & THERMODYNAMICS
KINETIC THEORY OF GASES & THERMODYNAMICS 64

The system and the surrounding does not come back to P 


the original state even after the completion of the process Therefore, W = nRT ln  1 
 P2 
in the spontaneous process.
Hence, The Reversible Nature of a Process is Dependent First Law of Thermodynamics
on Multiple Factors Such as non-elasticity, friction, Q =∆U + W
viscosity, electrical resistance etc.
V2
⇒Q=
nRT ln
7. THERMODYNAMIC PROCESSES V1
NOTE:
7.1 Isothermal Process
All the heat supplied is used entirely to do work against
Description: A thermodynamic process in which external surroundings. If heat is supplied then the gas
temperature remains constant expands and if heat is withdrawn then the gas contracts.
Condition: The walls of the container must be perfectly
conducting to allow free exchange of heat between gas Practical Examples:
and its surroundings. Melting of ice at 0°C
The process of compression or expansion should be slow
Boiling of water at 100°C
so as to provide time for exchange of heat.
7.2 Adiabatic Process
These both conditions are perfectly ideal.
Description: When there is no heat exchange with
Equation of State: T = Constant or PV = Constant
surroundings.
Indicator Diagram: Conditions: The walls of the container must be perfectly
non-conducting in order to prevent any exchange of heat
between the gas and its surroundings.
The process of compression or expansion should be
rapid, and so, there is no time for the exchange of heat.
These conditions are again ideal condition and are hard
to obtain
Equation of State:
Fig 15.12 PV γ = constant

dP or TV γ −1 = constant
Slope of P – V curve is at any point. γ
dV
or PT 1−γ = constant
PV = nRT Indicator Diagram
⇒ ( dP ) V + P ( dV ) =
0

dP P
⇒ =

dV V
∆U = 0 (Temperature remains constant)
v2

W= ∫ Pg dV
v2
Fig 15.13
dP
v2
nRT Slope of adiabatic curve =
= ∫v V dV [Using PV = nRT] dV
2
PV γ = const
V2
= nRT ln ⇒ PγV γ −1 ( dv ) + ( dP ) Vγ =0
V1

Since P1V1 = P2 V2 dP −γP


⇒ =
dV V

SCAN CODE
KINETIC THEORY OF GASES & THERMODYNAMICS
KINETIC THEORY OF GASES AND THERMODYNAMICS 65

As shown in graph adiabatic curve is steeper than


isothermal curve.
nfRdT nR ( T2 − T1 ) P2 V2 − P1V1
=
∆U = =
2 γ −1 γ −1
Work Done by Gas: If a gas adiabatically expands from Fig 15.14
V1 to V2
v2
nfR∆T
W= ∫ P ( dv )
v
∆U =
2
2
Work
v2
dV W = 0 as gas does not expands
= cons tan t ∫ γ First Law of thermodynamics
v2 V
Q =∆U + W
 PV γ = constant  nfR∆T
  ⇒Q=
⇒ P = constant  2
 Vγ 
NOTE:
V2
V  −γ+1
constant  1 1  Since we have studied earlier, that when heat is supplied
= constant
= ×   γ−1 − γ−1  to any process. Its temperature increases according to
 1 − γ  V1 1 − γ  2
V V1 
relation.
Also we know

P= γ
P= γ = nC∆T
Q
1V1 2 V2 constant
Q
1  P2 V2γ P1V1γ 
⇒C= .... (1)
⇒ − n∆T
 
1 − γ  V2γ −1 V1γ −1  Now this C depends upon external conditions for gases.
∆Q 
P2 V2 − P1V1 nR ( T1 − T2 ) Here it is referred as ( 2)
=W = n∆T  v
γ −1 γ −1
i.e. Molar heat capacity at constant volume Comparing
First Law of Thermodynamics equation 1 and 2
Q =∆U + W fR
We get C v = ... ( 3)
Substituting the values 2

We get Q = 0 7.4 Isobaric Process

NOTE: Description: When pressure remains constant


If gas expands adiabatically then its temperature Condition: When in one container, the piston is free to
decreases and vice versa. move and is not connected by any agent.
Equation of State: P = constant
Practical Example V
• Propagation of sound waves in the form of = cons tan t
T
compression and rarefaction. Indicator Diagram:
• Sudden bursting of a cycle tube.
7.3 Isochoric Process
Description: Volume remains constant
Condition: A gas being heated or cooled inside a rigid
container. Fig 15.15
P nfR∆T
Equation of State: V = constant or = constant ∆U = same as always
T 2

SCAN CODE
KINETIC THEORY OF GASES & THERMODYNAMICS
KINETIC THEORY OF GASES & THERMODYNAMICS 66

W= ∫ PdV= P∆V (as pressure is constant) 7.5 Melting Process


=PV2 − PV1 =nR∆T In any case first law is always applicable
First Law of Thermodynamics Q = mLf as learned earlier.
Q =∆U + W W=0
nfR∆T (In the change of state from solid to liquid we ignore any
⇒= Q + nR∆T
2 expansion or contraction as it is very small)
 fR 
⇒Q = n  + R  ∆T ... ( 4 ) According to first law of thermodynamics
 2  ∆U = Q − W
Similar to Cv, we can define molar heat capacity at ∆U =mLf
constant pressure
Q  NOTE:
⇒ Cp =  ... ( 5 )
n∆T  p The heat given during melting is used in increasing the
internal energy of any substance
From equation 4 and 5
fR
+ R ... ( 6 )
7.6 Boiling Process
We get C=
p
2
Here, Q = mL v
From equation 3 and 6
fR W P [ V2 − V1 ]
=
Replacing by Cv we get
2 (Pressure is constant during boiling and it is equal to
V=
p Cv + R atmosphere pressure)
⇒ ∆U= Q − W
which is also called Mayer’s Relation.
Similar to molar specific heat at constant pressure and ∆U= mL v − P ( V2 − V1 )
molar specific heat at constant volume, we can define 7.7 Polytropic Process
molar specific heat for any process. A polytropic process is a thermodynamic process that
For example: obeys the relation:
Cadiabatic = 0 PV n = C
Cisothermal = ∞ Where P is the pressure, V is volume, n is the polytropic
Basically gas does not possess a unique specific heat. index, and C is a constant. The polytropic process
Mainly we have Cp and C v . equation can describe multiple expansion and
compression processes which include heat transfer.
• Specific Heat at Constant Volume: It is defined as Particular Cases:
the amount of heat required to raise the temperature Some specific values of n correspond to particular cases:
of 1g of a gas through 1°C, when its volume is kept n = 0 for an isobaric process,
constant. It is denoted as CV. n = +∞ for an isochoric process
• Specific Heat at Constant Pressure: It is defined In addition, when the ideal gas law applies:
as the amount of heat required to raise the n = 1 for an isothermal process,
temperature of 1g of a gas through 1°C keeping its n = γ for an isentropic process.
pressure constant. It is denoted as Cp.
Where γ is the ratio of the heat capacity at constant
NOTE: pressure ( CP ) to heat capacity at constant volume ( C v )
c P , c v means Molar heat Capacity and CP , C V means 7.8 Free-Expansion
specific heat capacity A process in which gas is allowed to expand in vacuum
=CV Mc = V and C P Mc P where M stands for molar and this happens so quickly that no heat leaves or enters
mass of any sample. the system this type of process is also known as adiabatic
R process and because this happens so fast the gas does not
cp − c v =
M cross the system boundaries, hence no work is done by
the system or on the system, then the expansion is called
the free expansion.

SCAN CODE
KINETIC THEORY OF GASES & THERMODYNAMICS
KINETIC THEORY OF GASES AND THERMODYNAMICS 67

We can for the equate free expansion into 8.1 Thermal Efficiency
U f − U i =Q − W Thermal Efficiency of a heat engine is defined of the
Now, as know heat is exchange and no work is done ratio of net work done per cycle by the engine to the total
Q = 0 and W = 0 i.e., U f = U i
amount of heat absorbed per cycle by the working
substance from the source.
Further, as we see there is no change in the internal
W
energy, Hence, the temperature remains constant. It is denoted by η = ... (1)
Q1
7.9 Limitation of First Law of Thermodynamics Using equation 1 and 2 we get
• Q
The first law does not indicate the direction in which η = 1− 2 ... ( 2 )
the change can occur. Q1
• The first law gives no idea about the extent of Ideally engines should have efficiency = 1
change.
NOTE:
• The first law of thermodynamics gives no
The mechanism of conversion of heat into work vanes
information about the source of heat. i.e. whether it
for different heat engines.
is a hot or a cold body.
The system heated by an external furnace, as in a steam
8. HEAT ENGINE engine. Such engines are called as external combustion
engine.
The system in which heat is produced by burning the fuel
It is a device that converts heat energy into mechanical
inside the main body of the engine is called as Internal
energy.
Combustion Engine.
Key Elements:
• A source of heat at higher temperature 9. CARNOT CYCLE
• A working substance
Sadi Carnot devised on ideal cycle of operation for a heat
• A sink of heat at lower temperature.
engine called as Carnot cycle.
Working:
• The working substance goes through a cycle Engine used for realising this ideal cycle is called as
consisting of several processes. Carnot heat engine.

• In some processes it absorbs a total amount of heat The essential parts of an Ideal heat engine are shown in
Q1 from the source at temperature T1. figure.

• In some processes it rejects a total amount of heat Q2


to the sink at some lower temperature T2.
• The work done by the system in a cycle is transferred
to the environment via some arrangement.
Schematic Diagram

Fig 15.17
• Source of heat: The source is maintained at a fixed
higher temperature T1, from which the working
substance draws heat. The source is supposed to
Fig 15.16 possess infinite thermal capacity and as such any
amount of heat can be drawn from it without
First Law of Thermodynamics changing its temperature.
⸪ Energy is always conserved • Sink of heat: The sink is maintained at a fixed lower
⇒ Q1 = W + Q 2 temperature T2, to which any amount of heat can be
rejected by the working substance.

SCAN CODE
KINETIC THEORY OF GASES & THERMODYNAMICS
KINETIC THEORY OF GASES & THERMODYNAMICS 68

It has also infinite thermal capacity and as such its • Adiabatic Expansion: The cylinder is now
temperature remains constant at T2, even when any removed from source and is placed on the perfectly
amount of heat is rejected to it. insulating pad. The gas is allowed to expand further
• Working substance: A perfect gas acts as the from B (P2, V2) to C (P3, V3). Since the gas is
working substance. It is contained in a cylinder with thermally insulated from all sides, therefore the
non-conducting sides but having a perfectly R (T − T )
conducting-base. This cylinder is fitted with processes is adiabatic q 2 = 0 ∆U 2 = 2 1
γ −1
perfectly non-conducting and frictionless piston.
R ( T1 − T2 )
• Apart from these essential parts, there is a perfectly =W2 = Area BCNMB
insulating stand or pad on which the cylinder can γ −1
be placed. It would isolate the working substance • Isothermal Compression: The cylinder is now
completely from the surroundings. Hence, the gas removed from the insulating pad and is placed on the
can undergo adiabatic changes. sink at a temperature T2. The piston is moved slowly
The Carnot cycle consists of the following four so that the gas is compressed until is pressure is P4
stages: and volume is V4.
• Isothermal expansion ∆U 3 = 0
• Adiabatic expansion V
W3 =
−RT2 ln 4 =
−Area CDLNC
• Isothermal compression V3
• Adiabatic compression
V4
The cycle is carried out with the help of the Carnot q 3 = −RT2 ln
V3
engine as detailed below:
q3 = Heat absorbed in this process
W3 = Work done by gas
• Adiabatic Compression: The cylinder is again
placed on the insulating pad, such that the process
remains adiabatic. Here the gas is further
compressed to its initial P1 and V1.
R (T − T )
∆U 4 = 1 2
γ −1
−R ( T1 − T2 )
Fig 15.18 W4 = = −area DAKLD
γ −1
Consider one gram mole of an ideal gas enclosed in the
cylinder. Let V1, P1, T1 be the initial volume, pressure q4 = 0
and temperature of the gas. The initial state of the gas is W4 = work done by the gas
represented by the point A on P−V. diagram, We shall 9.1 Analysis of Carnot Cycle
assume that all the four processes are quasi-static and
dissipative, the two conditions for their reversibility. Total work done by the engine per cycle.
Steps = W1 + W2 + W3 + W4
• Isothermal Expansion: The cylinder is placed on = W1 + W3
the source and gas is allowed to expand by slow V2 V
outward motion of piston. Since base is perfectly =W RT1 ln − RT2 ln 4
V1 V3
conducting therefore the process is isothermal.
Now Q1 = Total heat absorbed = q1
∆U1 = V
0 = RT1 ln 2
V1
V2
= =
q1 W1 RT1 ln = Area ABMKA Q2 = Total heat released = −q3
V1
[q3 = Heat absorbed and not heat released]
q1 → Heat absorbed by gas
V
W1 → Work done by gas = RT2 ln 3
V4

SCAN CODE
KINETIC THEORY OF GASES & THERMODYNAMICS
KINETIC THEORY OF GASES AND THERMODYNAMICS 69

We can see that for heat engine Step - 3: Let engine I absorb Q1 heat from the source
W= Q1 − Q 2 deliver work W1 and release the balance Q1 − W1 to the
= Area under ABCDA sink in one cycle.

9.2 Efficiency of Carnot Engine

W Q
η= = 1− 2
Q1 Q1
Now steps 2 is adiabatic and step 4 is also adiabatic
⇒ T1V2γ −1 =
T2 V3γ −1
and T1V1γ −1 = T2 V4γ −1 Fig 15.19
Step - 4: Arrange R, such that it returns same heat Q, to
V2 V3
⇒ = ... ( 21) the source, taking Q2 from the sink and requiring work
V1 V4 W = Q1 – Q2 to be done on it.
From equation 19, 20 and 21 we get Step - 5: Suppose ηR < ηI (i.e.) If R were to act as an
Q1 T1
= engine it would give less work output than that of I (i.e.)
Q 2 T2 W < W1 for a given Q1 and Q1 – W > Q1 – W1
T Step - 6: In totality, the I-R system extracts heat (Q1 −
∴ n Carnot =
1− 2
T1 W) − (Q1 − W1) = W1 − W and delivers same amount of
work in one cycle, without any change in source or
• ηCarnot engine − depends only upon source
anywhere else. This is against second Law of
temperature and sink temperature. Thermodynamics. (Kelvin - Planck statement of second
• ηCarnot engine =
1 only when T2 = 0 K or T1 = ∞ which law of thermodynamics)
is impossible to attain. Hence the assertion η1 > ηR is wrong.
• If T2= T1 ⇒ η= 0 ⇒ Heat cannot be converted to Similar argument can be put up for the second statement
mechanical energy unless there is some difference of Carnot theorem, (i.e) Carnot efficiency is independent
between the temperature of source and sink. of working substance.
∴ We use ideal gas for calculating but the relation.
9.3 Carnot Theorem
Q1 T1
= will always hold true for any working
Statement: Carnot theorem states that all reversible Q 2 T2
engines working between same two temperatures have
substance used in a Carnot engine.
same efficiency irrespective of the nature of working
substance. The source and the sink works between the 9.4 Second Law of Thermodynamics
same temperature. The second law of thermodynamics states that the heat
• Working between two given temperatures, T1 of hot energy cannot transfer from a body at a lower
reservoir (the source) and T2 of cold reservoir (the temperature to a body at a higher temperature without the
sink), no engine can have efficiency more than that addition of energy.
of the Carnot engine. There are number of ways in which this law can be
• The efficiency of the Carnot engine is independent stated. Though all the statements are the same in their
of the nature of the working substance. contents, the following two are significant.
Engine used for realizing this ideal cycle is called as Kelvin Planck Statement: No process is possible whose
Carnot heat engine. sole result is the absorption of heat from a reservoir and
Proof: the complete conversion of the heat into work.
Step - 1: Imagine a reversible engine R and an Clausius Statement: No process is possible whose sole
irreversible engine-I working between the same source result is the transfer of heat from a colder object to a
(hot reservoir T1) and sink (cold reservoir T2). hotter object.
Step - 2: Couple two engines such that I acts like heat Significance: 100% efficiency in heat engines or infinite
engine and R acts like refrigerator. CoP in refrigerators is not possible.

SCAN CODE
KINETIC THEORY OF GASES & THERMODYNAMICS
KINETIC THEORY OF GASES & THERMODYNAMICS 70

10. REFRIGERATION 10.1 Coefficient of Performance


Coefficient of Performance of refrigerator (β) is defined
A refrigerator or heat pump is a device used for cooling
as the ratio of quantity of heat removed per cycle from
things.
contents of the refrigerator (Q2) to the energy spent per
Key Elements:
cycle (W) to remove this heat.
• A cold reservoir at temperature T2.
Q
• A working substance. β= 2 ... ( 2 )
W
• A hot reservoir at temperature T1.
Using equation 1 and 2 we get
Working
Q2
• The working substance goes through a cycle β=
consisting of several process. Q1 − Q 2
• A sudden expansion of the gas from high to low Ideally heat pumps should have β = ∞
pressure which cools it and converts it into a
vapour-liquid mixture.
• Absorption by the cold fluid of heat from the region
to be cooled, converting it into vapour.
• Heating up of the vapour due to external work done
on the working substance.
• Release of heat by the vapour to the surroundings
bringing it to the initial state and completing the
cycle.
Schematic Diagram.

Fig 15.20

First Law of Thermodynamics


Q2 + W =
Q1 ... (1)

SCAN CODE
KINETIC THEORY OF GASES & THERMODYNAMICS
KINETIC THEORY OF GASES & THERMODYNAMICS 71

Solved Examples

pV
Example - 1 Sol. (a) The dotted plot shows that is a constant
T
Estimate the fraction of molecular volume to the actual quantity = (µR).
volume occupied by oxygen gas at STP. Take the
diameter of an oxygen molecule to be 3 Å. This signifies the ideal gas behaviour.

Sol. At S.T.P., actual volume occupied by 1 mole of (b) Here T1 > T2


oxygen is: (c) At the point where the curve meets the y-axis,
V = 22400 ml pV
= 22400 cm3 we have = µR, where µ is the number of moles
T
Also, molecular volume, of oxygen gas.
4
V=′ πR 3 N Here, Mass of oxygen,
3

=
where R
D 3 o
= A m 1.00 × 10−3 kg
=
2 2
3 Also, molecules mass,
=× 10−8 =
1.5 × 10−8 cm
2
M= 32 × 10−3 kg
=
and N 6.023 × 10 23

Hence, V ′ = × 3.143 × (1.5 × 10−8 ) × 6.023 × 1023


4 3 m 1.00 × 10−3 1
No. of moles, µ
= = =
3 M 32 × 10−3 32
= 8.52 cm3 pV 1
=µR = × 8.31 =0.26 J k −1
Hence, fraction of molecular volume to actual volume T 32
is
V′ 8.52
= = = 3.8 × 10−4 ≈ 4 × 10−4.
V 22400 pV
(d) Since the value of depends upon the number
Example - 2 T

Figure shows plot of pV/T versus p for 1.00 × 10−3 kg pV


of moles, we will not get the same value for
of oxygen gas at two different temperatures. T
in case of hydrogen. To obtain the same value of
 1  pV
µ  i.e.,  and hence , we must have
 32  T

Mass of hydrogen
=
Molecular mass of hydrogen

1
(a) What does the dotted plot signify? =
(b) Which is true: T1 > T2 or T1 < T2? 32
(c) What is the value of pV/T where the curves meet Hence, mass of hydrogen
on the y-axis.
(d) If we obtained similar plots for 1.00 × 10−3 kg of =
1
× molecular mass
hydrogen, would we get the same value of pV/T 32
at the point where the curves meet on the y-axis? 1
If not, what mass of hydrogen yields the same = × 2 × 10−3 kg
32
value of pV/T (for low pressure high temperature
region of the plot)? (Molecular mass of = 6.25 × 10−5 kg
H2 = 2.02u, of O2 = 32.0 u, R = 8.31 J mol−1 K−1).
KINETIC THEORY OF GASES & THERMODYNAMICS 72

Example - 3 Example - 4
An oxygen cylinder of volume 30 litres has an initial An air bubble of volume 1.0 cm3 rises from the
gauge pressure of 15atm and a temperature of 27°C. bottom of a lake 40 m deep at temperature of 12°C.
After some oxygen is withdrawn from the cylinder, To what volume does it grow when it reaches the
the gauge pressure drops to 11atm and its temperature surface, which is at a temperature of 35°C?
drops to 17°C. Estimate the mass of oxygen taken out Sol. Volume of the bubble inside,
of the cylinder.
V1 = 1 cm3 = 1× 10−6 m3
(R = 8.31 J mol−1 K−1, molecular mass of O2 = 32u).
Sol. Under the initial conditions, Pressure on the bubble = Pressure of water +
atmospheric pressure,
V = 30 litre = 30 × 10−3m3
P = 15 atm = 15 × 1.01 × 105 Pa =ρgh + 1.01× 105
T = 27°C = 273 + 27 = 300 K = 1000 × 9.8 × 40 + 1.01× 105 = 4.93 × 105 Pa
Also, R = 8.31 J mol−1 K−1 and molar mass, Temperature,
M = 32 × 10−3 kg. T1 = 12°C = 273 + 12 = 285 K
Using the relation,
Also, pressure outside the lake
pV = nRT
p 2 1.01× 105 Nm −2
=
pV
⇒n= Temperature,
RT
T = 35°C = 35 + 273 = 308 K
15 × 1.01× 105 × 30 × 10−3
= 18.23
8.31× 300 Volume, V2 = ?

m p1V1 p 2 V2 p1V1 T1
Now, n = Now, = or =
V2 ×
M T1 T2 T1 p 2

⇒ m = nM = 18.23 × 32 × 10−3 kg 4.93 × 105 × 1× 10−6 × 308


= = 5.3 × 10−6 m3 .
285 × 1.01× 105
or m = 0.58 kg
Example - 5
Under the final condition:
Estimate the average thermal energy of a helium atom
V=′ 30 litre= 30 × 10−3 m3
at (i) room temperature (27°C), (ii) the temperature
P' =11atm =11× 1.01× 105 Pa on the surface of the Sun (6000 K), (iii) the
temperature of 10 million kelvin (the typical core
T ′ = 17°C = 17 + 273 = 290 K temperature in the case of a star).
P 'V ′ Sol. The average thermal energy of an atom is
We have, n ' =
RT ′ 3
−3
E= k BT
11× 1.01× 10 30 × 10
5
2
= 13.83
8.31× 290 k B 1.38 × 10−23 J K −1 is the Boltzmann’s
where =
m′ = n 'M const.
−3
= 13.83 × 32 × 10 kg= 0.44 kg (i). At room temperature,
T = 27°C = 27 + 273 = 300 K
Mass of the oxygen taken out of the cylinder
3
=m − m′ =0.58 − 0.44 =0.14 kg E = × 1.38 × 10−23 × 300 =6.21× 10−23 J
2
(ii). At the temperature on the surface of the sun,
i.e. T = 6000 K
KINETIC THEORY OF GASES & THERMODYNAMICS 73

3 1 1
E = × 1.38 × 10−23 × 6000 =
1.24 × 10−19 J = =
2πnd 2 2πn ( 2r )
2
2
(iii). At
1
T = 10 million Kelvin =
( )
2
= 10 × 106 K. 1.414 × 3.14 × 5.06 × 1025 × 2 × 1× 10−10
3 = 1.1× 10−7 m
E = × 1.38 × 10−23 × 10 × 106
2
3RT
= 2.1× 10−16 J. Also, v rms =
M
Example - 6
3 × 8.31× 290
Estimate the mean free path and collision frequency of = = 5.08 × 102 ms −1
28 × 10−3
a nitrogen molecule in a cylinder containing nitrogen
Collision frequency,
at 2.0 atm and temperature 17°C. Take the radius of a
nitrogen molecule to be roughly 1.0 Å. Compare the v rms 5.08 × 102
= =
collision time with the time the molecule moves freely λ 1.1× 10−7
between two successive collisions (Molecular mass of Time between successive collisions
N2 = 28.0 u).
1 1
= = = 1.97 × 10−10 s
Sol. Here, p = 2 atm v rms 5.08 × 109

=
2 × 1.013 × 105 Pa Also, the collision time
d 2r
= 2.023 × 105 Pa = =
v rms v rms
T= 17°C
2 × 1× 10−10
= = 3.94 × 10−29 s.
=17 + 273 =290 K 5.08 × 109
o Example - 7
Radius, R = 10 A = 1× 10−10 m
Consider in ideal gas with following distribution of
Molecular, mass (m) speeds
=28u =28 × 1.66 × 10−27 RMS Speed (m/s) % of molecules
200 10
= 4.65 × 10−26 kg
400 20
−1 −1
Also, R = 8.31 J mole K
600 40
k B 1.38 × 10−23 J K −1
= 800 20
Now, for one mole of a gas, 1000 10
pV = RT (i) Calculate Vrms and hence T
(m = 3 × 0 × 10−26 kg)
RT (ii) If all the molecules with speed 1000 m/s escape
or V =
p from the system, calculate new Vrms and hence
T.
8.31× 290
= Sol. (i) By definition,
2.026 × 105
=
or V 1.189 × 10−2 m3 ∑n V i i
2

V 2
= i

∑n
rms
Number of molecules per unit volume, i
N
n= 10 ( 200 ) + 20 ( 400 ) + 40 ( 600 ) + 20 ( 800 ) + 10 (1000 )
2 2 2 2 2
V =
10 + 20 + 40 + 20 + 10
6.023 × 1023
⇒ n= = 5.06 × 1025 m −3 105 [ 4 + 32 + 144 + 128 + 100]
1.189 × 10−2 V=2
rms = 408 × 103
100
Now, mean free path, λ
KINETIC THEORY OF GASES & THERMODYNAMICS 74

Vrms = 408 × 103 = 6.39 × 102 m / s


Example - 10
1 3
As mv 2rms = kT
2 2 Two different adiabatic parts for the same gas
intersect two isothermals at T1 and T2 as shown in P-
V diagram. How does the ratio (Va/Vd) compare with
mv 2rms 3 × 10−26 × 4.08 × 105 the ratio (Vb/Vc)?
∴ T= = −23
= 2.96 × 102 K.
3k 3 × 1.38 × 10

(ii) When all the molecules with speed 1000 m/s


escape from the system.
10 ( 200 ) + 20 ( 400 ) + 40 ( 600 ) + 20 ( 800 )
2 2 2 2
2
Vrms =
10 + 20 + 40 + 20

105 ( 4 + 32 + 144 + 128 ) 308


= = × 104
90 9
308
Vrms = × 102 m / s =
5.85 × 102 m / s
For adiabatic curve BC : T1Vb( ) = T2 Vc( )
γ−1 γ−1
9 Sol.
2
mVrms 3 × 10−26 × 308 × 104 For adiabatic curve AD : T1Va( ) = T2 Vd( )
γ −1 γ −1
=
As above, T =
3k 3 × 1.38 × 10−23 Dividing, we get
T = 248 K. ( γ −1) ( γ −1)
 Va  V 
  = d 
 Vb   Vc 
Example - 8 Va Vb
or =
The temperature of the surface of sun is about 6000 Vd Vc
K. Can we produce a temperature of 7000 K by Va Vd
converging sun’s rays using a large convex lens? ∴ =
Vb Vc
Sol. No, as according to the second law of
thermodynamics, heat cannot be transferred on its i.e., the ratio remains the same.
own, from a body at lower temperature to another at
higher temperature. Theoretically, the maximum
temperature we can produce by this method is 6000 Example - 11
K. A quantity of air at normal temperature is compressed
(a) slowly (b) suddenly to one third of its volume.
Example - 9 Find the rise in temperature, if any, in each case,
γ = 1.4
A refrigerator is to maintain eatables kept inside at
1
9°C. If room temperature is 36°C, calculate the Sol. Here,=
V2 V1 ; ( T2 −=
T1 ) ?
coefficient of performance. 3
γ = 1.4, T1 = 0°C = 273 K
Sol. Given: T1 = 36°C = ( 36 + 273) = 309 K (a). When the gas is compressed slowly, the change
is isothermal, i.e., temperature remains constant.
T2 = 10°C = (10 + 273) = 283 K (b). When the compression is sudden, the change is
adiabatic,
T 283 283
T2 V2(
γ −1)
= T1V1(
γ −1)
⇒ COP = 2 = = =10.9
T1 − T2 309 − 283 26
( γ −1)
V 
T2 = T1  1 
 V2 
(1.4 −1)
= ( 3)
T2 273= = ( 3) 423.6 K
0.4
273
T2 − T=
1 426.6 − 273
= 150.6 K
= 150.6°C
KINETIC THEORY OF GASES & THERMODYNAMICS 75

Example - 12 Example - 14

Water of one kg mass at 373 K is converted into From what height must a block of ice be dropped in
steam at the same temperature and at atmospheric order that it may melt completely. It is assumed that
pressure. On boiling, 1 Litre. of water takes a volume the whole of energy is retained by ice.
of 1671 Litre. Calculate the change in internal energy (Latent heat of ice = 3.33 × 105 J/kg)
of the system, taking the heat of vaporisation to be
540 cal g −1 540. Sol. Let mass of ice block = m (kg)
Sol. Here, m = 1 kg = 103 g Height through which it is dropped = h (m)

Initial volume, V1 = 10 ml
3
PE of the block, W = mgh (J)

Final volume, =
V2 1671× 103 ml Since it is assumed that whole of this energy is
retained by ice and is converted into heat Q,
P = 1 atmosphere
Q = mgh ( J ) ... ( i )
= 1.013 × 106 dyne / cm 2
From first law of thermodynamics If this heat is to melt the ice, then
dU =dQ − dW =mL − P ( V2 − V1 ) J Q= ( mkg ) × ( 3.33 ×105 J / kg )
1.013 × 106 × 103 (1671 − 1)
=103 × 540 − = 3.33 × 105 m ( J ) ... ( ii )
4.2 × 107
dU = 540 × 103 − 40.16 × 103 From equation (i) and (ii),

= 499.84 × 103 cal. = 3.33 × 105 m


mgh
= 499.84 Kcal.
3.33 × 105
Example - 13 or h = or =
h 3.4 × 104 m
9.8
A heat engine operates between a cold reservoir at
temp. T2 = 300 K and a hot reservoir at temp. T1. It
takes 200 J of heat from hot reservoir and delivers Example - 15
120 J of heat to cold reservoir in a cycle. What could Is it possible to heat a room by means of a
be the minimum temperature of hot reservoir?
refrigerator? If so, is this method the most
Sol. Work done by the engine in one cycle advantageous from energetic point of view?
W = Q1 − Q 2 = 200 − 120 = 80 J
Sol. Yes, a refrigerator can remove heat from the outside
W 80 air and reject it into the room. If P is the power
=
η = = 0.40
Q1 200 consumed by the refrigerator and Q is the heat
removed from the outside air in a unit time, the heat
According to Carnot theorem, no engine can have
rejected into the room per second = P + Q.
efficiency greater than that of Carnot engine.

 T  300
∴ 0.40 ≤ 1 − 2  =1 −
 T1  T1

300
or ≤ 1 − 0.40 =0.60
T1

300
∴ T1 ≥ =
500 K
0.6
Hence, minimum temperature of hot reservoir is
T1 = 500 K
KINETIC THEORY OF GASES & THERMODYNAMICS 76

EXERCISE – 1: BASIC OBJECTIVE QUESTIONS


Introduction to Kinetic Theory of Gases 7. One mole of an ideal gas at an initial temperature of T
kelvin does 6 R joule of work adiabatically. If the ratio
1. Cooking gas containers are kept in a lorry moving with
of specific heats of this gas at constant pressure and at
uniform speed. The temperature of the gas molecules
inside will 5
constant volume is , the final temperature of the gas
(a) increase 3
(b) decrease will be
(c) remain same (a) (T + 4) K (b) (T – 4) K
(d) decrease for some, while increase for others (c) (T + 2.4) K (d) (T – 2.4) K
2. If the pressure of the gas is doubled at constant volume Speeds of Gas Molecules
and mass, the frequency of collision of the molecules
with the walls of a container will 8. The density of a gas is 6 × 10–2 kg/m3 and the RMS
(a) Not change (b) Increase four times velocity of the gas molecules is 500 m/s. The pressure
exerted by the gas on the walls of the vessel is
(c) Be doubled (d) Increase by 2 times
(a) 5 × 103 N/m2 (b) 0.83 × 10–4 N/m2
3. If pressure and temperature of an ideal gas are doubled
(c) 1.2 × 10–4 N/m2 (d) 30 N/m2
and volume is halved, the number of molecules of the
9. The mean free path of a gas molecule at 27°C is 2 cm.
gas
If the rms velocity of the gas at that temperature is
(a) become half (b) become 4 times
10 m/s, what is the time interval between two
(c) become two times (d) remain constant
successive collisions?
4. Real gases obey gas laws more closely at
1 1
(a) High pressure and low temperature (a) s (b) s
(b) Low pressure and high temperature 5 500
(c) High pressure and high temperature 1 1
(c) s (d) s
(d) Low pressure and low temperature 50 250
5. By what percentage should the pressure of a given 10. Mean free path (λ) is
mass of a gas be increased so as to decrease its volume (a) inversely proportional to the diameter of molecule
by 10 % at a constant temperature? (b) inversely proportional to the square of diameter of
(a) 8.1 % (b) 10.1 % molecule
(c) 9.1 % (d) 11.1 % (c) inversely proportional to the number of molecules
6. A cylinder containing an ideal gas is in vertical per unit volume
position and has a piston of mass M that is able to move (d) both b and c
up or down without friction, as shown in figure. If the 11. Four molecules of speed 2 km/s, 3 km/s, 4 km/s and
temperature is increased. 5 km/s. The RMS speed of these molecules in km/s is
(P and V are the initial pressure and volume of the gas 54
respectively) (a) (b) 3.5
4
54
(c) (d) 3 3
2

(a) both P and V of the gas will change


(b) only P will increase according to Charles law
(c) V will change but not P
(d) P will change but not V
KINETIC THEORY OF GASES & THERMODYNAMICS 77

12. A cubic vessel (with faces horizontal + vertical) 16. 1 mole of an ideal gas is contained in a cubical vessel,
contains an ideal gas at NTP. The vessel is being ABCDEFGH at 300 K, figure.
carried by a rocket which is moving at a speed of B C
500 m/s in vertical direction. The pressure of the gas
inside the vessel as observed by us on the ground: A D
(a) remains the same because 500 m/s is very much
smaller than vrms of the gas F G
(b) remains the same because motion of the vessel as a E
whole does not affect the relative motion of the gas H
molecules and the walls One face (EFGH) of the vessel is made up of a material
which totally absorbs any gas molecule incident on it.
+ ( 500 )
2 2
vrms
(c) will increase by a factor equal to 2
, At any given time:
vrms (a) the pressure on EFGH would be zero
where vrms was the original root mean square (b) the pressure on all the faces will be equal
velocity of the gas (c) the pressure on EFGH would be double the pressure
(d) will be different on the top wall and bottom wall of on ABCD
the vessel (d) the pressure on EFGH would be half that on ABCD
17. If C p and Cv denote the specific heats of nitrogen per
Energy of gas molecules
unit mass at constant pressure and constant volume
13. The increase in internal energy of a gas per unit mass
respectively, then
per unit rise in temperature is equal to ( Cv and C p are
R
specific heat capacities at constant volume and = R
(a) CP – CV  28 (b) CP – C
 V =
28
pressure respectively)
R
(a) Cp (b) Cp + Cv (c) CP – C
 V = (d) CP – CV = R
 
14
(c) Cp – Cv (d) Cv
18. Two monoatomic gases are at absolute temperature
14. Which of the following statements is correct for any 300K and 350K respectively. The ratio of average
thermodynamic system? kinetic energies of their molecules is
(a) The internal energy changes in all processes
3.5 3
(b) Internal energy and entropy are state functions (a) (b)
3 3.5
(c) The change in entropy can never be zero
(d) The work done in an adiabatic process is always (c) 6 : 7 (d) 36 : 49
zero 19. A sample of oxygen and a sample of hydrogen have
15. A system goes from A to B via two process I and II as the same mass, volume and pressure. The ratio of their
absolute temperature is
shown in figure. If ∆U1 and ∆U 2 are the changes in
(a) 1/4 (b) 1/16
internal energies in the processes I and II respectively, (c) 4 (d) 16
then

(a) ∆U 2 > ∆U1


(b) ∆U 2 < ∆U1
(c) ∆U1 =
∆U 2
(d) relation between ∆U1 and ∆U 2 cannot be
determined
KINETIC THEORY OF GASES & THERMODYNAMICS 78

20. In the diagrams (i) to (iv) of variation of volume with 23. An air sample consists of 78% N2 and 22% oxygen.
changing pressure is shown. A gas is taken along the The values of Cv for N2 and O2 is 0.2 kcal/kg/K and
path ABCD. The change in internal energy of the gas 0.15 kcal/kg/K. Heat required to raise the temperature
will be of 1 kg of air through 1°C is
(a) 0.350 cal (b) 0.189 cal
(c) 0.350 kcal (d) 0.189 kcal
24. A thermally insulated vessel contains an ideal gas of
molecular mass M and ratio of specific heats γ . It is
moving with speed v and it is suddenly brought to rest
Assuming no heat is lost to the surroundings, its
temperature increases by
(γ − 1) γ Mv 2
(a) Mv 2 K (b) K
2γ R 2R
(γ − 1) (γ − 1)
(c) Mv 2 K (d) Mv 2 K
2R 2 ( γ + 1) R
25. The difference between two principal specific heats of
nitrogen is 300 J/kg.K and ratio of the two specific
heats is 1.4. Value of Cp (in J/kg.K ) is:
(a) Positive in all cases (i) to (iv) Introduction to Thermodynamics
(b) Positive in cases (i), (ii) and (iii) but zero in (iv) 26. The internal energy of an ideal gas depends on:
case (a) Pressure (b) Volume
(c) Negative in cases (i), (ii) and (iii) but zero in (iv) (c) Temperature (d) Size of the molecule
case 27. Which one of the following is not a thermodynamic
(d) Zero in all four cases co-ordinate?
21. Consider a process shown in the figure. During this (a) V (b) R
process the work done by the system (c) T (d) P
28. Which one of the following gases possesses the
largest internal energy
(a) 2moles of helium occupying 1m3 at 300 K
(b) 56kg of nitrogen at 107 Nm −2 and 300 K
(c) 8 grams of oxygen at 8atm and 300 K
(d) 6 × 1026 molecules of argon occupying 40 m3 at
(a) Continuously increases 900 K
(b) Continuously decreases 29. If the gases have initial temperature 300K and they are
(c) First increases, then decreases mixed in an adiabatic container having the same
(d) First decreases, then increases volume as the previous containers. Now the
22. Three perfect gases at absolute temperature T1, T2 and temperature of the mixture is T and pressure is P. Then
T3 are mixed. The masses of molecules are m1, m2 and
m3 and the number of molecules are n1, n2 and n3
respectively.
Assuming no loss of energy, the final temperature of
the mixture is (a) P > PA , T > 300 K (b) P > PB , T =
300 K
n1T1 + n2T2 + n3T3 nT +n T +n T
1 1
2
2 2
2
3 3
2
(c) P < PA , T =
300 K (d) P > PA , T < 300 K
(a) (b)
n1 + n2 + n3 n1T1 + n2T2 + n3T3
n12T12 + n22T22 + n32T32 (T1 + T2 + T3 )
(c) (d)
n1T1 + n2T2 + n3T3 3
KINETIC THEORY OF GASES & THERMODYNAMICS 79

30. The molar heat capacity in a process of a diatomic gas 34. A and B are two adiabatic curves for two different
Q gases. Then A and B correspond to
if it does a work of when a heat of Q is supplied to
4
it is
2 5
(a) R (b) R
5 2
10 6
(c) R (d) R
3 7
First Law of Thermodynamics
31. A gas is compressed at a constant pressure of 50 N/m2
(a) Ar and He respectively (b) He and H2 respectively
from a volume of 10 m3 to a volume of 4 m3. Energy of
(c) O2 and H2 respectively (d) H2 and He respectively
100 J is then added to the gas by heating. Its internal
energy is: 35. In a given process for an ideal gas,
(a) increased by 400 J (b) increased by 200 J dW = 0 and dQ < 0. Then for the gas,
(c) increased by 100 J (d) decreased by 200 J (a) temperature will decrease
32. The slopes of the isothermal and adiabatic curves are (b) volume will increase
related as: (c) pressure will remain constant
(a) isothermal curve slope = adiabatic curve slope (d) temperature will increase
(b) isothermal curve slope = γ × adiabatic curve slope 36. Starting with the same initial conditions, an ideal gas
expands from volume V1 to V2 in three different ways.
(c) adiabatic curve slope = γ × isothermal curve slope
(d) adiabatic curve slope = (1/2) × isothermal curve The work done by the gas is W1 if process is
slope isothermal, W2 if isobaric and W3 if adiabatic. Then,
33. Consider P-V diagram for an ideal gas shown in figure (a) W2 > W1 > W3 (b) W2 > W3 > W1
P
(c) W1 > W2 > W3 (d) W1 = W2 = W3
1 37. At 27°C, a gas is compressed suddenly such that its
constant
P= pressure becomes (1/8)th of its original pressure. Final
V
temperature will be (γ = 5/3):
2
(a) 420 K (b) 300 K
V (c) –142°C (d) 327°C
Out of the following diagrams, which represents the T- 38. In an adiabatic change, the pressure P and temperature
P diagram?
T T of a diatomic gas are related by the relation P ∝ TC,
T
2
where c is equal to:
2 (a) 5/3 (b) 2/5
(i) (ii) (c) 3/5 (d) 7/2
1 1 39. The P-V diagram shows seven curved paths (connected
P P by vertical paths) that can be followed by a gas. Which
T T two of them should be parts of a closed cycle if the net
work done by the gas is to be at its maximum value
2 1 1 2
P a
(iii) (iv)
b
c
P P d
(a) (iv) (b) (ii) e
(c) (iii) (d) (i) f
g
V
(a) ac (b) cg
(c) af (d) cd
KINETIC THEORY OF GASES & THERMODYNAMICS 80

40. Consider two containers A and B containing identical 45. In the cyclic process shown in the figure, the work
gases at the same pressure, volume, and temperature. done by the gas in one cycle is
The gas in container A is compressed to half of its
original volume isothermally while the gas in container
B is compressed to half of its original value
adiabatically. The ratio of final pressure of gas in B to
that of gas in A is:
γ −1
1
(a) 2γ–1 (b)  
2
γ −1 2
 1   1 
(c)   (d)  
1− γ   γ −1  (a) 28PV (b) 14PV
1 1 1 1
41. In the following indicator diagram, the net amount of
(c) 18PV
1 1 (d) 9PV
1 1
work done by the gas will be
46. Heat energy absorbed by a system in going through a
P
cyclic process shown in figure is

1 2

V
(a) Positive (b) Negative
(c) Zero (d) Infinity
42. One mole of an ideal gas requires 207 J heat to raise
(a) 107 π J (b) 104 π J
the temperature by 10K, when heated at constant
pressure. If the same gas is heated at constant volume (c) 102 π J (d) 10−3 π J
to raise the temperature by 10K, then heat required (in 47. In a cyclic process, work done by the system is
joules) is (R = 8.3 J/mol.K) (a) zero
Cyclic and Non-cyclic Processes (b) Equal to heat given to the system
(c) More than the heat given to system
43. An ideal gas undergoes cyclic process ABCDA as
(d) Independent of heat given to the system
shown in given P-V diagram
48. In a cyclic process, the internal energy of the gas
P
(a) Increases (b) Decreases
D C
2P0 (c) Remains constant (d) Becomes zero
49. A thermodynamic system is taken through the cycle
P0 B
A PQRSP process. The net work done (in joules) on the
V system is
V0 3V0
The amount of work done by the gas is:
(a) 6P0V0 (b) –2P0V0
(c) +2P0V0 (d) +4P0V0
44. Work done by the gas in the process shown in figure is

(a) positive (b) negative


(c) zero (d) cannot say
KINETIC THEORY OF GASES & THERMODYNAMICS 81

Thermodynamic Processes Heat Engine


50. A given system undergoes a change in which work 55. An ideal heat engine working between temperature T1
done by the system equals the decrease in its internal and T2 has an efficiency h, the new efficiency if both
energy. The system must have undergone
the source and sink temperature are doubled, will be
(a) isothermal change (b) adiabatic change
η
(c) isobaric change (d) isochoric change (a) (b) η
51. The temperature of a hypothetical gas increases to 2
2 times when compressed adiabatically to half the (c) 2η (d) 3η
volume. Its equation can be written as 56. The thermal efficiency of a hat engine for which the
(a) PV 3/ 2 = constant (b) PV 5/ 2 = constant work output is 2500 J/cycle and the heat input is
(c) PV 7/3 = constant (d) PV 4/3 = constant 10000 J/cycle, is
52. Two samples A and B of same gas have equal volumes (a) 75% (b) 40%
and pressures. The gas in sample A is expanded
isothermally to double its volume and the gas in (c) 25% (d) 0.25%
sample B is expanded to double its volume 57. An ideal gas heat engine operates in the Carnot cycle
adiabatically. If work done by the gas is same in two between 227°C and 127°C . It absorbs 6 × 104 cal of
processes, then value of γ for the gas is best given by heat at higher temperature. Amount of heat converted
(a) 1 − 2−γ =( γ − 1) ln 2 into work, is
(a) 1.2 × 104 cal (b) 2.4 × 104 cal
(γ 1) ln 2
(b) 1 − 2−(γ −1) =−
(c) 6.0 × 104 cal (d) 4.8 × 104 cal
(c) 1 − 2 − ( γ −1)
=2 ( γ − 1) ln 2
58. A thermodynamic system is taken through the cycle
(d) None of these ABCD as shown in figure. Heat rejected by the gas
53. Two samples A and B of a gas initially at the same during the cycle is:
pressure and temperature are compressed from volume
V
V to (A isothermally and B adiabatically). The
2
final pressure of A is
(a) Greater than the final pressure of B
(b) Equal to the final pressure of B
(c) Less than the final pressure of B (a) PV (b) 2 PV
(d) Twice the final pressure of B 1
(c) 4 PV (d) PV
54. An ideal gas undergoes four different processes from 2
the same initial state, as shown in figure. Four 59. The temperature-entropy diagram of a reversible
processes are adiabatic, isothermal, isobaric and engine cycle is given in the figure. Its efficiency is
isochoric. Out of 1, 2, 3 and 4 which one is adiabatic?
P

4
3
2
1

1 2
(a) (b)
3 3
1 1
(c) (d)
2 4
KINETIC THEORY OF GASES & THERMODYNAMICS 82

60. The shown P-V diagram represents the Carnot Cycle


thermodynamics cycle of an engine, operating with an 66. Even Carnot engine cannot give 100% efficiency
ideal monoatomic gas. The amount of heat, extracted because we cannot
from the source in a single cycle is:
(a) prevent radiation
(b) find ideal sources
(c) reach absolute zero temperature
(d) eliminate friction
67. Which statement is incorrect?
(a) all reversible cycles have same efficiency
(b) reversible cycle has more efficiency than the
irreversible one
(c) Carnot cycle is a reversible one
(d) Carnot cycle has the maximum efficiency in all
cycles
1
68. A Carnot engine, having an efficiency of η = as
 13  10
(a) PV
0 0 (b)   PV 0 0
2 heat engine, is used as a refrigerator. If the work done
on the system is 10 J, the amount of energy absorbed
 11 
(c)   PV 0 0 (d) 4PV
0 0
from the reservoir at lower temperature is
2 (a) 100 J (b) 99 J
61. A reversible engine converts one-sixth of the heat (c) 90 J (d) 1 J
input into work. When the temperature of the sink is 69. A Carnot engine takes 3 × 106 cal of heat from a
reduced by 62°C , the efficiency of the engine is reservoir at 627°C, and gives it to a sink at 27°C. The
doubled. The temperatures of the source and sink are work done by the engine is
(a) 80°C ,37°C (b) 95°C , 28°C (a) 4.2 × 106 J (b) 8.4 × l06 J
(c) 90°C ,37°C (d) 99°C ,37°C 6
(c) 16.8 × l0 J (d) zero
62. A heat engine operates between a cold reservoir at 70. Two Carnot engines A and B are operated in
temperature T2 = 300 K and a hot reservoir at succession. The first one, A receives heat form a
temperature T1 . It takes 200 J of heat from hot reservoir source at T = 800 K and rejects to sink at T2 . The
and delivers 120 J of heat to cold reservoir in a cycle. second engine B receives heat rejected by the first
What could be the minimum temperature of hot engine and rejects to another sink at T3 = 300 K . If the
reservoir?
work outputs of two engines are equal, then the value
(a) 200K (b) 300K
of T2 is
(c) 400K (d) 500K
(a) 100 K (b) 300 K
63. An ideal heat engine exhausting heat at 77°C is to
(c) 550 K (d) 700 K
have a 30% efficiency. It must take heat at
Refrigeration
(a) 127°C (b) 227°C
71. A refrigerator is to maintain eatables kept inside at
(c) 327°C (d) 673°C
9°C . If room temperature is 36°C , calculate the
64. A reversible heat engine coverts one-fourth of the heat coefficient of performance.
input into work. When the temperature of the sink is
(a) 10.9 (b) 20.9
reduced by 52K, its efficiency is doubled.
(c) 30.9 (d) 40.9
The temperature in Kelvin of the source will be ………
72. The coefficient of performance of a Carnot refrigerator
65. For an ideal heat engine, the temperature of the source
working between 30°C and 0°C is
is 127°C . In order to have 60% efficiency the
temperature of the sink should be ……… kelvin (a) 10 (b) 1
(Round off to the Nearest Integer). (c) 9 (d) 0
73. A refrigerator is to maintain eatables kept inside at
9°C , calculate the coefficient of performance?
(a) 10.44 (b) 11.44
(c) 12.44 (d) 13.44
KINETIC THEORY OF GASES & THERMODYNAMICS 83

74. The coefficient of performance for a refrigerator 78. The freezer in a refrigerator is located at the top section
should be: so that
(a) Greater than 1 (b) smaller than 1 (a) The entire chamber of the refrigerator is cooled
(c) Equal to 1 (d) None of these quickly due to convection
(b) The motor is not heated
75. A refrigerator is
(c) The heat gained from the environment is high
(a) Heat engine
(d) The heat gained from the environment is low
(b) An electric motor
79. The inside and outside temperature of a refrigerator are
(c) Heat engine working in backward direction
273 K and 303 K respectively. Assuming that
(d) Air cooler refrigerator cycle is reversible, for every joule of work
76. An ideal refrigerator has a freezer at a temperature of done the heat delivered to the surrounding will be
−13°C. The coefficient of performance of the engine (a) 10 J (b) 20 J
is 5. The temperature of the air (to which heat is (c) 30 J (d) 50 J
rejected) will be:
80. A refrigerator works between 4°C and 30°C . It is
(a) 325°C (b) 325K required to remove 600 cal of heat every second in
(c) 39°C (d) 320°C order to keep the temperature of the refrigerated space
77. The coefficient of performance of a Carnot refrigerator constant. The power required is (take, 1 cal = 4.2J)
working between 30°C and 0°C is (a) 23.65W (b) 236.5W
(a) 10 (b) 1 (c) 2365W (d) 2.365W
(c) 9 (d) 0
KINETIC THEORY OF GASES & THERMODYNAMICS 84

EXERCISE – 2: PREVIOUS YEARS JEE MAINS QUESTIONS


1. ‘n’ moles of an ideal gas undergo a process A → B as 5. Which of the following shows the correct relationship
shown in the figure. The maximum temperature of the between the pressure ‘P’ and density r of an ideal gas
gas during the process will be: [2016] at constant temperature? [2016]

3 P0V0 9 P0V0
(a) (b)
2nR 2nR
9 P0V0 9 P0V0
(c) (d)
nR 4nR
2. An ideal gas undergoes a quasi-static, reversible 6. C p and Cv are specific heats at constant pressure and
process in which its molar heat capacity C remains constant volume respectively. It is observed that
constant. If during this process the relation of pressure C p − Cv =a for hydrogen gas, C p − Cv = b for
P and volume V is given by PVn = constant, then n is
nitrogen gas. The correct relation between a and b is:
given by (Here C p and CV are molar specific heat at
[2017]
constant pressure and constant volume, respectively): 1
[2016] (a) a = b (b) a = b
14
C − CP C −C (c) a = 14 b (d) a = 28 b
(a) n = (b) n = P
C − CV C − CV 7. N moles of a diatomic gas in a cylinder are at a
C − CV C temperature T. Heat is supplied to the cylinder such
(c) n = (d) n = P
C − CP CV that the temperature remains constant but n moles of
the diatomic gas get converted into monoatomic gas.
3. The ratio of work done by an ideal monoatomic gas to
What is the change in the total kinetic energy of the
the heat supplied to it in an isobaric process is:
gas? [2017]
[2016]
1
3 2 (a) nRT (b) 0
(a) (b) 2
5 5
3 5
3 2 (c) nRT (d) nRT
(c) (d) 2 2
2 5
8. An engine operates by taking n moles of an ideal gas
4. A Carnot freezer takes heat from water at 0ºC inside it
through the cycle ABCDA shown in figure. The
and rejects it to the room at a temperature of 27ºC. The
thermal efficiency of the engine is:
latent heat of ice is 336 × 103J kg-1. If 5 kg of water at
(Take Cv =1.5 R, where R is gas constant) [2017]
0ºC is converted into ice at 0ºC by the freezer, then the
energy consumed by the freezer is close to: [2016]
5 6
(a) 1.67 × 10 J (b) 1.68 × 10 J
(c) 1.51 × 105J (d) 1.71 × 107J

(a) 0.24 (b) 0.15


(c) 0.32 (d) 0.08
KINETIC THEORY OF GASES & THERMODYNAMICS 85

9. An ideal gas has molecules with 5 degrees of freedom. 13. Two Carnot engines A and B are operated in series.
The ratio of specific heats at constant pressure (Cp) and Engine A receives heat from a reservoir at 600 K and
at constant volume (Cv) is: [2017] rejects heat to a reservoir at temperature T. Engine B
7 receives heat rejected by engine A and in turn rejects it
(a) 6 (b)
2 to a reservoir at 100 K. If the efficiencies of the two
5 7 engines A and B are represented by η A and η B
(c) (d)
2 5 ηB
respectively, then what is the value of ? [2018]
10. For the P-V diagram given for an ideal gas, out of the ηA
following which one correctly represents the T-P
diagram? [2017]
12 7
(a) (b)
7 12
12 5
(c) (d)
5 12
14. The value closest to the thermal velocity of a Helium
atom at room temperature (300K) in ms-1 is:
[kB = 1.4 × 10-23 J/K; mHe = 7 ×10-27 kg] [2016]
(a) 1.3 × 10 4
(b) 1.3 × 10 3

(c) 1.3 × 105 (d) 1.3 × 102


15. One mole of an ideal monoatomic gas is taken along
the path ABCA as shown in the PV diagram. The
maximum temperature attained by the gas along the
path BC is given by: [2018]

11. A Carnot’s engine works as a refrigerator between


250 K and 300 K. It receives 500 cal heat from the 25 PV 25 PV
0 0 0 0
reservoir at the lower temperature. The amount of (a) (b)
16 R 8 R
work done in each cycle to operate the refrigerator is:
[2018] 25 PV
0 0 5 PV
0 0
(c) (d)
(a) 420 J (b) 772 J 4 R 8 R
(c) 2100 J (d) 2520 J 16. Two moles of helium are mixed with n moles of
12. One mole of an ideal monoatomic gas is compressed C 3
hydrogen. If P = for the mixture, then the value of
isothermally in a rigid vessel to double its pressure at CV 2
room temperature, 27oC. The work done on the gas will n is: [2018]
be: [2018] (a) 1 (b) 3
(a) 300 R (b) 300 R ln 6 3
(c) 300 R ln 2 (d) 300 R ln 7 (c) 2 (d)
2
KINETIC THEORY OF GASES & THERMODYNAMICS 86

17. Following figure shows two processes A and B for a 21. A thermally isolated cylindrical closed vessel of height
gas. If ∆QA and ∆QB are the amounts of heat absorbed 8 m is kept vertically. It is divided into two equal parts
by a diathermic (perfect thermal conductor)
by the system in two cases, ∆U A and ∆U B are changes
frictionless partition of mass 8.3 kg. Thus, the partition
in internal energies, respectively, then: [2019] is held initially at a distance of 4 m from the top, as
shown in the schematic figure below. Each of the two
parts of the vessel contains 0.1 mole of an ideal gas at
temperature 300 K. The partition is now released and
moves without any gas leaking from one part of the
vessel to the other. When equilibrium is reached, the
distance of the partition from the top (in m) will be
(a) ∆Q A < ∆Q B , ∆U A < ∆U B _______ (take the acceleration due to gravity
(b) ∆Q A > ∆Q B , ∆U A > ∆U B =10 ms-2 and the universal gas constant). [2020]
(c) ∆Q A > ∆Q B , ∆U A = ∆U B
(d) ∆Q A =
∆Q B , ∆U A =
∆U B
18. Under an adiabatic process, the volume of an ideal gas
gets doubled. Consequently, the mean collision time
between the gas molecules changes from τ 1 to τ 2 .
Cp τ2
If = γ for this gas, then a good estimate for is 22. A spherical bubble inside water has radius R. Take the
CV τ1
pressure inside the bubble and the water pressure to be
given by [2020] P0. The bubble now gets compressed radially in an
γ +1
1 1 2 adiabatic manner so that its radius becomes (R – a). For
(a) (b)  
2 2 a << R the magnitude of the work done in the process
γ is given by (4π p0 Ra 2 ) X , where X is a constant and.
1
(c)   (d) 2
2 γ = Cp / Cv = 41/ 30 . The value of X is [2020]
19. A gas mixture consists of 3 moles of oxygen and 23. If one mole of the polyatomic gas is having two
5 moles of argon at temperature T. Assuming the gases vibrational modes and β is the ratio of molar specific
to be ideal and the oxygen bond to be rigid, the total
 C 
internal energy (in units of RT) of the mixture is: heats for polyatomic gas  β = p  , then the value of
[2020]  Cv 

(a) 11 (b) 13 β is: [2021]


(c) 15 (d) 20 (a) 1.02 (b) 1.20
20. Consider one mole of helium gas enclosed in a (c) 1.25 (d) 1.35
container at initial pressure P1 and V1. It expands 24. On the basis of kinetic theory of gases, the gas exerts
isothermally to volume 4V1. After this, the gas expands pressure because its molecules: [2021]
adiabatically and its volume becomes 32V1. The work (a) continuously lose their energy till it reaches wall
done by the gas during isothermal and adiabatic (b) are attracted by the walls of container
expansion processes are Wiso and Wadia, respectively. (c) continuously stick to the walls of container
W (d) suffer change in momentum when impinge on the
If the ration iso = f ln 2, then f is ……… [2020]
Wadia walls of container
KINETIC THEORY OF GASES & THERMODYNAMICS 87

25. Given below are two statements: 31. The root mean square speed of molecules of a given
Statement I: In a diatomic molecule, the rotational mass of a gas at 27° C and 1 atmosphere pressure is
energy at a given temperature obeys Maxwell's 200 ms-1. The root mean square speed of molecules of
distribution. the gas at 127° C and 2 atmosphere pressure is
Statement II: In a diatomic molecule, the rotational x
energy at a given temperature equals the translational ms-1. The value of x will be [2021]
3
kinetic energy for each molecule.
32. A monatomic gas of mass 4.0 u is kept in an insulated
In the light of the above statements, choose the correct
container. Container is moving with velocity 30 m/s. If
answer from the options given below: [2021]
container is suddenly stopped then change in
(a) Both Statement I and Statement II are true.
x
(b) Statement I is false but Statement II is true. temperature of the gas (R= gas constant) is .
3R
(c) Both Statement I and Statement II are false.
(d) Statement I is true but Statement II is false. Value of x is ……… [2021]
26. The internal energy (U), pressure (P) and volume (V) 33. Match List-I with List-II
of an ideal gas are related as U = 3PV + 4. The gas is List-I List-II
[2021] A Isothermal (i) Pressure constant
(a) Diatomic only B Isochoric (ii) Temperature constant
(b) Polyatomic only C Adiabatic (iii) Volume constant
(c) Either monoatomic or diatomic D Isobaric (iv) Heat content is constant
(d) Monoatomic only Choose the correct answer from the options given
27. Calculate the value of mean free path (λ) for oxygen below [2021]
molecules at temperature 27°C and pressure 1.01 × 105 (a) A – (iii), B – (ii), C – (i), D – (iv)
Pa. Assume the molecular diameter 0.3 nm and the gas (b) A – (ii), B – (iv), C – (iii), D – (i)
is ideal. (k = 1.38 × 10-23 JK-1) [2021] (c) A – (i), B – (iii), C – (ii), D – (iv)
(a) 86 nm (b) 102 nm (d) A – (ii), B – (iii), C – (iv), D – (i)
(c) 32 nm (d) 58 nm 34. n moles of a perfect gas undergoes a cyclic process
28. A polyatomic ideal gas has 24 vibrational modes. What ABCA (see figure) consisting of the following
is the value of γ ? [2021] processes.
(a) 1.03 (b) 1.30 A → B: Isothermal expansion at temperature T so that
(c) 1.37 (d) 10.3 the volume is doubled from V1 to V2 = 2V1 and
pressure changes from P1 to P2.
29. Two ideal polyatomic gases at temperatures T1 and T2
B → C: Isobaric compression at pressure P2 to initial
are mixed so that there is no loss of energy. If F1 and volume V1.
F2 , m1 and m2 , n1 and n2 be the degrees of freedom, C → A: Isochoric change leading to change of pressure
masses, number of molecules of the first and second from P2 to P1.
gas respectively, the temperature of mixture of these Total work done in the complete cycle ABCA is:
two gases is: [2021] [2021]
nT +n T n FT + n F T
(a) 1 1 2 2 (b) 1 1 1 2 2 2
n1 + n2 n1 F1 + n2 F2
n1 F1T1 + n2 F2T2 n1 F1T1 + n2 F2T2
(c) (d)
F1 + F2 n1 + n2
30. What will be the average value of energy along one
degree of freedom for an ideal gas in thermal
equilibrium at a temperature T? ( k B is Boltzmann
 1
constant) [2021] (a) 0 (b) nRT  ln 2 − 
2  2
(a) k BT (b) k BT
3  1
(c) nRT ln 2 (d) nRT  ln 2 + 
1 3  2
(c) k BT (d) k BT
2 2
KINETIC THEORY OF GASES & THERMODYNAMICS 88

35. If one mole of an ideal gas at (P1, V1) is allowed to 39. A Carnot’s engine working between 400 K and 800 K
expand reversibly and isothermally (A to B) its has a work output of 1200 J per cycle. The amount of
pressure is reduced to one-half of the original pressure heat energy supplied to the engine from the source in
(see figure). This is followed by a constant volume each cycle is: [2021]
cooling till its pressure is reduced to one-fourth of the (a) 3200 J (b) 1800 J
initial value (B → C). Then it is restored to Initial state (c) 1600 J (d) 2400 J
by a reversible adiabatic compression (C to A). The net 40. Which one is the correct option for the two different
work done by the gas is equal to [2021] thermodynamic processes? [2021]

 1  RT
(a) RT  ln 2 −  (b) −
 2 ( γ − 1)  2 ( γ − 1)
(c) RT ln 2 (d) 0
7 5 (a) (c) and (a) (b) (c) and (d)
36. =
A diatomic gas, having CP = R and CV R, is
2 2 (c) (a) only (d) (b) and (c)
heated at constant pressure. The ratio dU : dQ : dW : 41. The P-V diagram of a diatomic ideal gas system going
[2021] under cyclic process is shown in figure. The work done
(a) 5 : 7 : 2 (b) 3 : 5 : 2 during an adiabatic process CD is (use γ = 1.4) :
(c) 5 : 7 : 3 (d) 3 : 7 : 2 [2021]
37. Thermodynamic process is shown below on a P-V
diagram for one mole of an ideal gas. If V2 = 2V1 then
T
the ratio of temperature 2 is: [2021]
T1

(a) – 400 J (b) 400 J


1 (c) 200 J (d) – 500 J
(a) (b) 2
2 42. Consider a sample of oxygen behaving like an ideal
1 gas. At 300 K, the ratio of root mean square (rms)
(c) (d) 2
2 velocity to the average velocity of gas molecule would
38. The volume V of an enclosure contains a mixture of be:
three gases, 16 g of oxygen, 28 g of nitrogen and 44 g (Molecular weight of oxygen is
of carbon dioxide at absolute temperature T. consider 32 g/mol; R = 8.3 J K-1 mol-1) [2021]
R as universal gas constant. The pressure of the
3 3π
mixture of gases is: [2021] (a) (b)
8 8
88RT 5 RT
(a) (b) 8 8π
V 2 V (c) (d)
3 3
3RT 4RT
(c) (d)
V V
KINETIC THEORY OF GASES & THERMODYNAMICS 89

43. For an adiabatic expansion of an ideal gas, the 47. A container is divided into two chambers by a
fractional change in its pressure is equal to (where γ partition. The volume of first chamber is 4.5 litre and
is the ratio of specific heats): [2021] second chamber is 5.5 litre. The first chamber contain
1 dV V 3.0 moles of gas at pressure 2.0 atm and second
(a) − (b) − γ chamber contain 4.0 moles of gas at pressure 3.0 atm.
γ V dV
After the partition is removed and the mixture attains
dV dV
(c) − γ (d) equilibrium, then, the common equilibrium pressure
V V existing in the mixture is x × 10−1 arm.
44. An ideal gas in a cylinder is separated by a piston in Value of x is ……… [2021]
such a way that the entropy of one part is S1 and that 48. The volume V of a given mass of monoatomic gas
of the other part is S2. Given that S1 > S2. If the piston changes with temperature T according to the relation
is removed then the total entropy of the system will be: 2

[2021] V = KT 3 . The work done when temperature changes


(a) S1 + S 2 (b) S1 − S 2 by 90 K will be xR. The value of x is: (Assume one
mole of gas only)
S1
(c) S1 × S 2 (d) [R = universal gas constant] [2021]
S2
Q
45. In a certain thermodynamical process, the pressure of 49. 1 mole of rigid diatomic gas performs a work of
5
a gas depends on its volume as kV3. The work done
when heat Q is supplied to it. The molar heat capacity
when the temperature changes from 100°C to 300°C
will be ……… nR, where n denotes number of moles xR
of the gas during this transformation is . The value
of a gas. [2021] 8
46. A reversible heat engine converts one-fourth of the of x is:
heat input into work. When the temperature of the sink [R = universal gas constant] [2021]
is reduced by 52 K, its efficiency is doubled. The 50. For an ideal heat engine, the temperature of the source
temperature in Kelvin of the source will be ……… is 127°C. In order to have 60% efficiency the
[2021] temperature of the sink should be ……… °C. (Round
off to the Nearest Integer) [2021]
KINETIC THEORY OF GASES & THERMODYNAMICS 90

EXERCISE – 3: ADVANCED OBJECTIVE QUESTIONS


Objective Questions I [Only one correct option] 7. A gas molecule of mass M at the surface of the Earth
has kinetic energy equivalent to 0°C. If it were to go
1. Which one of the following gases possesses the largest up straight without colliding with any other molecules,
internal energy how high it would rise? Assume that the height
(a) 2 moles of helium occupying 1 m3 at 300 K
attained is much less than radius of the earth. (kB is
(b) 56 kg of nitrogen at 107 Nm-2 and 300 K
Boltzmann constant)
(c) 8 grams of oxygen at 8 atm and 300 K
273k B 546 k B
(d) 6 × 1026 molecules of argon occupying 40 m3 at (a) (b)
2 Mg 3Mg
900 K
819 k B
2. What will be the temperature when the rms velocity is (c) 0 (d)
2 Mg
double of that at 300 K:
8. A gaseous mixture consists of 16 g of helium and 16 g
(a) 300 K (b) 600 K
of oxygen, The ratio CP/CV of the mixture is
(c) 900 K (d) 1200 K
3. If Maxwell distribution is valid and VP denotes the (a) 1.4 (b) 1.54
(c) 1.59 (d) 1.62
most probable speed, V the average speed and Vrms the 
root–mean–square speed, then: 9. Let v, v rms and vP respectively denote the mean speed,
(a) V < VP < Vrms (b) V < Vrms < VP root mean square speed and most probable speed of the
(c) VP < V < Vrms (d) VP < Vrms < V molecules in an ideal monoatomic gas at absolute
temperature T. The mass of the molecule is m. Then
4. The root–mean–square (rms) speed of oxygen
(a) no molecule can have a speed greater than
molecules (O2) at a certain absolute temperature is V.
( 2vrms )
If the temperature is doubled and the oxygen gas
dissociates into atomic oxygen, the rms speed would vp
(b) no molecule can have a speed less than
be: ( 2)
(a) V (b) 2V (c) v < v p < vrms
(c) 2V (d) 2 2V (d) the average kinetic energy of the molecules is
5. The temperature at which the root mean square speed 3
4
( mv 2p )
of a gas will be half its value at 0°C is (assume the
pressure remains constant): 10. The temperature-entropy diagram of a reversible
(a) – 86.4°C (b) – 204.75°C engine cycle is given in the figure. Its efficiency is
T
(c) – 104.75°C (d) – 68.25°C
6. Tyre of a bicycle has volume 2 × 10-3 m3. Initially the
tube is filled to 75% of its volume by air at atmospheric 2T0
pressure of p0 = 105 N/m2. When a rider rides the
bicycle the area of contact of tyre with road is T0
A = 24 × 10-4 m2. The mass of rider with bicycle is
120 kg. The number of strokes which delivers,
S
V = 500 cm3 volume of air in each stroke required to S0 2S0
inflate the tyres is [Take g = 10 m/s2]
(a) 1/3 (b) 2/3
(a) 10 (b) 11
(c) 1/2 (d) 1/4
(c) 20 (d) 21
11. When an ideal diatomic gas is heated at constant
pressure, the fraction of the heat energy supplied which
increases the internal energy of the gas, is
2 3
(a) (b)
5 5
3 5
(c) (d)
7 7
KINETIC THEORY OF GASES & THERMODYNAMICS 91

12. A monoatomic ideal gas, initially at temperature T1, is 17. The temperature of a hypothetical gas increases to 2
enclosed in a cylinder fitted with a frictionless piston. times when compressed adiabatically to half the
The gas is allowed to expand adiabatically to a volume. Its equation can be written as
temperature. T2 by releasing the piston suddenly. If L1 (a) PV3/2 = constant (b) PV5/2 = constant
and L2 are the lengths of the gas column before and 7/3
(c) PV = constant (d) PV4/3 = constant
after expansion respectively, then T1/T2 is given by 18. When an ideal gas (γ = 5/3) is heated under constant
2/3
L  L pressure, then what percentage of given heat energy
(a)  1  (b) 1
 L2  L2 will be utilised in doing external work
2/3
(a) 40 % (b) 30 %
L L  (c) 60 % (d) 20 %
(c) 2 (d)  2 
L1  L1  19. The shown P–V diagram represents the
13. A mono atomic gas is supplied the heat Q very slowly thermodynamic cycle of an engine, operating with an
keeping the pressure constant. The work done by the ideal monoatomic gas. The amount of heat, extracted
gas will be from the source in a single cycle is:
2 3
(a) Q (b) Q
3 5
2 1
(c) Q (d) Q
5 5
14. An ideal gas expands isothermally from a volume V1
to V2 and then compressed to original volume V1
adiabatically. Initial pressure is P1 and final pressure is
P3. The total work done is W. Then
(a) P3 > P1, W > 0 (b) P3 < P1, W < 0  13 
(a) P0V0 (b)   PV 0 0
(c) P3 > P1, W < 0 (d) P3 = P1, W = 0 2
15. A cylindrical tube of uniform cross-sectional area A is
 11 
fitted with two air tight frictionless pistons. The pistons (c)   PV 0 0 (d) 4P0V0
2
are connected to each other by a metallic wire. Initially
the pressure of the gas is P0 and temperature is T0, 20. Two samples A and B of a gas initially at the same
pressure and temperature are compressed from volume
atmospheric pressure is also P0. Now the temperature
V to V/2 (A isothermally and adiabatically). The final
of the gas is increased to 2T0, the tension in the wire pressure of A is
will be (a) Greater than the final pressure of B
(b) Equal to the final pressure of B
Wire (c) Less than the final pressure of B
(d) Twice the final pressure of B
21. A thermodynamic system is taken through the cycle
(a) 2 P0A (b) P0A PQRSP process. The net work done by the system is
P0 A
(c) (d) 4 P0A
2
16. The molar heat capacity in a process of a diatomic gas
Q
if it does a work of when a heat of Q is supplied to
4
it is
2 5
(a) R (b) R
5 2
(a) 20 J (b) – 20 J
10 6 (c) 400 J (d) – 374 J
(c) R (d) R
3 7
KINETIC THEORY OF GASES & THERMODYNAMICS 92

22. Two rigid boxes containing different ideal gases are 25. An ideal gas is taken through the cycle A → B → C →
placed on a table. Box A contains one mole of nitrogen A, as shown in the figure. If the net heat supplied to the
at temperature T0, while Box B contains one mole of gas in the cycle is 5 J, the work done by the gas in the
helium at temperature (7/3) T0. The boxes are then put process C → A is
into thermal contact with each other and heat flows
between them until the gases reach a common final
temperature. (Ignore the heat capacity of boxes). Then,
the final temperature of the gases, Tf, in terms of T0 is
5 3
(a) T f = T0 (b) T f = T0
2 7
7 3 (a) – 5 J (b) – 10 J
(c) T f = T0 (d) T f = T0
3 2 (c) – 15 J (d) – 20 J
23. An ideal gas of mass m in a state A goes to another state 26. In the cyclic process shown in the figure, the work
B via three different processes as shown in figure. If done by the gas in one cycle is
Q1, Q2 and Q3 denote the heat absorbed by the gas
along the three paths, then
P A

3
2
1

B
V (a) 28 P1V1 (b) 14 P1V
(a) Q1 < Q2 < Q3 (b) Q1 < Q2 = Q3 (c) 18 P1V (d) 9 P1V1
(c) Q1 = Q2 > Q3 (d) Q1 > Q2 > Q3 27. Heat energy absorbed by a system in going through a
cyclic process shown in figure is
24. A thermodynamic process is shown in the figure. The
pressures and volumes corresponding to some points
in the figure are:
PA = 3 × 104 Pa, PB = 8 × 104 Pa
and VA = 2 × 10–3m3, VD = 5 × 10–3 m3
In process AB, 600 J of heat is added to the system and
in process BC, 200 J of heat is added to the system.
The change in internal energy of the system in process (a) 107 π J (b) 104 π J
AC would be (c) 102 π J (d) 10-3 π J

(a) 560 J (b) 800 J


(c) 600 J (d) 640 J
KINETIC THEORY OF GASES & THERMODYNAMICS 93

28. One mole of diatomic ideal gas undergoes a cyclic 32. Two Carnot engines A and B are operated in
process ABC as shown in figure. The process BC is succession. The first one, A receives heat from a source
adiabatic. The temperatures at A, B and C are 400 K, at T = 800 K and rejects to sink at T2. The second
800 K and 600 K respectively. Choose the correct engine B receives heat rejected by the first engine and
statement: rejects to another sink at T3 = 300 K. If the work
outputs of two engines are equal, then the value of T2
is
(a) 100 K (b) 300 K
(c) 550 K (d) 700 K
33. A reversible engine converts one-sixth of the heat input
into work. When the temperature of the sink is reduced
by 62°C, the efficiency of the engine is doubled. The
(a) The change in internal energy in the process CA is temperatures of the source and sink are
700 R. (a) 80°C, 37°C (b) 95°C, 28°C
(b) The changes in internal energy in the process AB is (c) 90°C, 37°C (d) 99°C, 37°C
–350 R. 34. An ideal monoatomic gas is confined in a cylinder by
(c) The change in internal energy in the process BC is a spring-loaded piston of cross section 8.0 × 10–3m2.
–500 R. Initially the gas is at 300 K and occupies a volume of
(d) The change in internal energy in whole cyclic 2.4× 10–3 m3 and the spring is in its relaxed state as
process is 250 R. shown in figure. The gas is heated by a small heater
29. A thermodynamic system undergoes cyclic process until the piston moves out slowly by 0.1 m. The force
ABCDA as shown in figure. The work done by the constant of the spring is 8000 N/m and the atmospheric
system is pressure is 1.0 × 105 N/m2. The cylinder and the piston
P
C B are thermally insulated. The piston and the spring are
3P0
massless and there is no friction between the piston and
2P0 O the cylinder. The final temperature of the gas will be:
(Neglect the heat loss through the lead wires of the
P0 heater. The heat capacity of the heater coil is also
A D
negligible)
V
V0 2V0

(a) P0V0 (b) 2P0V0


PV
(c) 0 0 (d) Zero
2
30. Two samples A and B of same gas have equal volumes (a) 500 K (b) 300 K
and pressures. The gas in sample A is expanded (c) 800 K (d) 1000 K
isothermally to double its volume and the gas in sample 35. When a system is taken from state i to state f along the
path iaf, it is found that Q = 50 cal and W = 20 cal.
B is expanded to double its volume adiabatically. If
Along the path ibf, Q = 36 cal. W along the path ibf is
work done by the gas is same in two processes, then
value of γ for the gas is best given by
(a) 1 – 2–γ = (γ – 1) ln 2
(b) 1 – 2–(γ – 1) = (γ – 1) ln 2
(c) 1 – 2–(γ – 1) = 2 (γ – 1) ln 2
(d) None of these
31. If CV for an ideal gas is given by CV = 3 + 2T, where
T is the absolute temperature of gas, then the equation (a) 14 cal (b) 6 cal
of adiabatic process for this gas is (c) 16 cal (d) 66 cal
(a) VT2 = constant (b) VT2e2T = constant
3 –2T
(c) VT e = constant (d) VT3e2T = constant
KINETIC THEORY OF GASES & THERMODYNAMICS 94

36. A Carnot engine absorbs 1000 J of heat energy from a 42. Four curves A, B, C and D are drawn in the adjoining
reservoir at 127oC and rejects 600 J of heat energy figure for a given amount of gas. The curves which
during each cycle. The efficiency of engine and represent adiabatic and isothermal changes are
temperature of sink will be:
(a) 50% and – 20oC (b) 40% and – 33oC
o
(c) 70% and – 10 C (d) 20% and – 43oC
37. Equal masses of three liquids A, B and C have
temperatures 10°C, 25°C and 40°C respectively. If A
and B are mixed, the mixture has a temperature of
15°C. If B and C are mixed, the mixture has a (a) C and D respectively (b) D and C respectively
temperature of 30°C. If A and C are mixed, the mixture (c) A and B respectively (d) B and A respectively
will have a temperature of:
Objective Questions II
(a) 16°C (b) 20°C
[One or more than one correct option]
(c) 25°C (d) 29°C
38. The temperatures of equal masses of three different 43. During an experiment, an ideal gas is found to obey a
liquids A, B and C are 12°C, 19°C and 28°C P2
condition = constant [ ρ = density of the gas]. The
respectively. The temperature when A and B are ρ
mixed is 16°C, and when B and C are mixed, it is gas is initially at temperature T, pressure P and density
23°C. What will be the temperature when A and C are ρ . The gas expands such that density changes to ρ /2
mixed ?
(a) 15.6 °C (b) 23.2 °C (a) The pressure of the gas changes to 2P
(c) 20.3 °C (d) 25.8 °C (b) The temperature of the gas changes to 2T
39. An ideal gas at 27°C is compressed adiabatically to (c) The graph of the above process on the P-T diagram
8 is parabola
of its original volume. The rise in temperature is
27 (d) The graph of the above process on the P-T diagram
 5 is hyperbola
γ = 
 3 44. Pick the correct statement (s)
(a) 475° C (b) 402° C (a) The rms translational speed for all ideal-gas
(c) 275° C (d) 375° C molecules at the same temperature is not the same
40. A mass of diatomic gas (γ = 1.4) at a pressure of but it depends on the mass
2 atmospheres is compressed adiabatically so that its (b) Each particle in gas has average translational
temperature rises from 27°C to 927°C. The pressure of 1 2 3
the gas in the final state is: kinetic energy and the equation mvrms = kT
2 2
(a) 256 atm (b) 8 atm establishes the relationship between the average
(c) 28 atm (d) 68.7 atm translational kinetic energy per particle and
41. In the following P-V diagram two adiabatics cut two temperature of an ideal gas. It can be concluded that
isothermals at temperatures T1 and T2 (fig.). The value single particle has a temperature.
V (c) Temperature of an ideal gas is doubled from 100°C
of a will be
Vd to 200°C. The average kinetic energy of each
particle is also doubled.
(d) It is possible for both the pressure and volume of a
monoatomic ideal gas to change simultaneously
without causing the internal energy of the gas to
change.

Vb Vc
(a) (b)
Vc Vb
Vd
(c) V (d) VbVc
a
KINETIC THEORY OF GASES & THERMODYNAMICS 95

45. For the P–V diagram shown, for path 1 → 2 → 3, 100 48. A monoatomic ideal gas having an initial volume of
J of heat is given to the system and 40 J of work is done 0.1 m3 and a pressure of 106 N/m2 absorbs 6 × 104 J of
by the system. For path 1 → 4 → 3, the work done by heat. For this situation mark out the correct
system is 10 J. Then statement(s).
(a) The change in internal energy if process is isobaric
is 3.6 × 104 J.
(b) The change in internal energy if process is isobaric
is 6 × 104 J.
(c) The change in internal energy if process is isochoric
is 6 × 104 J.
(d) The change in internal energy if process is
(a) U3 – U1 = 60 J isothermal is 6 × 104 J.
49. In the figure shown, a horizontal cylindrical tube
(b) For 3 → 1 the heat is rejected and equal to 85 J
connected to the vertical wall of a chamber of volume
(c) For 1 → 4 the work done by system is 10 J
V, contains V0 volume of air at atmospheric pressure
(d) For 1 → 4 the heat given to system is 20 J
46. In a certain process, final volume of the gas is equal to P0 and temperature T0 by means of a movable piston
its initial volume. Then for this process A and a valve B. In the beginning valve is closed and
(a) work done by the gas may be equal to zero the chamber is evacuated. The valve gets opened and
(b) work done by the gas is zero air starts leaking into the chamber very slowly. The
(c) change in internal energy of the gas may be equal volume V >> V0 . Take the molar heat capacity of air at
to zero
(d) ratio of final to initial pressure is equal to ratio of 5
constant volume equals R, where R is universal gas
final to initial temperature 2
constant. When air occupies the total volume of the
47. 1 mole of an ideal monoatomic gas is confined in a
chamber, (All walls and piston are made up of
cylinder fitted with a piston as shown in the figure.
insulating material):
Take mass of the piston as 0.5 kg, spring constant,
k = 100 N/m, atmospheric pressure p0 = 105 N/m2, and
the cross–section area of piston, A = 50 cm2. If energy
of 50 J has been supplied to the system, then piston
moves up by 5 cm [Take g = 10 m/s2]. Initially the
spring is in its natural position. Mark out the correct
statement(s).

(a) Its internal energy is 7PV


0 0

(b) Its temperature is 1.4T0


1.4PV0 0
(c) Its pressure is
V
(d) None

(a) Increase in internal energy of the system is 24.625J.


(b) Initial pressure of the gas is 1.01 × 105 N/m2.
(c) For the change described, the temperature of the
gas increases by 2 K.
(d) For the change described, the temperature of the
gas decreases by 2 K.
KINETIC THEORY OF GASES & THERMODYNAMICS 96

50. A monoatomic gas undergoes a cyclic process as 55. Determine efficiency of Carnot cycle if in adiabatic
shown in the figure. The ideal gas follows the equation expansion volume 3 times of initial value and (Given;
T = bV2 for the process B → C. During the cycle; the 1
ratio of maximum to minimum pressure is 2 and heat
= γ 1.5,= 0.58 )
3
supplied to the gas in B → C is QB → C = 120 J.
56. For a refrigerator, heat absorbed from source is 800 J
Assuming usual meanings for the symbols, pick
and heat supplied to sink is 600 J then find coefficient
correct option(s)
of performance is:
57. A Carnot engine works between 27ºC and 127ºC. Heat
supplied by the source is 500 J. Then heat ejected (in
joules) to the sink is:
58. In an isobaric process, the work done by a di-atomic
gas is 10J, the heat given (in joules) to the gas will be:
59. A sample of 0.1 g of water at 1000C and normal
pressure (1.0 × 105 Nm -2 ) requires 54 cal of heat
TA
(a) PA = PC (b) =2 energy to convert to steam at 1000C. If the volume of
TB the steam produced is 168 cc, the change in internal
TC energy (in joules) of the sample, is: (Given; 1cal = 4.2
(c) =2 (d) QC → A → B = –130J
TB joules)
51. The internal energy U of an ideal gas depends on 60. One mole of diatomic ideal gas undergoes a cyclic
pressure P and Volume V of the gas according to process ABC as shown in figure. The process BC is
equation U = 3PV, which of the following conclusion adiabatic. The temperatures at A, B and C are 400 K,
can you make regarding the gas? 800K and 600 K respectively. The magnitude of
(a) The gas is not a monatomic gas change in internal energy in the process BC is nR
(b) The gas can be a diatomic gas joules Where R is gas constant. Find the value of n.
(c) The gas can be a triatomic gas
(d) Molar specific heat of the gas in an isobaric
process is 4R
Numerical Value Type Questions
52. Two cylinders A and B fitted with pistons contain
equal amounts of an ideal diatomic gas at 300 K. The
piston of A is free to move, while that of B held fixed.
The same amount of heat is given to the gas in each Assertion & Reason
cylinder. Determine the greatest integral value of the For the following question choose the correct answer
ratio of change in temperature of B to the change in from the codes (A), (B), (C) and (D) defined as follows.
temperature of A. (A) Statement-I is true, Statement-II is also true;
53. On mole of an ideal monoatomic gas is taken through Statement-II is a correct explanation of Statement- I.
(B) Statement-I is true, Statement-II is also true;
a thermodynamic process shown in the p–V diagram.
Statement-II is not a correct explanation of
The heat supplied to the system in this process is Statement-I.
k × (π + 10) p0V0 . Determine the value of k. (C) Statement-I is true, Statement-II is false.
(D) Statement-I is false, Statement-II is true.
61. Statement-1: The internal energy of a given sample of
an ideal gas depends only its temperature according to
kinetic theory of gases.
Statement-2: The ideal gas molecules do not exert
inter-molecular forces.
54. If sink and source temperature of a refrigerator are 7ºC (a) A (b) B
and 847ºC respectively. Then efficiency of the (c) C (d) D
refrigerator is:
KINETIC THEORY OF GASES & THERMODYNAMICS 97

62. Statement-1: Internal energy change is zero if the Match the Columns
temperature is constant, irrespective of the process 68.Match the quantities given in column-I with column-II
being cyclic or non-cyclic.
Statement-2: dU = n CvdT for all process and is Column-I Column-II
(A) Adiabatic bulk modulus (p) – P/V
independent of path.
(B) Slope of P-V graph in (q) 2/(γ–1)
(a) A (b) B
isothermal process
(c) C (d) D
(C) Degree of freedom (r) γP
63. Statement-1: A quasi-static process is so called
because it is a sudden and large change of the system. (D) The ratio of molar heat (s) γ
Statement-2: An adiabatic process is not quasi-static capacity at constant γ −1
because it is a sudden and large change of the system. pressure to universal gas
(a) A (b) B constant R
(c) C (d) D 69. Thermodynamic processes are indicated in the
following diagram.
64. Statement-1: When an ideal gas is taken from a given
thermodynamics state A to another given
thermodynamic state B by any polytropic process, the
change in the internal energy of the system will be the
same in all processes.
Statement-2: Internal energy of the gas depends only
upon its absolute temperature.
(a) A (b) B
(c) C (d) D Match the following:
65. Statement-1: Work done by a gas in isothermal Column-I Column -II
expansion is more than the work done by the gas in the
P Process I a. Adiabatic
same expansion adiabatically.
Statement-2: Temperature remains constant in Q Process II b. Isobaric
isothermal expansion but not in adiabatic expansion. R. Process III c. Isochoric
(a) A (b) B S. Process IV d. Isothermal
(c) C (d) D 70. One mole of a monatomic ideal gas is taken along two
66. Statement-1: In an isothermal process whole of the cyclic processes E → F → G → E and E → F →
heat energy supplied to the body is converted into H → E as shown in the P–V diagram. The process
internal energy. involved are purely isochoric, isobaric, isothermal or
Statement-2: According to the first law of adiabatic
thermodynamics ∆Q = ∆U + P∆V
(a) A (b) B
(c) C (d) D
67. Statement-1: In an adiabatic process, change in
internal energy of a gas is equal to work done on or by
the gas in the process.
Statement-2: Temperature of gas remains constant in Match the paths in List I with the magnitudes of the
a adiabatic process. work done in List II and select the correct answer using
the codes given below the lists.
(a) A (b) B
(c) C (d) D List I List II

P. G → E 1. 160 PV
0 0 ln 2

Q. G → H 2. 36PV
0 0

R. F → H 3. 24PV
0 0

S. F → G 4. 31PV
0 0
KINETIC THEORY OF GASES & THERMODYNAMICS 98

Paragraph Type Questions 72. Identify the gas filled in the container A and B

Use the following passage, solve Q. 71 to Q. 75 (a) N2, Ne (b) He, H2


Passage
(c) O2, Ar (d) Ar, O2
Two closed identical conducting containers are found in the
73. Total number of molecules in 'A' (Here NA = Avogadro
laboratory of an old scientist. For the verification of the gas
number)
some experiments are performed on the two boxes and the
results are noted. 125
(a) N (b) 3.125 NA
64 A
125
(c) N (d) 31.25 NA
28 A
74. The initial internal energy of the gas in container 'A', if
the containers were at room temperature 300 K
initially
Experiment 1: When the two containers are weighed
WA = 225 g, WB = 160 g and mass of evacuated container (a) 1406.25 cal (b) 1000 cal
WC=100 g. (c) 2812.5 cal (d) none of these
Experiment 2: When the two containers are given same 75. If the gases have initial temperature 300 K and they are
amount of heat same temperature rise is recorded. The mixed in an adiabatic container having the same
pressure change found are ∆PA = 2.5 atm, ∆PB = 1.5 atm volume as the previous containers. Now the
temperature of the mixture is T and pressure is P. Then
Required data for unknown gas:
Mono (molar mass) (a) P > PA, T > 300K (b) P > PB, T = 300K
He Ne Ar Kr Xe Rd (c) P < PA, T = 300K (d) P > PA, T < 300K
4g 20 g 40 g 84 g 131 g 222 g
Dia (molar mass)
H2 F2 N2 O2 Cl2
2g 19 g 28 g 32 g 71 g
71. Identify the type of gas filled in container A and B
respectively
(a) Mono, mono (b) Dia, dia
(c) Mono, dia (d) Dia, mono
KINETIC THEORY OF GASES & THERMODYNAMICS 99

EXERCISE – 4: PREVIOUS YEARS JEE ADVANCED QUESTIONS


1. Two gases, Argon (atomic radius 0.07 nm, atomic 4. The PT diagram for an ideal gas is shown in the figure,
weight 40) and Xenon (atomic radius 0.1 nm, atomic where AC is an adiabatic process. Find the
weight 140) have the same number density and are at corresponding PV diagram. (2003)
the same temperature. The ratio of their respective
mean free time is closest to (2001)
(a) 4.67 (b) 2.04
(c) 1.83 (d) 3.67
2. P-V plots for two gases during adiabatic processes are
shown in the figure. Plots 1 and 2 should correspond
respectively to (2001)
(a) (b)

(c) (d) None of these

(a) He and O2 (b) O2 and He


5. If liquefied oxygen at 1 atmospheric pressure is heated
(c) He and Ar (d) O2 and N2
from 50 K to 300 K by supplying heat at constant rate,
3. An ideal gas is taken through the cycle the graph of temperature vs time will be (2004)
A → B → C → A, as shown in the figure, if the net
heat supplied to the gas in the cycle is 5 J, the work
done by the gas in the process C → A is (2002)
(a) (b)

(c) (d)

(a) - 5 J (b) - 10 J
(c) - 15 J (d) - 20 J 6. An ideal gas expands isothermally from a volume V1
to V2 and then compressed to original volume V1
adiabatically. Initial pressure is P1 and final pressure is
P3. The total work done is W. Then (2004)
(a) P3 > P1, W > 0 (b) P3 < P1, W < 0
(c) P3 > P1, W < 0 (d) P3 = P1, W = 0
KINETIC THEORY OF GASES & THERMODYNAMICS 100

7. Statement-1: The total translational kinetic energy of 10. The buoyancy force acting on the gas bubble is
all the molecules of a given mass of an ideal gas is 1.5 (Assume R is the universal gas constant) (2008)
times the product of its pressure and its volume.
( p0 + ρ gH ) 2/5
Statement-2: The molecules of a gas collide with each (a) ρ nRg T0
( p0 + ρ gy ) 2/5
other and the velocities of the molecules change due to
the collision. (2007) ρ nRgT0
(b)
(a) Statements-1 and 2 are true and statement-2 is a ( p0 + ρ gH ) [ p0 + ρ  g ( H − y )]3/5
2/5

correct explanation for statement-1.


( p0 + ρ  gH )3/5
(b) Statements-1 and 2 are true and statement-2 is not (c) ρ  nRgT0
a correct explanation for statement-1. ( p0 + ρ  gy )8/5
(c) Statement-1 is true, statement-2 is false. ρ nRgT0
(d) Statement-1 is false, statement-2 is true. (d)
( p0 + ρ gH )3/5 [ p0 + ρ g ( H − y )]2/5
Use the following passage, solve Q. 8 to Q. 10
11. CV and CP denote the molar specific heat capacities of
 5
A small spherical monoatomic ideal gas bubble  γ =  is a gas at constant volume and constant pressure,
 3
respectively.
trapped inside a liquid of density ρl (see figure). Assume that Then, (2009)
the bubble does not exchange any heat with the liquid. The (a) CP – CV is larger for a diatomic ideal gas than for a
bubble contains n moles of gas. The temperature of the gas monoatomic ideal gas
when the bubble is at the bottom is T0, the height of the liquid (b) CP + CV is larger for a diatomic ideal gas than for a
is H and the atmospheric pressure is p0. (Neglect surface monoatomic ideal gas
tension) C
(c) P is larger for a diatomic ideal gas than for a
CV
monoatomic ideal gas
(d) CP - CV is larger for a diatomic ideal gas than for a
monoatomic ideal gas
12. The figure shows the p–V plot an ideal gas taken
8. As the bubble moves upwards, besides the buoyancy through a cycle ABCDA. The part ABC is a semi–
force the following forces are acting on it (2008) circle and CDA is half of an ellipse. Then, (2009)
(a) Only the force of gravity
(b) The force due to gravity and the force due to the
pressure of the liquid.
(c) The force due to gravity, the force due to the
pressure of the liquid and the force due to viscosity
of the liquid
(d) The force due to gravity and the force due to
viscosity of the liquid
9. When the gas bubble is at a height y from the bottom,
its temperature is (2008) (a) The process during the path A → B is isothermal
2/5 (b) heat flows out of the gas during the path
 p0 + ρ gH 
(a) T0   B→C→D
 p + ρ gy
 0   (c) work done during the path A → B → C is zero
2/5
 p0 + ρ g ( H − y )  (d) positive work is done by the gas in the cycle
(b) T0  
 p0 + ρ gH
ABCDA
 
3/5
 p0 + ρ gH 
(c) T0  
 p + ρ gy
 0  
3/5
 p0 + ρ g ( H − y ) 
(d) T0  
 p0 + ρ gH
 
KINETIC THEORY OF GASES & THERMODYNAMICS 101

13. One mole of an ideal gas in initial state A undergoes a 19. One mole of a monatomic ideal gas is taken along two
cyclic process ABCA, as shown in the figure. Its cyclic processes E → F → G → E and E → F → H →
pressure at A is p0. E as shown in the p–V diagram.
Choose the correct option(s) from the following The process involved are purely isochoric, isobaric,
(2010) isothermal or adiabatic.

(a) Internal energies at A and B are the same


(b) Work done by the gas in process AB is p0V0 ln 4
Match the paths in List I with the magnitudes of the
(c) Pressure at C is p0/4
work done in List II and select the correct answer using
(d) Temperature at C is T0/4 the codes given below the lists. (2013)
14. A diatomic ideal gas is compressed adiabatically to List-I List-II
P. G → E 1. 160 p0V0 ln 2
1
of its initial volume. If the initial temperature of Q. G → H 2 36 p0V0
32
R. F → H 3 24 p0V0
the gas is Ti (in kelvin) and the final temperature is aTi,
S. F → G 4 31 p0V0
the value of a is (2010)
Codes
15. A real gas behaves like an ideal gas if its (2010) P Q R S
(a) pressure and temperature are both high (a) 4 3 2 1
(b) pressure and temperature are both low (b) 4 3 1 2
(c) pressure is high and temperature is low (c) 3 1 2 4
(d) pressure is low and temperature is high (d) 1 3 2 4
16. 5.6 L of helium gas at STP is adiabatically compressed
to 0.7 L. Taking the initial temperature to be T1, the
work done in the process is (2011)
9 3
(a) RT1 (b) RT1
8 2
15 9
(c) RT1 (d) RT1
8 2
17. A mixture of 2 moles of helium gas (atomic mass = 4
amu) and 1 mole of argon gas (atomic mass = 40 amu)
is kept at 300 K in a container. The ratio of the rms
 v ( helium ) 
speeds  rms
 v ( argon ) 
is (2012)
 rms 
(a) 0.32 (b) 0.45
(c) 2.24 (d) 3.16
18. Two non-reactive monoatomic ideal gases have their
atomic masses in the ratio 2 : 3. The ratio of their
partial pressures, when enclosed in a vessel kept at a
constant temperature, is 4 : 3. The ratio of their
densities is : (2013)
(a) 1 : 4 (b) 1 : 2
(c) 6 : 9 (d) 8 : 9
KINETIC THEORY OF GASES & THERMODYNAMICS 102

Use the following passage, solve Q. 20 to Q. 21 23. An ideal monoatomic gas is confined in a horizontal
cylinder by a spring loaded piston (as shown in the
In the figure a container is shown to have a movable (without
figure). Initially the gas is at temperature T1, pressure
friction) piston on top. The container and the piston are all
P1 and volume V1 and the spring is in its relaxed state.
made of perfectly insulating material allowing no heat
The gas is then heated very slowly to temperature T2,
transfer between outside and inside the container. The
pressure P2 and volume V2. During this process the
container is divided into two compartments by a rigid
piston moves out by a distance x. Ignoring the friction
partition made of a thermally conducting material that allows
between the piston and the cylinder, the correct
slow transfer of heat. The lower compartment of the container
statement(s) is (are) (2015)
is filled with 2 moles of an ideal monoatomic gas at 700 K
and the upper compartment is filled with 2 moles of an ideal
diatomic gas at 400 K. The heat capacities per mole of an ideal
3 5
monoatomic gas are CV = R , CP = R , and those for an
2 2
5 7 (a) If V2 = 2V1 and T2 = 3T1, then the energy stored in
ideal diatomic gas are CV = R , CP = R .
2 2 1
the spring is P1V1
4
(b) If V2 = 2V1 and T2 = 3T1, then the energy in internal
energy is 3P1V1
(c) If V2 = 3V1 and T2 = 4T1, then the work done by the
7
gas is P1V1
3
20. Consider the partition to be rigidly fixed so that it does
(d) If V2 = 3V1 and T2 = 4T1, then the heat supplied to
not move. When equilibrium is achieved, the final
temperature of the gases will be (2014) 17
the gas is P1V1
(a) 550 K (b) 525 K 6
(c) 513 K (d) 490 K 24. Heat given to process is positive, match the following
21. Now consider the partition to be free to move without option of column I with the corresponding option of
friction so that the pressure of gases in both column II. (2015)
compartments is the same. Then total work done by the
gases till the time they achieve equilibrium will be
(2014)
(a) 250 R (b) 200 R
(c) 100 R (d) –100 R
22. A thermodynamic system is taken from an initial state
i with internal energy Ui=100 J to the final state f along
two different paths iaf and ibf, as schematically shown
in the figure. The work done by the system along the
paths af, ib and bf are Waf =200 J, Wib=50J and Column-I Column-II
Wbf=100 J respectively. The heat supplied to the (a) JK (p) ∆ W > 0
system along the path iaf, ib and bf are Qiaf, Qib and Qbf (b) KL (q) ∆ Q < 0
respectively. If the internal energy of the system in the (c) LM (r) ∆W<0
state b is Ub=200 J and Qiaf= 500 J, the ratio Qbf/Qib is (d) MJ (s) ∆Q>0
(2014)
KINETIC THEORY OF GASES & THERMODYNAMICS 103

25. Column-I contains a list of processes involving 26. One mole of a monatomic ideal gas is taken through a
expansion of an ideal gas. Match this with Column-II cycle ABCDA as shown in the p–V diagram.
describing the thermodynamic charge during this Column- II gives the characteristics involved in the
process. Indicate your answer by darkening the cycle. Match them with each of the processes given in
appropriate bubbles of the 4 × 4 matrix given in the Column-I. (2015)
ORS. (2015)

Column-I Column-II
(a) An insulated container (p) The
two chambers separated temperature
by a valve. Chamber I of the gas
contains an ideal gas and decreases
the Chamber II has
Column-I Column-II
vacuum. The valve is
(a) Process A → B (p) Internal energy decreases
opened.
(b) Process B → C (q) Internal energy increases
(c) Process C → D (r) Heat is lost
(d) Process D → A (s) Heat is gained

(b) An ideal monoatomic gas (q) The (t) Work is done on the gas
expands to twice its temperature 27. A gas is enclosed in a cylinder with a movable
original gas increases or of the gas frictionless piston. Its initial thermodynamic state at
volume such that its increases pressure Pi = 105 Pa and volume Vi = 10-3 m3 changes
pressure remains to a final state at Pf = (1/32) × 105 Pa and Vf = 8 × 10-3
1 m3 in an adiabatic quasi-static process, such that P3V5
constants. p ∝ ,
V2 = constant. Consider another thermodynamic process
where V is the volume of that brings the system from the same initial state to the
the gas same final state in two steps: an isobaric expansion at
(c) An ideal monoatomic gas (r) The gas Pi followed by an isochoric (isovolumetric) process at
expands to twice its loses heat volume Vf. The amount of heat supplied to the system
original volume such that in the two-step process is approximately (2016)
1 (a) 112 J (b) 294 J
its pressure p ∝ , (c) 588 J (d) 813 J
V 4/3
28. One mole of a monatomic ideal gas undergoes an
where V is its volume.
adiabatic expansion in which its volume becomes eight
(d) An ideal monoatomic gas (s) The gas
times its initial value. If the initial temperature of the
expands such that its gains heat
gas is 100 K and the universal gas constant R = 8.0 J
pressure p and volume V
mol-1 K-1, the decrease in its internal energy, in Joule,
follows the behaviour
is (2018)
shown in the graph.
KINETIC THEORY OF GASES & THERMODYNAMICS 104

29. One mole of a monatomic ideal gas goes through a 31. In a thermodynamic process on an ideal monoatomic
thermodynamic cycle, as shown in the volume versus gas, the infinitesimal heat absorbed by the gas is given
temperature (V-T) diagram. The correct statement(s) by TdX, where T is temperature of the system and dX
is/are: [R is the gas constant] (2019) is the infinitesimal change in a thermodynamic
quantity X of the system. For a mole of monatomic
3 T  V 
=
ideal gas X Rn   + Rn   . Here, R is gas
2  TA   VA 
constant, V is volume of gas. TA and VA are constants.
The List-I below gives some quantities involved in a
process and List-II gives some possible values of these
quantities. (2019)
List-I List-II
(I) Work done by the system (P) 1
RT0 n2
in process 1 → 2 → 3 3
(a) Work done in thisthermodynamic cycle (II) Change in internal (Q) 1
1 energy in process RT
(1 → 2 → 3 → 4 → 1) is | W |= RT0 3
2 1→ 2→ 3
(b) The ratio of heat transfer during possesses (III) Heat absorbed by the (R) RT0
Q 5 system in process
1 → 2 and 2 → 3 is 1→ 2 = 1→ 2→ 3
Q2→3 3
(IV) Heat absorbed by the (S) 4
(c) The above thermodynamic cycle exhibits only RT0
isochoric and adiabatic processes. system in process 1 → 2 3
(d) The ratio of heat transfer during processes (T) 1
RT0 ( 3| n 2 )
Q 1 3
1 → 2 and 3 → 4 is 1→ 2 =
Q3→ 4 2 (U) 5
RT0
30. A mixture of ideal gas containing 5 moles of 6
monatomic gas and 1 mole of rigid diatomic gas is If the procession one mole of monatomic ideal gas is
initially at pressure P0, volume V0 and temperature T0. as shown in the TV-diagram with P0V0 = RT0, the
If the gas mixture is adiabatically compressed to a correct match is,
volume V0/4, then the correct statement(s)/are,

(Given 21.2 = 2.3;23.2 = 9.2; R is gas constant) (2019)

(a) The final pressure of the gas mixture after


compression is in between 9P0 and 10P0.
(b) The average kinetic energy of the gas mixture after
compression is in between 18RT0 and 19RT0
(c) Adiabatic constant of the gas mixture is 1.6
(d) The work |W| done during the process is 13RT0
(a) I → P, II → R, III → T, IV → S
(b) I → P, II → T, III → Q, IV → T
(c) I → S, II → T, III → Q, IV → U
(d) I → P, II → R, III → T, IV → P
KINETIC THEORY OF GASES & THERMODYNAMICS 105

32. Consider a gas of triatomic molecules. The molecules 35. A thermally insulating cylinder has a thermally
are assumed to be triangular and made of massless insulating and friction less movable partition in the
rigid rods whose vertices are occupied by atoms. The middle, as shown in the figure below. On each side of
internal energy of a mole of the gas at temperature T the partition, there is one mole of an ideal gas, with
is: (2020) specific heat at constant volume, CV = 2 R. Here, R is
the gas constant. Initially, each side has a volume V0
and temperature T0 . The left side has an electric
heater, which is turned on at very low power to
3 transfer heat Q to the gas on the left side. As a result,
(a) RT (b) 3 RT the partition moves slowly towards the right reducing
2
5 9 the right-side volume to V0 / 2. Consequently, the gas
(c) RT (d) RT
2 2 temperatures on the left and the right sides become TL
33. An ideal gas undergoes a four-step cycle as shown in
and TR , respectively. Ignore the changes in the
the P − V diagram below. During this cycle, heat is
absorbed by the gas in (2021) temperatures of the cylinder, heater and the partition.
(2021)

(a) steps 1 and 2 (b) steps 1 and 3 Q


(c) steps 1 and 4 (d) steps 2 and 4 The value of is
RT0
34. A thermally insulating cylinder has a thermally
insulating and frictionless movable partition in the (a) 4(2 2 + 1) (b) 4(2 2 − 1)
middle, as shown in the figure below. On each side of (c) (5 2 + 1) (d) (5 2 − 1)
the partition, there is one mole of an ideal gas, with
specific heat at constant volume, CV = 2 R. Here, R is
the gas constant. Initially, each side has a volume V0
and temperature T0 . The left side has an electric heater,
which is turned on at very low power to transfer heat
Q to the gas on the left side. As a result, the partition
moves slowly towards the right reducing the right-side
volume to V0 / 2. Consequently, the gas temperatures
on the left and the right sides become TL and TR ,
respectively. Ignore the changes in the temperatures of
the cylinder, heater and the partition. (2021)

TR
The value of is
T0
(a) 2 (b) 3
(c) 2 (d) 3
KINETIC THEORY OF GASES & THERMODYNAMICS 106

Find Answer Key and Detailed Solutions at the end of this book

KINETIC THEORY OF GASES &


THERMODYNAMICS
Please share your valuable feedback by
scanning the QR code.
ANSWER KEY 107

Answer Key
CHAPTER -14 THERMAL PHYSCIS

EXERCISE - 1 : EXERCISE - 2:
BASIC OBJECTIVE QUESTIONS ADVANCED OBJECTIVE QUESTION

DIRECTION TO USE -
DIRECTION TO USE -
Scan the QR code and check detailed solutions.
Scan the QR code and check detailed solutions.

1. (b) 2. (a) 3. (b) 4. (a) 1. (a) 2. (8.33) 3. (a) 4. (b)


5. (b) 6. (d) 7. (c) 8. (a)
5. (c) 6. (b) 7. (a) 8. (a)
9. (b) 10. (a) 11. (c) 12. (a) 9. (a) 10. (c) 11. (d) 12. (d)
13. (b) 14. (b) 15. (c) 16. (d) 13. (a) 14. (b) 15. (a)
17. (a) 18. (d) 19. (3.00) 20. (1.50)
21. (8.00) 22. (1.00) 23. 5.00 24. (a)
25. (c) 26. (b) 27. (d) 28. (b)
29. (d) 30. (c) 31. (b) 32. (8.00)
33. (885.00) 34. (1.00) 35. (9.00)
36. (270.00) 37. (5.8)
38. (20.26) 39. (0.5) 40. (40.00) 41. (6.00)
42. (b) 43. (b) 44. (a) 45. (b)
46. (c) 47. (212.50) 48. (60.00) 49. (2.00)
50. (2.00) 51. (3.00) 52. (4.00) 53. (c)
54. (b) 55. (c) 56. (c) 57. (c)
58. (a) 59. (b) 60. (b) 61. (b)
62. (b) 63. (b) 64. (a) 65. (c)
66. (b) 67. (d) 68. (a) 69. (d)
70. (b) 71. (c) 72. (d) 73. (2.00)
74. (25.00) 75. (28.00) 76. (5.00) 77. (2.00)
78. (6.00) 79. (28.00) 80. (0.92)
ANSWER KEY 108

CHAPTER -14 THERMAL PHYSCIS

EXERCISE - 3: EXERCISE - 4:
ADVANCED OBJECTIVE QUESTION PREVIOUS YEARS JEE ADVANCED QUESTIONS

DIRECTION TO USE - DIRECTION TO USE -


Scan the QR code and check detailed solutions. Scan the QR code and check detailed solutions.

1. (a) 2. (c) 3. (c) 4. (d) 1. (    2 s ) 2. (8.00) 3. (3.00)


5. (c) 6. (c) 7. (d) 8. (d)
4. (b, c, d) 5. (b) 6. (b) 7. (270.00)
9. (c) 10. (b) 11. (a) 12. (a)
8. (a) 9. (b) 10. (a) 11. (a)
13. (a) 14. (d) 15. (b) 16. (a, b) 12. (c) 13. (d) 14. (b) 15. (a)
17. (a) 18. (a) 19. (a) 20. (a) 16. (d) 17. (c) 18. (c) 19. (a, d)
21. (b) 22. (c) 23. (b) 24. (b) 20. (9.00) 21. (9.00) 22. (a, c, d) 23. (c)
25. (b) 26. (a) 27. (b) 28. (d) 24. (a) 25. (b, d) 26. (2) 27. (a)
29. (a) 30. (c) 31. (a) 32. (d) 28. (b) 29. (9) 30. (4.00)
33. (b) 34. (a) 35. (b) 36. (2.00)
37. (b) 38. (d) 39. (b) 40. (c)
41. (c) 42. (b) 43. (d) 44. (b, d)
45. (a, d) 46. (a) 47. (a) 48. (a)
49. (A - p; B - q; C - s; D - r) 50. (b)
ANSWER KEY 109

Answer Key
CHAPTER - 15 KINETIC THEORY OF GASES & THERMODYNAMICS

EXERCISE - 1 : EXERCISE - 2 :
BASIC OBJECTIVE QUESTIONS PREVIOUS YEARS JEE MAIN QUESTIONS

DIRECTION TO USE - DIRECTION TO USE -


Scan the QR code and check detailed solutions. Scan the QR code and check detailed solutions.

1. (c) 2. (d) 3. (a) 4. (b) 1. (d) 2. (a) 3. (d) 4. (a)


5. (d) 6. (c) 7. (b) 8. (a) 5. (d) 6. (c) 7. (a) 8. (b)
9. (b) 10. (d) 11. (c) 12. (b) 9. (d) 10. (c) 11. (a) 12. (c)
13. (d) 14. (b) 15. (c) 16. (d) 13. (a) 14. (b) 15. (b) 16. (c)
17. (b) 18. (c) 19. (d) 20. (d) 17. (c) 18. (c) 19. (c)
21. (a) 22. (a) 23. (d) 24. (c) 20. (1.77 to 1.78) 21. (6.00) 22. (2.05)
25. (1050) 26. (c) 27. (b) 28. (b) 23. (b) 24.(d) 25. (d) 26. (b)
29. (b) 30.(c) 31. (a) 32. (c) 27. (b) 28. (a) 29. (b) 30. (c)
33. (c) 34.(d) 35.(a) 36. (a) 31. (400) 32. (3600) 33.(d) 34.(b)
37. (c) 38.(d) 39.(c) 40. (a) 35. (a) 36.(a) 37. (b) 38.(b)
41. (b) 42.(124) 43.(b) 44. (b) 39. (d) 40.(b) 41. (d) 42.(b)
45. (d) 46.(c) 47.(b) 48. (c) 43.(c) 44.(a) 45.(50.00) 46.(208)
49. (20.00) 50.(b) 51. (a) 52. (b) 47.(26.00) 48.(60.00) 49.(25.00)
53. (c) 54.(2.00) 55.(b) 56. (c) 50.(-113.00)
57. (a) 58.(b) 59.(a) 60. (b)
61. (d) 62.(d) 63.(b) 64. (208)
65. (160) 66.(c) 67. (a) 68. (c)
69. (b) 70.(c) 71. (a) 72. (c)
73. (a) 74.(a) 75. (c) 76. (c)
77. (c) 78.(a) 79. (a) 80. (b)
ANSWER KEY 110

CHAPTER - 15 KINETIC THEORY OF GASES & THERMODYNAMICS

EXERCISE - 3: EXERCISE - 4:
ADVANCED OBJECTIVE QUESTION PREVIOUS YEARS JEE ADVANCED QUESTIONS

DIRECTION TO USE - DIRECTION TO USE -


Scan the QR code and check detailed solutions. Scan the QR code and check detailed solutions.

1. (b) 2. (d) 3. (c) 4. (c) 1. (b) 2. (b) 3. (a) 4. (d)


5. (b) 6. (d) 7. (d) 8. (d) 5. (c) 6. (c) 7. (b) 8. (d)
9. (d) 10. (a) 11. (d) 12. (d) 9. (b) 10. (b) 11. (b,d) 12. (b,d)
13. (c) 14. (c) 15. (b) 16. (c) 13. (a,b) 14. (4.00) 15. (d) 16. (a)
17. (a) 18. (a) 19. (b) 20. (c) 17. (d) 18. (d) 19. (a) 20. (d)
21. (b) 22. (d) 23. (a) 24. (a) 21. (d) 22. (2.00) 23. (a,b,c)
25. (a) 26. (d) 27. (c) 28. (c) 24. (a – q; b – p,q; c – s; d – q,r)
29. (d) 30.(b) 31. (d) 32. (c) 25. (a – q; b – p, r; c – p, s; d – q, s)
33. (d) 34.(c) 35.(b) 36. (b) 26. (a – p,r,t; b – p,r; c – q, s; d – r,t)
37. (a) 38.(c) 39.(d) 40. (a) 27. (c) 28. (900) 29. (a,b)
41. (a) 42.(c) 43.(b,d) 44. (a,d) 30. (a,c,d) 31. (d) 32. (b)
45. (a,b,c) 46.(a,c,d) 47.(a,b,c) 48. (a,c) 33. (c) 34.(a) 35.(b)
49. (b,c) 50.(a,b,d) 51. (a,c,d) 52. (1.00)
53. (1.00) 54.(0.75) 55.(0.42) 56. (3.00)
57. (375) 58.(35.00) 59.(210) 60. (500)
61. (a) 62.(a) 63.(d) 64. (a)
65. (a) 66.(d) 67. (c)
68. (A - r; B - p; C - q; D- s)
69. ( P  c, Q  a, R  d, S  b )
70. ( P  4; Q  3; R  2; S  1 )
71. (c) 72. (d) 73. (b) 74. (c)
75. (b)
MASTER INDEX .
VOLUME 1:
Units and Measurements & Basic Mathematics
Motion in a Straight line
Motion in a Plane & Relative Motion
Laws of Motion & Friction

VOLUME 2:
Work, Energy and Power
Circular Motion
Centre of Mass, Momentum and Collision
Rotational Motion
Gravitation

VOLUME 3:
Mechanical Properties of Solids
Fluid Mechanics
Simple Harmonic Motion
Waves

VOLUME 4:
Thermal Physics
Kinetic Theory of Gases and Thermodynamics
Creating Impact at Scale

65 Million+
Monthly
Youtube Views

49 Million+ 40+ Countries


Monthly Where Students
Web + App Visitors Take Live Classes

25 Million+ 26 Million+
Monthly Active Hours of
Users Live Learning

25 43
Parents
Happy
Trust
Students
Vedantu
MILLION+ MILLION+
Our Achievers of 2021
JEE Advanced 2021

Prerak Kevin Abhinav Vedantu students ace


JEE ADV. AIR 35 JEE ADV. AIR 78 JEE ADV. AIR 156 JEE Advanced 2021
JEE MAIN AIR 243 JEE MAIN AIR 533 JEE MAIN AIR 512

Ankit Shrey
6% of IITs’ upcoming batch
will be from Vedantu

JEE ADV. AIR 172 JEE ADV. AIR 174


JEE MAIN AIR 252 JEE MAIN AIR 238

JEE Main 2021

903 students in
Top 10,000
Vaibhav Bajaj Hrishit B P Sunrit Roy K

AIR 35 AIR 113 AIR 139

78 869
students scored students scored
99.9+ PERCENTILE 99.9+ PERCENTILE

68 444
students in students in
Top AIR 1000 Top AIR 5000 Kushagra Ganesh C Iyer
Sharma
AIR 158 AIR 179

NEET (UG) 2021

AIR 23
Pavit
Online Long Term Course

Score 710 AIR 92


Anirudh
Online Crash Course

Score 705
Shivank
Online Crash Course

AIR 143 Score 700


1172 NEET
Qualifiers

CBSE Class 12 ISC Class 12 CBSE Class 10

Annmary
Aatman Upreti Shreya Roshan Aastha N Raj Gitanjali Rajulal Shreya Nigam Khushi Arora Anshika Singha
Santhosh
98.4% 98.4% 98.0% 99.8% 98.4% 99.8% 99.6% 99.4%

ICSE Class 10

M.D.Sriya Varshil J Patel Mohammad Y Devika Sajeev Sakshi Semwal Aloki Upadhyay Ishita Surana Saumya Gupta

99.4% 97.4% 97.2% 97.2% 97.2% 99.4% 99.2% 99.2%

#HereForRealAchievers

You might also like